McGraw-Hill Emergency medicine Exam Review (Tintinalli's)

Ace your homework & exams now with Quizwiz!

The correct answer is D. Trench foot Explanation: Trench foot occurs when feet are exposed to cold and wet conditions, but not freezing. It is most often seen in military situations. Frostbite is an injury due to freezing that could superficially resemble trench foot. Chilblains are inflammatory lesions that typically occur on the hands of women after cold exposure. Cold urticaria is a hypersensitivity reaction to cold temperatures. Panniculitis involves necrosis of subcutaneous fat due to prolonged cold, but not freezing temperatures, and occurs mainly in children and the thighs and buttocks of women horseback riders. 62% of users answered correctly.

(P2) A 40-year-old homeless alcoholic man presents to the ED complaining of foot numbness. He has been sleeping in a drainage culvert to stay out of the wind. Nighttime temperatures have been well above freezing, but it has rained steadily for the past 5 days. His feet are pale and numb. They develop severe burning pain on rewarming. What is the most likely diagnosis? A Chilblains B Cold urticaria C Frostbite D Trench foot E Panniculitis

The correct answer is A. If you see injected conjunctiva, quiet anterior chamber, a positive Seidel test on fluorescein stain, you should get a computed tomography (CT) to rule out intraocular foreign body, give intravenous antibiotics, a tetanus booster, and request an ophthalmology consultation. Explanation: The Seidel test is performed by brushing a moistened fluorescein strip across the anesthetized cornea then under blue light checking for streaming indicative of corneal violation from a foreign object. Black tissue imbedded in the sclera is suggestive of uveal tissue herniation through a rent in the sclera caused by a foreign body. Any suspicion of FB violation of the eye, even with no obvious evidence of injury, should be worked up with a CT or ultrasound. If the mechanism of injury is highly suspicious treat with intravenous antibiotics, a tetanus booster, and get a ophthalmic consultation. A negative Seidel's test with a superficial corneal abrasion does not require CT or intravenous antibiotics. Rust rings or foreign bodies located in the visual axis should not be removed by an emergency physician but be deferred to the ophthalmologist. Do not attempt to measure IOP if globe rupture from penetrating or blunt trauma is suspected. 57% of users answered correctly.

(P2) A 45-year-old man comes in with acute eye pain that occurred suddenly while grinding a metal sculpture without safety goggles. His vital signs and visual acuity are normal. Eyes are grossly normal except for redness in the affected eye and mild photophobia after topical anesthesia. What is the most appropriate diagnostic test and treatment for the following slit-lamp exam findings? A If you see injected conjunctiva, quiet anterior chamber, a positive Seidel test on fluorescein stain, you should get a computed tomography (CT) to rule out intraocular foreign body, give intravenous antibiotics, a tetanus booster, and request an ophthalmology consultation. B If you see injected conjunctiva, quiet anterior chamber and a punctate corneal abrasion in the temporal inferior quadrant on fluorescein stain with a negative Seidel test, you should give a tetanus booster, treat with topical antibiotics, and obtain an ophthalmology consult in 24 hours. C If you see injected conjunctiva, quiet anterior chamber, a superficial piece of metal at the visual axis with a rust ring, you should remove the piece with an eye spud, remove the rust ring with a burr, give topical antibiotics, a tetanus booster, and obtain ophthalmology follow-up in 24 hours. D If you see injected conjunctiva with a black tissue in the temporal sclera, quiet anterior chamber, negative fluorescein stain uptake, you should give topical antibiotics, a tetanus booster, and ophthalmology follow-up in 24 hours. E If you see injected conjunctiva with normal sclera, clear cornea, small amount of blood in the anterior chamber, check the IOP, give pain meds, cycloplegics, and referral for recheck by ophthalmology within 24 hours.

The correct answer is B. Black widow spider (Latrodectus mactans) Explanation: Black widow spider bites are typically felt by the victim and can rapidly progress from local pain to severe cramps in large muscle groups. Patients may have associated hypertension. Treatment consists of adequate analgesia and Latrodectus antivenom. In contrast, Brown recluse spider bites are often initially painless. The wound may progress from erythematous lesion to blistering and necrosis and leave a poorly healing ulcer. Associated systemic symptoms, including hemolysis and disseminated intravascular coagulation are rare. Hobo spider bites may present similarly to Brown recluse bites. Headache, nausea, and fatigue are the most commonly associated systemic symptoms. Bark scorpion venom contains a neurotoxin that causes local pain and paresthesias. Severe cases may demonstrate cranial nerve dysfunction, excessive motor activity, nausea, vomiting, tachycardia, and agitation. Yellow jacket stings are typically painful and cause a local erythematous reaction with swelling which can mimic cellulitis. In sensitized individuals, they can cause anaphylaxis. 63% of users answered correctly

(P2) A 47-year-old man was moving wooden logs at a construction site when he was bit on the hand by "some bug." The bite was immediately painful and is now red and swollen. His arm began to ache and he now presents hypertensive with severe back pain. Which of the following is the most likely culprit? A Bark scorpion (Centruroides exilicauda) B Black widow spider (Latrodectus mactans) C Brown recluse spider (Loxosceles reclusa) D Hobo spider (Tegenaria agrestis) E Yellow jacket (Hymenoptera)

The correct answer is D. Progesterone 10 mg/day for 10 days Explanation: Ovulatory dysfunctional uterine bleeding can be treated with oral contraceptives, nonsteroidal anti-inflammatory drugs (NSAIDs), or progesterone. The progesterone will decrease the number of available estrogen receptors and can be given 10 mg daily for 10 days. In nonpregnancy patients, ultrasound can be helpful to determine the size of the uterine and the characteristics of the endometrium but, since she is not pregnant and is hemodynamically stable, it can be deferred for outpatient evaluation. Intravenous estrogen can be used in the ED management of life-threatening hemorrhage in the nonpregnancy patient. Although this patient will need follow-up, C is the answer given the duration of bleeding. NSAIDs, not acetaminophen, are helpful in dysfunctional uterine bleeding by reducing blood flow and reducing dysmenorrhea. 51% of users answered correctly.

39-year-old female presents to the emergency department (ED) with persistent vaginal bleeding for 3 weeks. She normally has regular periods, but she has not had a period in the 2 months prior to this bleeding. She denies abdominal pain and states she is soaking about four pads per day. Her vitals are normal, her pregnancy test is negative, and her hematocrit is 40. Which of the following is the most appropriate next step in management? A Acetaminophen 600 mg every 6 hours B Follow up with her gynecologist in 2 weeks C Intravenous estrogen D Progesterone 10 mg/day for 10 days E STAT pelvic ultrasound

The correct answer is D. He is best treated with topical beta-blockers, steroids, and cycloplegics to reduce IOP; get a CT of the orbits and follow up with ophthalmology in 24 hours. Explanation: This patient presents with two complications from blunt injury to the eye: a traumatic hyphema and a blowout fracture of the orbit with entrapment. Traumatic hyphema occurs because of bleeding from a ruptured iris vessel. Treatment involves keeping the head elevated, paralyzing the iris with cycloplegics, topical prednisone, and lowering the IOP with topical beta-blockers. If IOP is elevated, carbonic anhydrase inhibitors (CAIs) are indicated EXCEPT in sickle cell anemia in which the CAI can lower the pH of the aqueous humor and cause sickling of the cells thereby clogging the outflow tract of the trabecular matrix. Intravenous mannitol may be used as an alternative. Hyphemas occupying greater than one-third of the anterior chamber should be seen by an ophthalmologist emergently. Re-bleeding occurs in 3-5 days in up to 30%. In general, patient with hyphemas occupying less than one-third of the anterior chamber can be referred to an ophthalmologist for close outpatient follow-up. Blowout fractures should be evaluated with CT to rule out sinus fracture and oral antibiotics are often recommended. All blowout fractures should be referred to ophthalmologist for a full dilated exam in the next 3 to 10 days to rule out retinal tears or detachment (CT or ultrasound may help). Immediate bedside consultation is not indicated unless the consult ophthalmologist prefers to admit hyphemas. 15% of users answered correctly.

A 12-year-old boy with sickle cell disease was hit in the eye with a fist. His visual acuity is normal. On physical exam he has mild periorbital edema, mild enophthalmos, and upward gaze diplopia, fundus is normal. Slit-lamp exam is shown (Figure 6-2). His IOP measures 30 mm Hg. What is the most appropriate treatment for this patient? A He is best treated with topical beta-blockers, steroids, cycloplegics, and intravenous acetazolamide to reduce IOP; get an orbital CT and immediate ophthalmic consultation. B He is best treated with topical beta-blockers, steroids, and cycloplegics, and intravenous acetazolamide to reduce IOP; get an orbital CT and ophthalmic follow-up within 24 hours. C He is best treated with topical beta-blockers, steroids, and cycloplegics to reduce IOP; get a CT of the orbits and immediate ophthalmology consultation. D He is best treated with topical beta-blockers, steroids, and cycloplegics to reduce IOP; get a CT of the orbits and follow up with ophthalmology in 24 hours. E He is best treated with topical cycloplegics and acetazolamide to reduce IOP and urgent ophthalmology consult.

The correct answer is D. HSV encephalitis and treatment with IV acyclovir Explanation: The presentation of HSV encephalitis can closely mirror bacterial meningitis and must be considered in any patient who presents with fever, altered mental status, and focal neurologic signs. HSV encephalitis can represent a primary infection or a reactivation and may occur in normal immunocompetent hosts. It has a predilection for the medial temporal and inferior frontal lobes. Factors leading to encephalitis are largely unknown. The proper treatment is intravenous acyclovir, and even with treatment, survivors have a high rate of persistent neurological deficits. Dosing should be reduced in patients with renal dysfunction at baseline. Space-occupying lesions should be ruled out by CT scan prior to lumbar puncture in a patient with focal neurologic signs. Brain abscess could present with seizure and signs of cerebral edema, but would typically be apparent on CT scan. Coccidiomycosis, cryptococcosis, and toxoplasmosis typically invade the CNS in the setting of immunosuppression. Furthermore, the proper first-line treatment for CNS cryptococcal disease would be with amphotericin. Related Topics: viral encephalitis 72% of users answered correctly.

A 14-year-old otherwise healthy Caucasian female presents to the ED with fever and altered mental status. Per her mother, she has been complaining of a progressive headache for 2 days, has had trouble walking, and has vomited multiple times. The patient has a temperature of 39.2°C. She is disoriented and agitated, and responds only to painful stimuli. No rash is detected. You initiate treatment. A CT scan of the brain does not show any focal lesions. In addition to bacterial meningitis, what other diagnosis and therapy should you consider? A Brain abscess and consult neurosurgery for drainage B Coccidiomycosis meningitis and treatment with IV amphotericin C Cryptococcal meningitis and treatment with IV fluconazole D HSV encephalitis and treatment with IV acyclovir E Toxoplasmosis and treatment with pyrimethamine PO

The correct answer is D. Ultrasound Explanation: Bedside ultrasound can be very helpful in the identification of an inguinal hernia if the diagnosis remains in question. One study reported 100% sensitivity and 100% specificity of bedside emergency ultrasound for the diagnosis of groin hernia. Sonography has many advantages but is operator and body habitus dependent. The primary role of ultrasound is the identification of the hernia itself. Other roles include identification of blood flow by color Doppler to exclude strangulation, particularly prior to reduction attempts. Ultrasound is most useful for diagnosis in children and pregnant women. CT is the best radiographic test for diagnosis, and can identify uncommon hernia types (e.g., Spigelian or obturator) and demonstrate incarceration and strangulation. Related Topics:herniahernia, inguinal 78% of users answered correctly.

A 15-year-old female who works after school at a day care presents with an inguinal mass. Her sister was recently diagnosed with cervical cancer, and she is worried she may have cancer. She noticed the mass after lifting an infant at school. It is soft and reducible. On exam, you palpate a mass in the inguinal region on the right. What should occur NEXT to solidify a diagnosis and reassure the patient? A CBC B CT scan C Rectal exam D Ultrasound E Urinalysis

The correct answer is B. Soak the teeth in Hank's balanced salt solution for 30 minutes, then push the teeth firmly into the tooth sockets, securing them to the surrounding teeth with periodontal dressing and follow up with a dentist within 24 hours. Explanation: Total avulsion of the tooth requires reimplantation as soon as possible and can be successful if done within 3 hours of avulsion. Teeth should be kept moist with the mouth, or suspended in Hank's balanced salt solution, sterile normal saline, or milk in transport. If the teeth have been dry for 20-60 minutes, they should be soaked in Hank's balanced salt solution for 30 minutes. The socket can be gently rinsed with saline for clot removal but as little manipulation of the socket as possible should occur. Radiography is not necessary if physical exam does not suggest alveolar fracture. The teeth should be placed in the sockets with gentle firm pressure and securing them to the surrounding teeth with periodontal dressing and follow-up with a dentist within 24 hours. 36% of users answered correctly.

A 16-year-old comes to the ED after having been punched in the mouth 30 minutes ago. He is holding his two front teeth in his hand, which are complete from crown to root. His physical exam is normal except for two avulsed front teeth, minimal gingival ecchymosis, and a stable alveolar ridge. What is the best course of action? A Rinse the dry teeth under sterile saline, rinse the socket thoroughly cleaning out any debris, and then immediately reimplant the teeth and splint. B Soak the teeth in Hank's balanced salt solution for 30 minutes, then push the teeth firmly into the tooth sockets, securing them to the surrounding teeth with periodontal dressing and follow up with a dentist within 24 hours. C Suspend the teeth in Hank's balanced salt solution, sterile normal saline or milk, get a CT scan to rule out alveolar ridge fracture, give pain medications, and have the patient follow up with a dentist within 24 hours. D Suspend the teeth in Hank's balanced salt solution, sterile normal saline or milk, give pain medications, and have the patient follow up with a dentist within 24 hours. E Washing the teeth with sterile normal saline, gently place the teeth and the sockets and secure the teeth to the surrounding teeth with periodontal dressing, and follow up with a dentist within 24 hours.

The correct answer is B. Obtain a pelvic ultrasound Explanation: The patient is at risk of ovarian torsion given the history of ovarian cyst and pain that started after exercise; therefore, an ultrasound should be performed in the ED. Ultrasound with Doppler sonography is the primary imaging modality for suspected ovarian torsion. Ibuprofen and oral contraception can be used to treat dysfunctional uterine bleeding and dysmenorrhea. Culture swabs would be indicated if the patient had purulent discharge and cervical motion tenderness. 78% of users answered correctly.

A 16-year-old female with history of ovarian cysts presents with right lower quadrant pain for 3 hours that started after dance class. She states she has been seen for similar pain in the past and was told it was from an ovarian cyst. Her human chorionic gonadotropin (hCG) is negative and her pelvic exam is remarkable for right adnexal tenderness to palpation. Which of the following is the most appropriate next step in ED management? A Obtain a gynecology consult in the ED B Obtain a pelvic ultrasound C Obtain vaginal swabs for gonorrhea and chlamydia D Prescribe ibuprofen 600 mg three times a day for pain and outpatient referral to a gynecologist E Prescribe oral contraceptives to manage symptoms

The correct answer is D. Ask patient to take slow deep breaths with her mouth open Explanation: Given her age, normal physical examination, and negative workup, this patient is most likely hyperventilated and had a near syncopal episode. Asking her to take slow deep breaths with her mouth open might reproduce her symptoms and provide her with insight into her breathing pattern in response to stress. This patient does not require admission, although she might benefit from a referral for psychological counseling. In general, patients with recurrent syncope associated with falls should undergo tilt-table testing. EP testing is indicated for patients with documented dysrhythmia, preexcitation, or underlying heart disease. 53% of users answered correctly.

A 16-year-old girl presents to the ED with near syncope without antecedent headache, chest pain, shortness of breath, abdominal pain, or weakness. The patient was arguing with a boyfriend and reports feeling her extremities and lips tingling before she "practically passed out." She admits to social stressors but denies suicidal or homicidal ideation. Vital signs are BP 120/80, HR 72, RR 18, T 98.6, room air saturation 100%. Initial physical examination is normal. She had a prior ED visit for anxiety last month. Her urine pregnancy test is negative, her hemoglobin and hematocrit are normal, and an ECG shows normal sinus rhythm. What is the next MOST appropriate step in management? A Admit and arrange inpatient psychiatric consult B Arrange outpatient electrophysiologic (EP) testing C Arrange outpatient tilt-table testing D Ask patient to take slow deep breaths with her mouth open E Discharge immediately

The correct answer is C. Antipyretics and avoidance of contact sports for 4 weeks Explanation: This presentation is suggestive of infectious mononucleosis caused by EBV. Treatment is usually supportive and rarely requires admission. Patients may develop splenomegaly, and should be advised to avoid contact sports for at least 4 weeks to avoid splenic rupture. Acyclovir is active against EBV, but is only effective against oral hairy leukoplakia in HIV. Amoxicillin can be used for streptococcal pharyngitis, but when mistakenly used for EBV will cause a diffuse morbilliform rash. The patient does not have signs of sepsis, is not at risk for endocarditis, and has a presentation classic for EBV and does not warrant hospitalization. Dexamethasone is often utilized in streptococcal pharyngitis, but increased complications are noted in the setting of EBV. It should be avoided unless the patient has airway compromise. 82% of users answered correctly.

A 16-year-old male presents complaining of fever, fatigue, and sore throat for the previous 2 days. His temperature is 102.1°C, blood pressure is 120/80, and heart rate is 98. On exam, you notice mild tonsillar erythema and significant exudates. He also has diffuse lymphadenopathy and a prominent spleen. A complete blood cell count is within the normal range but shows a lymphocytosis with atypical lymphocytes. What is the proper treatment? A Acyclovir B Amoxicillin C Antipyretics and avoidance of contact sports for 4 weeks D Blood cultures, broad-spectrum antibiotics, and admission E Dexamethasone

The correct answer is D. Recombinant factor VIII Explanation: Any patient with hemophilia who presents with a new headache, lateralizing neurologic signs, decreased level of consciousness, and/or head trauma requires immediate factor replacement followed by CT of the brain. This patient has hemophilia A and therefore requires an infusion of factor VIII—not factor IX. Surely consultation of a neurosurgeon will be required once the bleeding is addressed. The patient's hypertension and bradycardia are signs of increased intracranial pressure—known as the Cushing response. This is primarily addressed by treating the underlying cause (in this case, intracranial bleeding). 55% of users answered correctly.

A 19-year-old man with hemophilia A is brought into the emergency department for evaluation of unresponsiveness after a fall down six steps today. His blood pressure is 187/101 mm Hg, his heart rate is 46 beats/min—otherwise his vital signs are within normal limits. His Glasgow Coma Score is 6. Paramedics brought him in intubated with full spinal immobilization. After the primary and secondary surveys, which of the following is the next most appropriate step in the management of this patients? A Computed tomography (CT) of the brain B Consultation of a neurosurgeon C Nicardipene D Recombinant factor VIII E Recombinant factor IX

The correct answer is E. Transfusion-related acute lung injury is usually caused by the transfusion of FFP and/or platelets. Explanation: Transfusion-related acute lung injury occurs infrequently. It is usually caused by the infusion of platelets and/or FFP; it rarely occurs after an infusion of PRBCs alone. Each unit of PRBCs should raise the hemoglobin concentration by 1 g/dL (and the hematocrit by 3%). Citrate, used to preserve PRBCs, may cause hypocalcemia by chelation during a massive transfusion. Yersinia enterocolitica is the most likely bacterium to be transmitted during a blood transfusion, as it readily grows in refrigerated blood. The transfusion of at least 10 units of PRBCs with a 24-hour period constitutes a massive transfusion. 36% of users answered correctly.

A 19-year-old woman, struck by a car, is brought to your emergency department. Her vital signs are as follows: blood pressure 65/palpation; pulse 144 beats/min; respirations 34 breaths/min; and pulse oximetry 99% by face mask. Physical examination reveals full spinal immobilization, a Glasgow coma score of 12, and a traumatic amputation of her left lower extremity with uncontrolled bleeding. In the resuscitation of this patient, you administer 13 units of PRBCs and several units of fresh frozen plasma (FFP) and platelets. Which of the following is TRUE? A Each unit of PRBCs should raise the hemoglobin concentration by 5 g/dL. B Hypercalcemia may result from the transfusion of large quantities of PRBCs. C Staphylococcus aureus is the most likely bacterium to be transmitted during a blood transfusion. D Transfusing more than 20 units of PRBCs in a 24-hour period constitutes a massive transfusion. E Transfusion-related acute lung injury is usually caused by the transfusion of FFP and/or platelets.

The correct answer is C. Discharge home with diabetic instructions and follow-up in 24-48 hours. Explanation: Diabetes mellitus may be diagnosed in three ways: symptoms of diabetes plus a random glucose ≥200 mg/dL, a fasting plasma glucose ≥126 mg/dL, or a 2-hour plasma glucose level ≥200 mg/dL during an oral glucose tolerance test. Patients who are nonpregnant adults, stable, do not have life-threatening metabolic decompensation, and with no comorbid illnesses are safe to be discharged with appropriate diabetes instructions with follow-up within 1-2 days. Although metformin is a relatively safe drug to start, it is not recommended to start in the ED. 40% of users answered correctly.

A 20-year-old male presents to the ED complaining of several weeks of fatigue. He has not had any fevers, chills, cold symptoms, or cough. He has never traveled out of the country. His vital signs are all within normal range. On review of systems, he does state that he has frequent urination and has been drinking more fluids than normal because he feels that he is always thirsty. He ate lunch an hour ago and his finger stick is 279. His serum acetone is negative and his anion gap, potassium, bicarbonate, and sodium levels are all within normal limits. His serum glucose is 283. He receives 2 L of NS in the ED and 6 units of subcutaneous insulin. His repeat glucose is 119. Which of the following is the most appropriate step in this patient's care? A Admit to the hospital for new onset diabetes rehydration and insulin. B Admit to the hospital for fasting glucose and oral glucose tolerance test (OGTT). C Discharge home with diabetic instructions and follow-up in 24-48 hours. D Discharge home with metformin and follow-up with 24-48 hours. E Schedule for outpatient fasting glucose and oral glucose tolerance test (OGTT).

The correct answer is A. If clinical exam suggests an isolated nasal fracture, incise the lesion and pack the nares. Explanation: This picture shows a septal hematoma for which urgent incision and drainage, followed by nasal packing, is indicated. Imaging studies are usually not indicated for isolated nasal fractures since the fracture is best assessed by clinical exam. Closed reduction of an acute nasal fracture without significant edema can be attempted if there is fracture with obvious displacement or misalignment, followed by packing and external splinting and close follow-up. 62% of users answered correctly.

A 20-year-old man comes in after receiving a blow to the face during a basketball game. He complains of pain in his nose. His exam is shown in Figure 6-3. What is the appropriate management? A If clinical exam suggests an isolated nasal fracture, incise the lesion and pack the nares. B If clinical exam suggests an isolated nasal fracture, perform closed reduction of the above lesion with a blunt elevator, then send for CT to assess extent of fracture and alignment. C Order CT and if no fracture appreciated, send home with vasoconstrictors and analgesics to follow up with otolaryngology in 2-48 hours. D Order a CT of the facial bones to assess extent of nasal fracture. E Order plain films of the face including waters view to assess for nasal fracture.

The correct answer is C. Lorazepam Explanation: Patients with acute sympathetic crisis may not have signs of end-organ damage to the brain, heart, or kidneys, but treatment is nevertheless indicated to avoid complications to those organs. The first-line treatment for an acute sympathetic crisis due to cocaine or methamphetamine use is an IV benzodiazepine such as lorazepam or diazepam. Monitor the patient for respiratory depression or sedation. If benzodiazepine therapy is unsuccessful, nitroglycerin, calcium channel blockers, or phentolamine can be used. The text recommends nitroglycerin therapy, followed by nicardipine. Phentolamine is a potent alpha-1 and alpha-2 blocking properties, which beta-blockers are not recommended because of the risk of increased alpha effects in the presence of unopposed beta-blockade. Labetalol has weak alpha-blockage and strong beta-blocking effects, in a ratio of 1 to 7. Esmolol has shorter acting beta-blocking effects but is still not recommended for acute sympathetic crisis. 68% of users answered correctly.

A 20-year-old woman presents with palpitations and anxiety 2 hours after smoking crack cocaine for the first time. She denies headache, chest pain, or shortness of breath. Vital signs are BP 224/122, HR 110, RR 22, T 98.9, room air saturation of 100%. The patient is alert and diaphoretic, but otherwise has a normal physical examination. Which of the following medications is the most preferred initial treatment for this patient? A Esmolol B Labetalol C Lorazepam D Nicardipine E Phentolamine

The correct answer is C. Obtain a transvaginal ultrasound to evaluate for a tubo-ovarian abscess. Explanation: Women with suspected PID and asymmetric pelvic findings should have an ultrasound to evaluate for tubo-ovarian abscess. CT and MRI can be used to evaluate for PID and exclude other causes for pain, but, in this case, ultrasound is the more appropriate imaging modality. Antibiotics and outpatient follow-up may be appropriate for hemodynamically stable patients with PID, but admission is required for all patients with tubo-ovarian abscess. If a patient has a history and clinical findings concerning for PID, antibiotics should be started immediately before cervical cultures return. 60% of users answered correctly.

A 21-year-old female presents with lower abdominal pain for 3 days. Her last menstrual period was 1 week ago, she is sexually active with one partner but does not use condoms. Her physical exam includes a temperature of 100.4, heart rate of 112, and blood pressure of 104/45. She is tender to palpation in the right and left lower quadrants. Her bimanual is notable for cervical motion tenderness and right adnexal tenderness. Her hCG is negative. The next best step in management is: A Obtain a noncontrast abdominal/pelvic CT scan. B Obtain a urine analysis. C Obtain a transvaginal ultrasound to evaluate for a tubo-ovarian abscess. D Send cervical cultures and, if positive, start her on antibiotics for pelvic inflammatory disease (PID). E Start her on antibiotics for presumed PID and arrange outpatient follow-up with gynecology.

The correct answer is B. Direct pressure Explanation: Management of this patient requires only direct pressure, as patients with platelet counts exceeding 50,000/mm3 infrequently necessitate other specific treatments. Clinicians should consider steroids in patients with platelet counts between 20,000/mm3 and 30,000/mm3 or with life-threatening bleeding—neither of which this patient has. Clinicians should strongly consider platelet infusion in patients with life-threatening bleeding and/or a count less than 10,000/mm3—again neither of which this patient has. Similarly, infusion of polyclonal antibody should be reserved for patients with a very low platelet count and/or significant or life-threatening bleeding. There is no specific role for cryoprecipitate in the care of the patient with idiopathic thrombocytopenic purpura. 79% of users answered correctly.

A 21-year-old man with idiopathic thrombocytopenic purpura presents to the emergency department for care of a cut to his face he sustained while shaving. His vital signs are normal, and the examination reveals bleeding from a small abrasion to his left cheek. Laboratory studies reveal a platelet count of 56,000/mm3. What is the appropriate management for this patient? A Cryoprecipitate infusion B Direct pressure C Oral steroids D Platelet infusion E Polyclonal antibody infusion

The correct answer is B. Chlamydia trachomatis Explanation: The most common sexually transmitted disease, and also the most common cause of cervicitis/urethritis, is C. trachomatis. This is followed by N. gonorrhoeae, which usually has a much more profuse, purulent discharge than C. trachomatis. Given high rates of coinfection, patients are often empirically treated for both infections. Uncomplicated C. trachomatis is treated with either azithromycin 1 g single dose or doxycycline 100 mg twice daily for 7 days. Uncomplicated N. gonorrhoeae cervicitis/urethritis is treated with ceftriaxone 125 mg intramuscularly. Single-dose azithromycin 2 g is an alternative. Fluoroquinolones are no longer recommended due to high rates of resistance. Sodium chloride microscopy is used to detect T. vaginalis or clue cells suggestive of bacterial vaginosis. Trichomonads are motile and therefore detectable for approximately 10-20 minutes after plating. T. vaginalis should be a consideration when first-line treatment for urethritis fails in male patients. Potassium hydroxide microscopy is utilized for hyphae of C. albicans. Human papillomavirus causes genital warts and is known to lead to cervical cancer. Other less common causes of urethritis are Ureaplasma urealyticum, Mycoplasma genitalium, HSV, and adenovirus. 63% of users answered correctly.

A 22-year-old female is complaining of vaginal discomfort and discharge for 2 days. She also complains of mild dysuria. On exam, you notice a thin yellowish discharge, a friable cervix, but no cervical motion or adnexal tenderness. Sodium chloride and potassium hydroxide microscopy slides are negative. What is the MOST likely cause? A Candida albicans B Chlamydia trachomatis C Human papillomavirus D Neisseria gonorrhoeae E Trichomonas vaginalis

The correct answer is C. Obtain a confirmatory test such as a CT scan. Explanation: The patient exhibits several features of drug-seeking behavior. Confirming the diagnosis of kidney stones with additional testing is appropriate in this situation. Hematuria can be factitious or menstrual and a witnessed or catheter specimen can be obtained if you are concerned that the patient is trying to obtain narcotics fraudulently. Nonaddictive pain medications should be offered to patients who state they have pain, but ibuprofen (Motrin) would not be the best choice given the patient's stated allergy. Agonist-antagonist opioids should not be given to patients in the emergency department who are suspected of being addicted to opioids, as they may precipitate withdrawal.

A 22-year-old female presents to the emergency department on a Tuesday evening with a complaint of kidney stone and blood in her urine. She has no records at your facility as she is visiting from out of town. She forgot to bring a prescription for oxycodone that her urologist gave to her and says she is allergic to ibuprofen (Motrin), which causes itching. The patient appears comfortable and the exam is normal but lab results show hematuria. Which is the BEST choice for treatment among those listed? A An agonist-antagonist opioid should be prescribed. B Give ibuprofen (Motrin); the patient is clearly lying about her allergies. C Obtain a confirmatory test such as a CT scan. D Refuse any pain medicines; the patient is demonstrating drug-seeking behavior. E The stated prescription should be refilled because the hematuria confirms the history of kidney stone.

The correct answer is E. Cover the dried exposed tooth with glass ionomer dental cement base and prescribe oral analgesics then follow up with a dentist in 24 hours. Explanation: This patient presents with a Ellis class II fracture, which accounts for 70% of tooth fractures. The treatment of Ellis class II (dentin exposed) is dry the fractured surface of the tooth and seal with glass ionomer dental cement to decrease dental contamination by oral bacteria and give 24 hours dental follow-up. In Ellis class III fracture with pulp exposed, the surface should be covered with calcium hydroxide base and then covered with glass ionomer dental cement as is Ellis II fractures. Oral analgesics are indicated. Topical analgesics should be avoided. No antibiotic treatment is necessary. 40% of users answered correctly.

A 22-year-old man complains of having tripped and hit his front tooth against the edge of the table. He has no complaints except for a chipped tooth with sensitivity to air passing over it. Physical exam shows his right incisor with a chip from the bottom third of the tooth (see Figure 6-5). It is exquisitely tender to palpation, no bleeding, no laxity, with normal overlying gingiva. What is the most appropriate management? A Give the patient oil of clove for topical anesthesia, prescribe oral analgesics, and follow up with dentist in 24 hours. B Give the patient oral analgesics and antibiotics and follow up with dentist in 24 hours. C Cover the dried exposed tooth with calcium hydroxide base, prescribe oral analgesics and antibiotics, and then follow up with dentist in 24 hours. D Cover the dried exposed tooth with glass ionomer dental cement base and prescribe oral analgesics and antibiotics then follow up with a dentist in 24 hours. E Cover the dried exposed tooth with glass ionomer dental cement base and prescribe oral analgesics then follow up with a dentist in 24 hours.

The correct answer is D. Place warm (38-40°C) gauze soaks on the right auricle, do not debride vesicles that form, incise and debride the subcutaneous blood, and place a sutured bolstered dressing on the left auricle. Explanation: This patient has both frostbite of his right ear and traumatic injury of his left ear. Frostbite injuries should be quickly rewarmed with aseptic warm (38-40°C) saline-soaked gauze with good adjuvant analgesia. Vesicles that form should not be debrided but allowed to reabsorb naturally. The left ear with a hematoma needs to be debrided to prevent necrosis of the cartilage and formation of the cauliflower ear. Incision and drainage with sutured bolster dressing has found to have best outcome. Antibiotics should be reserved for immunocompromised patients. Silversulfadiazine should not be used for burn care from the clavicle upward for cosmetic reasons as it can cause skin pigmentation changes. Sterile pledget bolster may be coated with nonsulfa antibiotic ointment. 46% of users answered correctly.

A 22-year-old man is brought in after being caught in an avalanche while back country skiing. Among his injuries, he has white nontender right auricle and a swollen ecchymotic left auricle and perforated left TM without hemotympanum. You should: A Place lukewarm (35-38°C) gauze soaks on the right auricle while you give IV antibiotics. Do not violate the skin of the left auricle. B Place lukewarm (35-38°C) gauze soaks on the right auricle while you aspirated the subcutaneous blood from the left auricle and give IV antibiotics. C Place warm (38-40°C) gauze soaks on the right auricle, debride the vesicles that form, incise and debride the subcutaneous blood from the left auricle, and place a sutured bolster dressing. D Place warm (38-40°C) gauze soaks on the right auricle, do not debride vesicles that form, incise and debride the subcutaneous blood, and place a sutured bolstered dressing on the left auricle. E Place warm (38-40°C) gauze soaks on the right auricle while you aspirated the subcutaneous blood from the left auricle and place a sutured bolster covered with silversulfadiazine and gentle dressing.

The correct answer is C. Bilirubin >20 Explanation: Patients with acute hepatitis require supportive treatment with pain management and fluid resuscitation. Consider admission for high-risk patients including the elderly and pregnant women. Admit those who have a bilirubin ≥20 mg/dL, prothrombin time 50% above normal, hypoglycemia, low albumin, or any signs of GI bleeding. Related Topics:hepatitisacute hepatitis 44% of users answered correctly.

A 22-year-old nonpregnant female presents with acute nausea, vomiting, abdominal pain, and jaundice. Her diagnostic tests reveal an AST of 1500 and ALT of 2000. Which factor will determine if the patient should be admitted? A ALT >1000 B AST >2000 C Bilirubin >20 D Glucose >200 E INR greater than 25% normal

The correct answer is B. Metronidazole 250 mg three times a day for 7 days is appropriate therapy. Explanation: Since bacterial vaginosis has been associated with spontaneous abortion, preterm labor, and premature rupture of membranes, pregnant women should be treated with metronidazole 250 mg PO three times a day for 7 days. Metronidazole 2 g single-dose therapy has the lowest efficacy; therefore, it should not be used in pregnant patients. The diagnosis of bacterial vaginosis is associated with a vaginal pH >4.5; thin, gray discharge; clue cells; and a positive whiff test. Cultures of Gardnerella are not useful since it is also found in normal vaginal flora. 53% of users answered correctly.

A 23-year-old female 10 weeks pregnant presents with vaginal irritation and a malodorous discharge. Pelvic exam demonstrates a thin gray discharge and the following wet mount is obtained (Figure 8-2). Which of the following statements is correct regarding her diagnosis? A Metronidazole 2 g single-dose therapy has the highest efficacy for treatment. B Metronidazole 250 mg three times a day for 7 days is appropriate therapy. C This diagnosis is not associated with spontaneous abortion and preterm rupture of membranes. D This diagnosis is also associated with a vaginal pH <4.5. E Vaginal cultures of Gardnerella confirm the diagnosis.

The correct answer is D. Obtain an obstetric consult in the ED Explanation: An obstetrician should be consulted for concern of an ectopic pregnancy with a hCG above the discriminatory zone and an empty uterus on transvaginal ultrasound. The discriminatory zone for transvaginal ultrasound is 1500 mIU/mL and transabdominal is 6000 mIU/mL. The patient should not be discharged without obstetric evaluation given her high risk for an ectopic pregnancy. Rhogam is not indicated since the patient is Rh positive. 61% of users answered correctly.

A 23-year-old female presents with lower abdominal cramping and vaginally bleeding for 2 days. Her last menstrual period was 6 weeks ago. Her hCG is 2000 mIU/mL and she is Rh positive. A transvaginal ultrasound shows no intrauterine pregnancy. Which of the following is the next best step in management? A Administer Rhogam 50 MIU IM B Discharge home with ectopic precautions and obstetric follow-up in 2 days for a repeat hCG C Have her return to ED the next day for a repeat ultrasound and hCG D Obtain an obstetric consult in the ED E Schedule her an appointment with an obstetrician and follow up ultrasound in 1 week

The correct answer is A. Discharge without any further testing and with reassurance Explanation: When bismuth combines with sulfur in the GI tract, bismuth sulfate is formed which is black. However, unlike with melena, stool guaiac testing will be negative. Given that the patient has a normal exam, a normal hemoglobin, and he gives the history of taking a bismuth-containing medication that accounts for his guaiac-negative black stools, he can be safely discharged home without further testing. The other interventions would be appropriate in a patient with a high clinical suspicion for a GI bleed. 82% of users answered correctly.

A 23-year-old male presents to the ED complaining of black stools. He states he has been having an upset stomach and diarrhea for the past 2 days. He started taking an over-the-counter, bismuth-containing medication, with relief of his symptoms. However, today he noticed the onset of black stools and became alarmed. He has a normal exam except for black stool in his rectal vault that is guaiac negative. Lab results show hemoglobin of 15 gm/dL and BUN:creatinine ratio of <20. Which of the following is MOST appropriate? A Discharge without any further testing and with reassurance B Initiation of oral proton pump inhibitor and close follow-up C Insertion of a nasogastric tube to evaluate for an UGI bleed D Intravenous proton pump inhibitor and admission E Type and screen for possible transfusion of packed red blood cells

The correct answer is B. Elevated prothrombin time suggests a problem with the extrinsic and/or common coagulation pathways. Explanation: Basic laboratory evaluation of the bleeding patient should include a complete blood count, prothrombin time, and activated partial thromboplastin time. Having said this, there are many other laboratory tests that can be performed to evaluate the cause for bleeding. The prothrombin time is a measure of the function of the extrinsic and common coagulation pathways. Although a patient may have a normal platelets, the platelets may not have normal function. Treatment with coumadin causes a delay in the prothrombin time. The D-dimer is elevated in a number of disease states, including in disseminated intravascular coagulation. Finally, protein S is a vitamin D-dependent protein. 73% of users answered correctly.

A 23-year-old man presents to the emergency department with persistent bleeding after cutting himself while shaving. Which of the following is TRUE? A D-dimer is lowered in disseminated intravascular coagulation. B Elevated prothrombin time suggests a problem with the extrinsic and/or common coagulation pathways. C Normal platelet count excludes platelets as a cause for bleeding. D Warfarin causes prolongation of the activated partial thromboplastin time. E Protein C is dependent on vitamin E.

The correct answer is C. Three blood cultures from separate sites, antibiotics, and admit Explanation: Ten to fifteen percent of injection drug users who develop fever have endocarditis. The high rate of disease supports a workup. The absence of a discernable murmur, especially in a noisy ED, does not negate the diagnosis of endocarditis. This patient may have another reason for fever, but given the risks of disease, strong consideration should be made for admission and antibiotic therapy in a febrile and toxic injection drug user. Although it is unfortunate that his sister has systemic lupus erythematosus, this family history is less likely to have anything to do with the patient's clinical presentation. Related Topics: endocarditis 87% of users answered correctly.

A 23-year-old previously healthy male presents to the ED complaining of fever for 4 days. On further questioning, he admits to regularly using intravenous heroin. His temperature is 38.9°C, blood pressure is 110/60, and heart rate is 110. He appears ill. He has a sister with systemic lupus erythematosus. The rest of his exam appears nonfocal. What is the MOST appropriate management? A Erythrocyte sedimentation rate and antinuclear antibody B Reassure patient it is a viral syndrome and discharge home C Three blood cultures from separate sites, antibiotics, and admit D Thyroid function assessment E Urine drug screen

The correct answer is B. Obtain a chest radiograph Explanation: Pregnant women with severe dyspnea should be evaluated for pneumonia, congestive heart failure, and pulmonary embolism. In this situation, a chest x-ray should not be withheld because of fear of radiation exposure to the fetus. The use of D-dimer in pregnant patients remains controversial but is likely to be elevated in the second and third trimester; therefore, it is not a useful test for pulmonary embolism. If a pregnant patient has a deep vein thrombosis on ultrasound and shortness of breath, she could be started on heparin for presumed pulmonary embolism, but, in this scenario, a chest x-ray should be performed for concern of congestive heart failure. While both Lovenox and heparin are safe in pregnancy because they do not cross the placenta, these medications should not be started without imaging or high clinical suspicion for thrombosis given the potential complication of bleeding and the need to guide the duration of therapy. 34% of users answered correctly.

A 24-year-old female G2P1 at 34 weeks of pregnancy presents with progressive dyspnea on exertion. She states she is much more short of breath than with her last pregnancy. She notes a dry cough and bilateral leg swelling but denies fevers and chest pain. Her physical exam is notable for a HR 112, RR 24, and O2 saturation of 94%. Her breath sounds are clear. Which of the following statements is the most appropriate in her management? A Order lower extremity ultrasounds and, if positive for a deep vein thrombosis, start heparin for presumed pulmonary embolism B Obtain a chest radiograph C Order a ventilation perfusion scan D Prescribe Lovenox for a presumptive pulmonary embolism without imaging E Send a D-dimer for the evaluation of pulmonary embolism

The correct answer is C. Vitamin B2 Explanation: Vitamin B2, riboflavin, is found in meats, dairy, and leafy green vegetables. It is not stored in the body and must be replaced daily. It is important in the metabolism of fats, proteins, and carbohydrates. Vigorous exercise will significantly increase the body's need for riboflavin. 32% of users answered correctly.

A 24-year-old female vegetarian (no dairy/ animal by-products) training for a marathon for 1 month presents with persistent cracked lips, a reddened tongue, and eczema of the face and genitalia. What vitamin deficiency is causing her symptoms? A Vitamin A B Vitamin B1 C Vitamin B2 D Vitamin B12 E Vitamin K

The correct answer is B. Azithromycin by mouth in a single dose, needle aspiration of lymph nodes, and referral for human immunodeficiency virus (HIV) and syphilis testing. Explanation: Ulcerative genital lesions are usually due to herpes simplex, syphilis, or chancroid. The epidemiology of sexually transmitted infections is important in the evaluation and thus a travel history should be obtained. Chancroid is rare in the United States and other developed countries. However, in developing countries, it is extremely common. Chancroid, caused by Haemophilus ducreyi, typically presents as a painful papule that forms an ulcer on a friable base with a yellow-gray necrotic exudate. After 1-2 weeks, painful bilateral inguinal lymphadenopathy (buboes) can develop that may require incision and drainage or needle aspiration to prevent fistula formation. First-line treatment is azithromycin 1 gm by mouth in a single dose. Chancroid is also a prominent cofactor for transmission of HIV, and patients diagnosed with chancroid should also be screened for other communicable diseases. Sexual partners should be instructed to receive an examination and treatment. Single-dose azithromycin and ceftriaxone intramuscularly will treat urethritis or cervicitis. Herpes can be differentiated from chancroid by the smaller, depressed ulcerations without a preceding papule, as well as the presence of systemic symptoms. While many STDs are reportable, herpes is not. Penicillin is the treatment for syphilis. The chancre of syphilis can be differentiated from chancroid by its painless character. Doxycycline for 21 days is the treatment for the painless chancre caused by Lymphogranuloma Venereum.

A 24-year-old male presents to your emergency department (ED) complaining of a painful ulcer on his penis for 1 week (Figure 10-1). He denies constitutional symptoms or penile discharge. He reports having unprotected sexual intercourse with a prostitute in Nairobi 2 weeks ago. On exam, you also notice that he has bilateral tender and erythematous inguinal lymphadenopathy. Which of the following is MOST appropriate? A Azithromycin by mouth and ceftriaxone 125 mg intramuscularly along with a prescription to give to his partner. B Azithromycin by mouth in a single dose, needle aspiration of lymph nodes, and referral for human immunodeficiency virus (HIV) and syphilis testing. C Acyclovir 400 mg by mouth three times daily for 7 days and report the disease to the Centers for Disease Control and Prevention (CDC). D Benzathine penicillin G 2.4 million units intramuscularly and follow up with public health for repeat serological testing. E Doxycycline by mouth for 21 days, consideration for incision and drainage of lymph nodes, and referral for HIV and syphilis testing.

The correct answer is A. Ask her about a history of exposures to HIV, mononucleosis, and influenza and treat with salt gargling, oral hydration, and analgesics. Explanation: This patient presents with classic viral pharyngitis and should be queried about risk factors for the three viral causal organism that require diagnostic testing—mononucleosis, influenza, and HIV. However, acute diagnostic testing is usually not warranted. More than 90% of primary infections with HIV are associated with acute pharyngitis developing 2-4 weeks postexposure, but antibody titer may not be positive for up to 4-6 months. If suspicion for HIV is high, an HIV RNA viral load may be drawn. Up to 25% of patients with mononucleosis in the first week of symptoms have a false-negative test. She does not meet three of the four CDC criteria for GABHS, which are tonsillar exudates, tender anterior cervical adenopathy, absence of cough, and history of fever, so testing is not recommended. She has no signs of significant soft-tissue edema threatening airway compromise and so there is no role for dexamethasone. Fungal pharyngitis presents with hyperemia and exudates and usually has buccal mucosal involvement. There is no role for acyclovir in this scenario. 48% of users answered correctly. Bad question, she should get a RADT. Both A and C

A 24-year-old student presents with 5 days of low-grade fever, cough, and worsening sore throat. On physical exam, she has a petechial pattern on her soft palate, grade II tonsils with erythema, and no exudates and nontender cervical adenopathy. What is the most appropriate plan of action? A Ask her about a history of exposures to HIV, mononucleosis, and influenza and treat with salt gargling, oral hydration, and analgesics. B Ask her about recent antibiotics and immune status and treat with fluconazole, oral hydration, and analgesics. C Give her single-dose oral dexamethasone and discharge with salt gargling, oral hydration, and analgesics. D Test her for Group A Beta-Hemolytic Streptococci (GABHS) using the rapid antigen test and, if negative, treat with salt gargling, oral hydration, and analgesics. E Test her for heterophile antibody and treat with acyclovir, oral hydration, and analgesics.

The correct answer is B. Factor V Leiden mutation Explanation: Factor V Leiden mutation is the most prevalent inherited clotting disorder. It occurs in approximately 5% of American whites, with a much lower rate in other races. 78% of users answered correctly.

A 24-year-old woman presents for the second time to your emergency department with the complaint unilateral lower leg swelling and pain. Similar to year ago, the patient is found to have a deep venous thrombosis—this time of the opposite leg. On further questioning, you discover that both of her siblings have also had a lower extremity deep vein thrombosis without a clear cause. What is the most common heritable hypercoagulable disorder? A Antithrombin deficiency B Factor V Leiden mutation C Protein C deficiency D Protein S deficiency E Prothrombin gene mutation

The correct answer is D. Echocardiography Explanation: This patient has mitral stenosis and atrial fibrillation, which is why there is no presystolic accentuation of the rumbling, mid-diastolic murmur. In this case, the duration of atrial fibrillation is uncertain, but the current rapid ventricular rate has caused increased left atrial pressure and pulmonary congestion. A fluid bolus is contraindicated. Aspirin could be given but does not address the patient's heart rate or valvular condition. Controlling the heart rate in atrial fibrillation is very important, but this patient is at risk for a systemic embolus if she medically cardioverts with diltiazem. Hence, an echocardiogram to assess the presence of a left-ventricular thrombus would be highly appropriate. Transesophageal echocardiography can give a more complete analysis of mitral valve function than a transthoracic study. However, transthoracic echocardiography is usually done first, due to easier availability. If echocardiography demonstrates a thrombus, the patient would then be anticoagulated, initially with heparin and later with coumadin. 57% of users answered correctly.

A 24-year-old woman presents to the ED with acute exertional dyspnea for the past week. Vital signs include HR 130-140, BP 132/80, RR 24, T 37, room air SaO2 96%. The patient is alert and cooperative. Physical examination is notable for rales, an irregular heart rate, a loud S1, split S2, followed by a high-pitched opening snap heard best to the right of the apex, and a mid-diastolic rumbling murmur, without presystolic accentuation. Peripheral edema, hepatomegaly, and ascites are absent. ECG shows narrow complex atrial fibrillation with a rapid ventricular rate in the 130s. Which of the following ED actions is MOST appropriate? A Aspirin 325 mg orally B Coumadin 5 mg orally C Crystalloid bolus D Echocardiography E Low-molecular-weight heparin

The correct answer is A. Administer 2 cc of 2% lidocaine to the preauricular space bilaterally, grab the posterior mandibular molars with padded thumbs, and exert steady pressure downward and posteriorly. Explanation: This patient presents with a spontaneous atraumatic anterior dislocation. A radiograph is not required as the diagnosis is clinical unless the reduction is unsuccessful or difficult. This procedure does not require a consult service unless there are other issues or time constraints. The proper method would be anesthesia either locally or with IV narcotics and muscle relaxants, and then grab the posterior mandibular molars with padded thumbs and exert steady pressure downward and posteriorly. 49% of users answered correctly.

A 25-year-old Japanese tourist presents to the ED complaining of inability to close her jaw after yawning widely this morning. She was in her normal state of health prior. No history of trauma. On physical exam, she has a gaping jaw with the mandibular teeth protruding anterior to the alveolar teeth. What is the most appropriate management? A Administer 2 cc of 2% lidocaine to the preauricular space bilaterally, grab the posterior mandibular molars with padded thumbs, and exert steady pressure downward and posteriorly. B Administer IV anesthesia, grab the anterior inferior molar with padded thumbs, and exert anterior rotational torque. C Obtain a panorex radiograph, administer intravenous antibiotics, and call an oral surgery consult. D Obtain a panorex radiograph, administer intravenous narcotics and muscle relaxants, and exert steady pressure on the mentus posteriorly. E Obtain a panorex radiograph, administer intravenous narcotics and muscle relaxants, grab the posterior mandibular molars with padded thumbs, and exert steady pressure downward and posteriorly.

The correct answer is A. Begins as a symmetrically distributed macular rash on the trunk and flexor surfaces and may extend to involve the palms, soles, and oral mucosa. Explanation: The rash of secondary syphilis typically occurs 4-6 weeks after the initial infection. It develops as nonpruritic, rough, red or reddish brown macules on the trunk and flexor areas and often extends to the palms, soles, and oral mucosa. The involvement of the palms and soles is an important diagnostic clue to secondary syphilis. Moist areas, especially the vulva and scrotum, may develop flat, wart-like lesions known as condyloma latum; these lesions are extremely infectious. Secondary syphilis is often associated with systemic symptoms. The rash will resolve without treatment, but the disease may progress to latent disease and possibly tertiary syphilis. Intramuscular benzathine penicillin remains the treatment of choice. 57% of users answered correctly.

A 26-year-old female is complaining of a diffuse rash, as well as fever. Syphilis is on the differential. Which of the following is TRUE of the typical rash of secondary syphilis? A Begins as a symmetrically distributed macular rash on the trunk and flexor surfaces and may extend to involve the palms, soles, and oral mucosa. B Is not associated with systemic symptoms such as fever, fatigue, or myalgia. C May include wart-like lesions known as condyloma latum in moist areas of the body that are not infectious. D Typically develops 6-8 months after the appearance of the primary chancre. E Will not resolve without treatment with penicillin.

The correct answer is B. HIV testing Explanation: Herpes zoster is a reactivation of latent infection with the varicella zoster virus. The papulovesicular rash typically has a unilateral, dermatomal distribution that does not cross the midline, and at most may involve two to three contiguous dermatomal segments. It is predominantly a disease of older patient populations due to waning cell immunity. Zoster in younger patients, or lesions involving multiple noncontiguous dermatomes, should raise suspicion for an immunocompromised state. Testing this patient for HIV is prudent. The diagnosis of zoster is typically made on clinical grounds. Viral culture may be warranted in severe disseminated disease or when the diagnosis is uncertain, but would not apply to this case. In a typical presentation of zoster, a biopsy would not add to the case. A Tzanck smear may reveal giant cells with multiple nuclei but cannot differentiate between varicella-zoster and other herpes viruses and is not useful. 63% of users answered correctly.

A 26-year-old male presents to the ED with 3 days of malaise and the painful rash depicted in Figure 10-4. In addition to treating with antiviral agents, what further testing should be considered in this patient? A Aspiration of a vesicle for viral culture B HIV testing C Punch biopsy for histology and cytology D Skin scraping for KOH prep E Swab of an unroofed vesicle for Tzanck smear

The correct answer is C. False-positive tests can occur with the presence of Trichomonas vaginalis. Explanation: Amniotic fluid has a pH of 7.0-7.4 and will turn nitrazine paper blue. False-positive tests can occur with blood, lubricant, the presence of T. vaginalis, semen, or cervical mucus. Once membranes have broken, digital examination should be avoided because there is an increased risk of infection after even a single digital examination. Thick brownish green fluid is not a normal finding in amniotic fluid and is consistent with meconium. This should alert the physician of possible fetal complications from meconium aspiration. 40% of users answered correctly.

A 26-year-old woman who is 36 weeks pregnant presents after feeling a gush of fluid after her yoga class and is concerned that her water has broken. Which of the following is true regarding confirmation of rupture of membranes? A A digital exam should be performed for confirmation. B Amniotic fluid has a pH of 4.5-5.5 and turns the nitrazine strip yellow. C False-positive tests can occur with the presence of Trichomonas vaginalis. D Lubricant should be used during the sterile speculum exam to decrease patient discomfort. E Thick brownish green fluid is a normal finding with uncomplicated rupture of membranes.

The correct answer is B. Due to her HIV status, she should be admitted to the hospital with intravenous antibiotics for presumed PID. Explanation: Patients with PID should be admitted if they are pregnant, fail to respond to outpatient therapy, are unable to tolerate or comply with outpatient therapy, have severe toxicity, tubo-ovarian abscess, or HIV infection irrespective of their CD4 count. Parenteral treatment includes cefoxitin plus doxycycline or clindamycin plus gentamicin. Due to emerging resistant, fluoroquinolones are not longer recommended to treat gonorrhea. Intrauterine devices should be removed after antibiotics are started. 53% of users answered correctly.

A 27-year-old female presents with lower abdominal pain and fever. She is HIV positive with a CD4 count of 700. She has an intrauterine device in place. Her hCG is negative. She is afebrile with a blood pressure of 125/60. Exam demonstrates cervical motion tenderness and adnexal tenderness. Her ultrasound is unremarkable. Which of the following statements is correct regarding her management? A As she is not an adolescent and is not planning on becoming pregnant, discharge home on oral antibiotics for presumed PID. B Due to her HIV status, she should be admitted to the hospital with intravenous antibiotics for presumed PID. C Her intrauterine device does not need to be removed as long as antibiotics are started. D Levaquin with doxycycline is the treatment of choice. E Since her CD4 count is above 200, she can be discharged home on oral antibiotics for presumed PID.

The correct answer is B. 50 mL/hour Explanation: It is important to know how to treat seizures caused by hyponatremia as too rapid of a correction may lead to central pontine myelinolysis. Administration of 3% saline should be considered when hyponatremia is severe (<120 mEq/L), develops rapidly (>0.5 mEq/L, decrease in serum Na per hour), or when patients are in extremis (i.e., seizure or coma). The dosage is 25-100 mL/hour and requires close observation of serum sodium levels. To reduce the risk of complications from rapid correction of sodium deficits, sodium levels should not rise more than 0.5-1.0 mEq/L per hour. If seizures are present, the rise in sodium can be increased to 1.0-2.0 mEq/L per hour. In patients with hyponatremia, it is important to assess volume status. This patient presents with a euvolemic hyponatremia, most likely caused by her cystic fibrosis. In patients who present with hyponatremia and volume overload (pulmonary edema, ascites, pitting edema), syndrome of inappropriate antidiuretic hormone should be considered. Related Topics:hyponatremia 52% of users answered correctly.

A 27-year-old patient with a history of cystic fibrosis presents to the ED complaining of 2 days of gradual onset throbbing headache. She has had no fevers, chills, upper respiratory infection symptoms, and has never gotten headaches before. She is very healthy and has been going to the gym a lot in the last 2 weeks. Her husband states that she has been "grumpy" the last day or two and he has noticed she occasionally has resting "muscle twitches." She does not drink alcohol, smoke, or use illegal drugs. Her last hospitalization was more than 2 years ago. Her vital signs are within normal range and she is afebrile. The patient proceeds to have a grand mal seizure in the ED and while the nurse is giving IV benzodiazepines, the laboratory calls you for a serum sodium level of 109. Which of the following is the most appropriate rate for a 3% sodium chloride infusion? A 5 mL/hour B 50 mL/hour C 150 mL/hour D 250 mL/hour E 400 mL/hour

The correct answer is A. Because of the potential vasoconstriction of the uteroplacental circulation, epinephrine should only be used in the most critically ill pregnant patients. Explanation: Epinephrine may cause vasoconstriction of the uteroplacental circulation; therefore, it should only be used in severely ill asthmatic patients. Peak expiratory flow rates are not altered in pregnancy and should be used to guide therapy and need for admission. Terbutaline and steroids are considered safe in pregnancy and may be used if clinically indicated. In general, during pregnancy, asthma follows a rule of one-thirds: about one-third of the patients have improvement, one-third of the patients worsen, and one-third of the patients have their asthma symptoms remain the same. 43% of users answered correctly.

A 28-year-old female G2P1 at 24 weeks with history of asthma presents to the ED with 3 days of cough and wheezing with shortness of breath. She is speaking full sentences and her oxygen saturation is 92% on room air. Which of the following statements is true? A Because of the potential vasoconstriction of the uteroplacental circulation, epinephrine should only be used in the most critically ill pregnant patients. B Most pregnant patients will have worsening of their underlying asthma during the last trimester of pregnancy. C Peak expiratory flow rates are decreased in pregnancy and, therefore, cannot be reliably used in pregnant patients as a marker of disease severity. D Steroids should be reserved for the most critically ill pregnant asthma patients. E Terbutaline is contraindicated during asthma flares in pregnant patients.

The correct answer is D. Surgery consultation Explanation: External hemorrhoidal hematoma formation is usually self-limiting with resolution in 1 week. Therapy for thrombosed external hemorrhoids depends on the severity of symptoms. If the thrombosis has been present for more than 48 hours, the swelling has started to shrink, is not tense, and the pain is tolerable, the patient may be treated with sitz baths and bulk laxatives. Suppositories, which are placed proximal to the anorectal ring, are of no help. If, on the other hand, the thrombosis is acute, has lasted less than 48 hours, and is extremely painful, significant relief can be provided by clot excision. Excision should not be performed in the ED on immunocompromised patients, children, pregnant women, patients with portal hypertension, and those who are anticoagulated or have a coagulopathy. 38% of users answered correctly.

A 28-year-old pregnant female (32 weeks of gestation) presents to the ED complaining of painful bowel movements. The pain has been present for 24 hours, and is extremely painful. She is unable to sit during the exam and must lay on her side. On rectal exam, two thrombosed external hemorrhoids are visualized. What is the MOST appropriate next step? A Clot excision B Laxatives C Sitz baths D Surgery consultation E Topical hydrocortisone

The correct answer is A. Acute chest syndrome Explanation: Acute chest syndrome is the leading cause of death in patients with sickle cell disease. It is also the second most common cause for admission, after vaso-occlusive pain crisis, in patients with sickle cell disease. 63% of users answered correctly.

A 29-year-old African American man with sickle cell disease presents to the emergency department complaining of bilateral elbow and lower leg pain. These symptoms are typical of his pain crises. Which of the following complications is the leading cause of death in patients with sickle cell disease? A Acute chest syndrome B Aplastic crisis C Bacteremia D Hypersplenism E Stroke

The correct answer is D. Potassium Explanation: This patient is likely suffering from a hyperosmolar hyperglycemic state (HHS). His glucose is greater than 600 mg/dL. While all of the labs are helpful, the most important electrolyte abnormality is hypokalemia and should be anticipated. As treatment of HHS with fluids and insulin often alter the serum potassium levels, the importance of monitoring of serum potassium levels and repletion cannot be overemphasized. Serum sodium is often abnormal in patients with HHS. They often have low sodium levels. In severe hyperglycemia, the sodium must be corrected for with the formula: corrected [Na] = measured [Na] + [1.6 × (glucose − 100)]/100. 75% of users answered correctly.

A 29-year-old male is brought in by EMS after being found by a friend stuporous in his apartment. His friend tells you that the patient is a migrant construction worker from Mexico and had not shown up for work for the past 5 days. As far as the friend knows, the patient has no medical problems and does not use drugs or alcohol. He is unsure about the patient's family medical history. EMS reports that the patient's glucose was "high" (>600). His vital signs reveal hypotension and tachycardia. On exam, the patient is stuporous, has dry mucous membranes, clear lung fields, and poor skin turgor. His abdomen is soft and nontender. The patient receives two large bore IVs and 2 L of normal saline (NS). His vital signs remain normal and the patient is able to answer simple questions. On neurological exam, the patient has profound weakness in his upper and lower extremities. Which of the following electrolyte abnormalities pose the most immediate risk for bad outcome in this patient? A Chloride B Glucose C Magnesium D Potassium E Sodium

The correct answer is E. Pneumonia Explanation: The description is classic for chickenpox caused by the varicella zoster virus. Pregnant women are typically screened for antibodies due to a higher rate of complications. Chickenpox in the first or second trimester may lead to congenital embryopathy. Disease in the third trimester does not typically affect the fetus, but has a much higher rate of serious pulmonary complications in the mother. Smallpox has been eradicated through global vaccination, but has resurfaced as a potential biochemical weapon. Clinically, the lesions of smallpox tend to be uniform (either all vesicles or all pustules), as opposed to the variable stages of lesions seen in chickenpox. Additionally, the lesions in smallpox are denser peripherally and include the palms and soles. The rash of meningococcemia is initially petechial. As the disease progresses, it may become purpuric with central necrosis, and patients demonstrate severe systemic toxicity. 40% of users answered correctly.

A 30-year-old G1P0 female at 34 weeks of gestation presents complaining of a rash for 2 days. There are papules, vesicles, and crusted lesions on her torso and face. When queried, she denies all childhood immunizations. What concern do you have for this patient? A Bioterrorism B Congenital deformity C Meningococcemia D No concerns E Pneumonia

The correct answer is A. Incision of the abscess with placement of a Word catheter and treatment with ceftriaxone, doxycycline, and metronidazole. Explanation: Appropriate treatment in the ED for a Bartholin gland abscess is incision with placement of a Word catheter. Use of the Word catheter promotes fistula formation and decreases the rate of reformation. Antibiotics can be added as clinically necessary. Cephalexin plus metronidazole has been standard therapy if a sexually transmitted disease is not suspected, as the infection is typically polymicrobial vaginal flora. There have been reports of MRSA causing vaginal infections and thus there is a trend toward obtaining cultures. Although less common, both N. gonorrhoeae and C. trachomatis can each cause a Bartholin gland abscess. If clinical suspicion is high for sexually transmitted disease, treatment should be the typical regimen for cervicitis plus metronidazole. A Bartholin gland abscess in perimenopausal women may require biopsy due to a higher relationship with carcinoma. Simple incision and drainage with iodoform gauze packing has a high rate of discomfort and recurrent abscess formation. Marsupialization involves an elliptical incision and suturing usually performed by a gynecologist. 65% of users answered correctly.

A 30-year-old female comes to the ED complaining of pelvic pain and discharge. She acknowledges multiple sexual partners in the previous month. On exam, there is a tender and fluctuant 2-cm mass located on the medial labia minora with mild overlying erythema at the 4 o'clock position. There is also a thin, yellowish discharge. What is the MOST appropriate management? A Incision of the abscess with placement of a Word catheter and treatment with ceftriaxone, doxycycline, and metronidazole. B Incision of the abscess with placement of a Word catheter and treatment with cephalexin and metronidazole. C Incision and drainage without packing and Sitz baths at home. D Incision and drainage with iodoform gauze and treatment with cephalexin and metronidazole. E Marsupialization of the abscess and referral to gynecology for biopsy.

The correct answer is B. Obtain an ultrasound to evaluate for retained products of conception. Explanation: Complications of dilatation and curettage include uterine rupture, retained products of conception, and postabortal endometritis. When postabortal endometritis is suspected with fever and tender uterus, an ultrasound should be performed to look for retained products of conceptions. Rhogam is indicated if the patient is Rh negative and should be given at the time of the dilatation and curettage. Most uterine perforations occur at the time of surgery and are asymptomatic. Delayed presentations of uterine perforation include abdominal pain and shock that would warrant a STAT gynecologic consult. 63% of users answered correctly.

A 30-year-old female presents 2 days postdilatation and curettage for a missed abortion at 12 weeks. Her physical exam is notable for fever 102°F, heart rate of 110, blood pressure of 120/80. Her pelvic exam is notable for a closed os with firm, tender uterus. What is the next appropriate step in her management? A Obtain an upright chest x-ray to evaluate for free air. B Obtain an ultrasound to evaluate for retained products of conception. C Rhogam 300 MIU IM. D Start oral trimethoprim/sulfamethoxazole for likely staph infection. E STAT gynecology consult for concern of uterine perforation.

The correct answer is E. Supportive care and observation with benzodiazepines if need for sedation Explanation: Psilocybe and Gymnopilus mushrooms can cause euphoria, hallucinations, tachycardia, and other neurological symptoms but are not associated with GI toxicity. Treatment is supportive with benzodiazepines as needed. Inocybe and Clitocybe mushrooms can cause the SLUDGE syndrome due to their cholinergic properties and toxicity may be treated with atropine. Many different types of mushrooms are associated with GI symptoms and can be treated with supportive care including antiemetics. Decontamination with activated charcoal is also indicated in most cases of mushroom poisoning presenting with GI symptoms as empiric treatment for Amanita phalloides and Amanita bisporigera poisoning. These two mushrooms are associated with delayed liver toxicity and liver failure. 65% of users answered correctly.

A 30-year-old man presents with hallucinations, tachycardia, and ataxia after eating some mushrooms his roommate brought home from a camping trip. What is the most appropriate therapy? A Activated charcoal and monitoring of liver enzymes B Atropine and intravenous fluids C Immediate evaluation for liver transplant D Intravenous hydration and antiemetics E Supportive care and observation with benzodiazepines if need for sedation

The correct answer is D. Hematocrit is decreased in pregnancy. Explanation: In pregnancy, there is an increase in blood volume and a smaller increase in red blood cell mass leading to a dilutional anemia and a drop in the hematocrit. Cardiac output is increased by 30-40% in pregnancy. Many respiratory changes occur during pregnancy. The minute ventilation, tidal volume, and respiratory rate increase during pregnancy resulting in a decreased functional residual capacity and a slight respiratory alkalosis. Blood pressure decreases to a nadir in the second trimester. 59% of users answered correctly.

A 30-year-old woman G1P0 at 36 weeks of pregnancy presents to the ED following a motor vehicle collision. Which of the following statements correctly describes a physiologic change in pregnancy that would impact the resuscitation of this patient? A A baseline tachycardia is expected in the last trimester pregnancy. B Blood pressure decreases to a nadir in the third trimester. C Functional residual capacity is increased in pregnancy. D Hematocrit is decreased in pregnancy. E Pregnant women will not efficiently compensate for blood loss because of a decrease in cardiac output.

The correct answer is A. Acute necrotizing ulcerative gingivitis Explanation: This patient has acute necrotizing ulcerative gingivitis also known as Vincent's disease or trench mouth considered an opportunistic infection with lowered host immunity—anaerobic bacteria such a Treponema invade otherwise healthy tissue. HIV is the most important predisposing factor. Treatment consists mainly of bacterial control with chlorhexidine washes, professional debridement, analgesics, and oral antibiotics that cover anaerobes nutritional support and follow-up with an oral surgeon within 24 hours. However, it can be difficult to differentiate from herpes gingivostomatitis. Herpes gingivostomatitis usually has more systemic symptoms and a lack of interdental papilla involvement. Hand, foot, and mouth disease involves other surfaces of the mouth as well, not just the gingival. Phenytoin causes gingival hyperplasia and bleeding. 79% of users answered correctly.

A 32-year-old HIV-positive man presents with painful teeth, bleeding gums, and foul metallic taste in his mouth (Figure 6-4). On exam he has a low-grade fever, fetid breath, and the following dental exam. What is this patient's diagnosis? A Acute necrotizing ulcerative gingivitis B Hand, foot, and mouth disease C Herpes gingivostomatitis D Phenytoin-induced gum disease E Vitamin deficiency

The correct answer is D. You answered E. Explanation: Stress incontinence occurs when intra-abdominal pressure exceeds intraurethral pressure in the absence of bladder contractions. This type of incontinence is associated with pregnancy, multiparity, menopause, and chronic cough. The nonsurgical treatment includes Kegel exercises, estrogen, and alpha-adrenergic medications. Overflow incontinence is the continuous leakage from impaired detrusor muscle contractility or bladder outlet obstruction. Urge incontinence is related to uninhibited bladder contractions often the result of interstitial cystitis. Although all urinary tract infections, including asymptomatic bacteruria, should be treated in pregnant women, a urinalysis and culture should be performed before antibiotics are started. 26% of users answered correctly.

A 32-year-old female 24 weeks pregnant presents very concerned because she is occasionally incontinent of small amounts of urine. She notices it is worse when she coughs. Which of the following statements is true regarding her diagnosis? A She has overflow incontinence. B She has urge incontinence. C She should be treated for a presumptive urinary tract infection. D This likely occurs because intra-abdominal pressure is lower (greater) than intraurethral pressure. E Treatment includes Kegel exercises.

The correct answer is D. She can continue to breast-feed without interruption. Explanation: Mastitis is treated with frequent breast emptying, analgesia, and anti-staphylococcal penicillin or cephalosporin plus Bactrim if methicillin-resistant staphylococcus aureus (MRSA) is a concern. There is no need to interrupt breastfeeding with mastitis. Fluoroquinolones are not used for the treatment of mastitis. If an infection fails to respond to antibiotics, an abscess should be suspected and an ultrasound and surgical consult should be obtained for incision and drainage or needle aspiration. 80% of users answered correctly.

A 32-year-old female presents 2 weeks postpartum with a painful right breast and chills. She is breastfeeding without difficulty. Her right breast is firm, red, and diffusely tender with no obvious fluctuant mass. Which of the following statements is correct regarding her diagnosis and management? A If the infection fails to respond to the initial antibiotics, intravenous antibiotics should be initiated. B Fluoroquinolones are the treatment of choice. C Most cases are caused by Gram-negative bacteria. D She can continue to breast-feed without interruption. E She should not breast-feed but rather "pump and dump" until the erythema has resolved.

The correct answer is E. STAT OB/GYN consult Explanation: The ultrasound shows an empty uterus with free fluid in the cul-de-sac concerning for a ruptured ectopic pregnancy. Since the patient is unstable, she needs a STAT obstetric consult for possible laparotomy before a formal ultrasound is obtained. While a serum hCG should be obtained, it should not delay an obstetric consult. An abdominal CT scan is not useful in the evaluation of an ectopic pregnancy. Serum progesterone may help differentiate an early normal from a pathologic pregnancy but is not readily available and not indicated in the unstable patient. 80% of users answered correctly.

A 32-year-old female presents to the ED with severe lower abdominal pain. She cannot remember when her last menstrual period was because she is irregular. Physical exam reveals a pale female with a blood pressure of 80/50 and heart rate of 120. Her physical exam is remarkable for left lower quadrant tenderness. Her urine hCG is positive (Figure 8-1). Bedside ultrasound is performed. Which of the following is the next best step in management? A Obtain a formal pelvic ultrasound B Obtain a noncontrast abdominal CT scan C Obtain a serum hCG D Obtain a serum progesterone level E STAT OB/GYN consult

The correct answer is D. No study is needed Explanation: The patient met the criteria for the application of the PERC rule. If all criteria are met, the patient has a probability of PE of less than 2%, which is under the commonly accepted threshold for diagnostic testing (see Table 60-7). 51% of users answered correctly. The PERC criteria are used to help the clinician do a bedside assessment to determine if a patient is at "very low risk" for pulmonary embolism and does not warrant additional diagnostic evaluation, including a D-dimer. The PERC criteria includes: - Age < 50 yrs - Pulse < 100 bpm - SaO2 > 94% - No unilateral swelling - No hemoptysis - No recent trauma or surgery - No prior PE or DVT - No hormone use

A 32-year-old male presents to the ED with chest pain and mild shortness of breath. He has no prior history of VTE, no recent surgery or trauma, no hemoptysis, estrogen use, or leg swelling. His initial vitals are BP 130/80, HR 92, RR 20, T 37.0, and saturation 98%. What is the most appropriate diagnostic test to rule out a PE in this patient? A Angiogram of chest B CT scan of chest C D-dimer D No study is needed E V/Q scan

The correct answer is C. Highly active antiretroviral therapy Explanation: Cryptosporidium rarely causes serious disease in normal hosts, but may cause protracted disease in AIDS and often warrants hospitalization due to dehydration and malabsorption. Nitazoxanide and azithromycin are effective against cryptosporidium, but neither affects a cure with a CD4 count below 100. The most effective therapy is highly active antiretroviral therapy, and symptoms generally subside when the CD4 count is above 100. Ciprofloxacin is effective therapy for most typical bacterial pathogens. Metronidazole is active against Giardia, Entamoeba, and Clostridium difficile.

A 32-year-old male with AIDS presents with diarrhea for the past 4 days. He denies travel or recent antibiotics. He is afebrile and his blood pressure is 100/50 and heart rate is 120. His is negative for fecal blood. His last known CD4 count was 26. If this patient has the most common cause of diarrhea in AIDS patients, what treatment is likely to be most beneficial? A Azithromycin B Ciprofloxacin C Highly active antiretroviral therapy D Metronidazole E Nitazoxanide

The correct answer is E. Type and screen for blood, emergent bronchoscopy, consult with a thoracic surgeon Explanation: This patient has mild hemoptysis but is at risk for severe hemoptysis due to mitral stenosis-induced pulmonary hypertension. Arrangements should thus be made for possible blood transfusion, emergent bronchoscopy, and consult with a thoracic surgeon. Ideally, patients with symptomatic mitral stenosis should receive a mechanical intervention such a balloon valvotomy, valve repair, or valve replacement before severe pulmonary hypertension develops. Coumadin would be contraindicated in the face of hemoptysis, and beta-blockers are contraindicated in the face of pulmonary hypertension. Judicious use of furosemide might help decrease pulmonary congestion but does not address the patient's hemoptysis and need for mechanical intervention. 51% of users answered correctly.

A 32-year-old woman with known mitral stenosis presents acutely short of breath, with an intermittent cough productive of small amounts of frankly bloody sputum. The ECG shows sinus rhythm with narrow complexes at a rate of 82 without ST or T-wave changes. The patient is placed on a cardiac monitor, and a non-rebreather face mask, and is currently stable, with an SaO2 of 100%. An IV is started, and multiple tubes of blood are drawn. Cardiology is consulted, and echocardiography is arranged. What is the MOST definitive therapy for this patient? A Beta-blocker therapy to control heart rate below 60 B Coumadin therapy to prevent systemic emboli C Furosemide 40 mg intravenously over 2 minutes D Type and screen for blood E Type and screen for blood, emergent bronchoscopy, consult with a thoracic surgeon

The correct answer is B. Cleanse the wound with soap and water, administer as much HRIG as possible directly into the wound with remaining in the right deltoid, and administer rabies vaccine into the left deltoid. Explanation: It is critical to know the indications for PEP for rabies, as the goal is disease prevention (Table 152-5). Once a patient shows signs of rabies, it is almost always uniformly fatal. PEP in nonimmunized individuals largely depends on the animal and route of exposure. Raccoons, skunks, foxes, bats, and many other large terrestrial carnivores are regarded as rabid unless direct animal testing proves negative. Bites from healthy dogs, cats, and ferrets do not need prophylaxis if the animal can be observed for 10 days and it does not exhibit signs of rabies. Treatment begins with immediate, thorough cleansing with soap and water, as the virus is highly lipophilic. HRIG should be injected directly into the wound without compromising tissue integrity, and any remaining is injected into the ipsilateral deltoid. The rabies vaccine should always be injected into the deltoid on the opposite side of the body to avoid neutralization by HRIG. Failure rates have been associated with gluteal injection and should be avoided. Current guidelines suggest giving 4 or 5 doses of HRIG depending on the patient's immune status. Related Topics: rabies rabies prophylaxis 61% of users answered correctly.

A 33-year-old intoxicated male presents to the ED 2 hours after being bit by a raccoon he was trying to feed while hiking. He states, "the raccoon did not appear rabid." It retreated back into the woods and is unavailable. He has a small puncture wound proximal to his right thumb. What is the proper treatment? A Cleanse the wound with soap and water and administer both rabies vaccine and human rabies immunoglobulin (HRIG) directly into the wound. B Cleanse the wound with soap and water, administer as much HRIG as possible directly into the wound with remaining in the right deltoid, and administer rabies vaccine into the left deltoid. C Irrigate the wound with high-pressure normal saline, administer HRIG directly into the wound with remaining into the right gluteus, and administer rabies vaccine into the left gluteus. D Scrub the wound with povidone-iodine, administer HRIG into the left deltoid, and follow up with public health for vaccination. E Scrub the wound with povidone-iodine and discharge home with amoxicillin/clavulanate.

The correct answer is B. Elevate the presenting fetal part and remain in place as the patient is prepared for surgery. Explanation: In the event of a prolapsed cord, the examiner should not remove his/her hand and should elevate the fetal presenting part to decrease compression of the cord. The examiner's hand should remain in place as the patient is prepared for surgery. A prolapsed cord should never be reduced. Placing the patient on her left side and applying oxygen may increase maternal blood flow and help during decelerations but is not as essential as relieving pressure on the cord. 69% of users answered correctly.

A 34-year-old female at 41 weeks presents in active labor. Bimanual exam reveals a palpable, pulsating cord. The next step in management is: A Carefully reduce the prolapsed cord. B Elevate the presenting fetal part and remain in place as the patient is prepared for surgery. C Give terbutaline to halt uterine contraction and increase blood flow to the fetus. D Place the patient on her left lateral side and apply oxygen. E Start to prep the patient's abdomen for an ED C-section.

The correct answer is A. Electrolytes Explanation: Although alcoholic ketoacidosis (AKA) is most common in chronic alcoholics, it also occurs in first time binge drinkers. Patients usually present with nonspecific complaints of abdominal pain, nausea, and vomiting. The anion gap in AKA is positive and ranges between 16 and 33 and is caused by ketonemia, primarily from β-hydroxybutyrate. The blood glucose ranges from low to mildly elevated. Urine ketones may be often low or negative initially as the nitroprusside reagent used to measure urine and serum ketones only measure acetoacetate. Acetone and β-hydroxybutyrate are not reactive and are more common early in the process of AKA. Gastritis can accompany AKA; however, improvement of symptoms with a "GI cocktail" does not rule out AKA, and without a high suspicion, AKA might be missed. Pancreatitis may also accompany AKA, but this patient does not have a history and physical consistent with pancreatitis. The patient's exam and history are also not consistent with gallbladder disease, so an ultrasound would not aid in making the diagnosis of AKA. 42% of users answered correctly.

A 34-year-old male with no past medical history presents to the ED complaining of 1 day of nausea, vomiting, and epigastric pain after drinking "a lot" over the weekend. He denies hematemesis, bright red blood per rectum or melena. He states that he has never been much of a drinker but drank excessively this weekend when some college friends were in town. He has vomited more than 15 times and has not been eating much because of the nausea and vomiting. His finger stick glucose is 97 and his vital signs are stable. His exam is significant for dry mucous membranes. He is anicteric. His abdomen is soft with mild discomfort in the epigastric region with no abdominal distention, masses, rebound, or guarding. Which of the following is MOST important in determining the patient's diagnosis? A Electrolytes B Improvement with GI cocktail. C Lipase D Ultrasound of gallbladder E Urine ketones

The correct answer is D. Obtain a formal ultrasound Explanation: Since this patient underwent in vitro fertilization, a formal ultrasound should be obtained to evaluate for a heterotopic pregnancy. In this population of patients, heterotopic pregnancy should be considered in symptomatic patients even when a bedside ultrasound demonstrates an intrauterine pregnancy. Rhogam is not indicated since the patient is Rh positive. Laparoscopy is used for patients with suspected ectopic pregnancy and a nondiagnostic ultrasound. 53% of users answered correctly.

A 35-year-old female G1P0 presents with right lower abdominal pain and vaginal bleeding described as spotting for the last 3 hours. Her last menstrual period was 5 weeks ago and she had a positive pregnancy test at home 2 days ago. She and her husband underwent in vitro fertilization for this pregnancy. hCG is 6000 mIU/mL and she is Rh positive. ED bedside ultrasound confirms an intrauterine pregnancy. The next most appropriate step in management is: A Administer Rhogam 50 MIU IM B Congratulate her on her pregnancy and provide reassurance C Discharge home with the diagnosis of threatened abortion and obstetric follow-up in 2 days D Obtain a formal ultrasound E Obtain urgent obstetric consult for a laparoscopy

The correct answer is E. Streptococcus pneumoniae Explanation: In order to provide appropriate care to patients with HIV, it is critical to have an understanding of diseases relative to the absolute CD4 + T cell count. Patients with CD4 counts above 500 typically will develop febrile illnesses similar to the general population. A patient with a CD4 count between 200 and 500, who presents with fever, is most likely to have a bacterial mediated pulmonary infection, and S. pneumoniae is the most common cause. However, tuberculosis is also dramatically increased at this CD4 level, often without the classic radiographic findings of upper lobe involvement or cavitary lesions. It may be prudent to place this patient in respiratory isolation until ruled out by acid-fast bacilli stain and sputum culture. Pneumocystis jirovecii pneumonia is the most common opportunistic pathogen in AIDS patients. It becomes the primary consideration with fever or shortness of breath and CD4 count below 200. Although less likely, cryptococcal pneumonia is also a consideration, and may be included in the workup. Disseminated M. avium complex typically occurs when the CD4 count is below 100. Symptoms are typically fever and night sweats and may include malaise, diarrhea, and weight loss. 51% of users answered correctly.

A 35-year-old male with AIDS presents complaining of fever and a productive cough for 1 day. His last known CD4 count 1 month ago was 350. The following chest radiograph was obtained (Figure 10-5). What is the MOST likely diagnosis? A Cryptococcus neoformans pneumonia B Disseminated Mycobacterium avium complex C Pneumocystis carinii pneumonia D Pulmonary tuberculosis E Streptococcus pneumoniae

The correct answer is A. Aeromonas spp. Explanation: The most common cause of infection in seawater-associated wounds is bacteria found as part of normal skin flora including staph and strep species. Vibrio species are the most commonly implicated marine-specific organisms. Mycobacterium marinum is an atypical mycobacterium that causes cutaneous granulomatous disease associated with both fresh- and salt-water exposure. Aeromonas organisms are seen in fresh water-associated infections. 36% of users answered correctly

A 35-year-old woman presents with a red, swollen, hot foot that is draining pus out of a wound she sustained while snorkeling in the ocean last week. Which of the following organisms is the LEAST likely to be implicated? A Aeromonas spp. B Mycobacterium marinum C Beta-hemolytic streptococci D Staphylococcus aureus E Vibrio spp.

The correct answer is C. Perform an immediate fiberoptic endotracheal intubation Explanation: The primary goal in the management of acute laryngeal fracture is maintenance of the airway. The best approach is endotracheal intubation over a flexible bronchoscope. If the laryngeal lumen cannot be visualized because of edema or bleeding, then emergent tracheotomy at a level lower than usual should be perform. Cricothyroidotomy is contraindicated. Dexamethasone and racemic epinephrine are helpful in reducing edema after an airway has been placed. 26% of users answered correctly.

A 36-year-old man brought in by ambulance after his falling off his horse when he was struck in the neck with a tree limb. He comes in complaining of neck pain and dysphagia. On physical exam, he is speaking with hoarse voice, has edema and crepitus over his anterior midline neck. What is the best course of action? A Administer intravenous dexamethasone and racemic epinephrine nebulizer and obtain a CT of the neck B Perform an immediate cricothyroidotomy then consult ENT for a definitive airway placement C Perform an immediate fiberoptic endotracheal intubation D Perform an immediate fiberoptic endotracheal intubation, if the laryngeal lumen cannot be visualized then perform a cricothyroidotomy E Placed a cervical collar and obtain a CT scan of the neck

The correct answer is D. Hypovolemia and dehydration account for many of the nonspecific symptoms. Explanation: Many of the vague symptoms attributable to hypercalcemia of malignancy are caused by hypovolemia and dehydration, and as such, saline infusion is the mainstay of treatment. How patients present has more to do with the rate of rise of the serum calcium level than the absolute serum calcium concentration. Hypercalcemia is not an infrequent complication, occurring in upward of 30% of patients with advanced cancer. Steroids can be used to treat hypercalcemia, particularly when caused by steroid-sensitive tumors, such as lymphomas and multiple myeloma. Furosemide is no longer routinely recommended in the treatment of cancer-related hypercalcemia, as it has little additive effect to the use of saline alone in patients with normal cardiac and renal function. 67% of users answered correctly.

A 36-year-old woman with metastatic breast cancer presents to the emergency department complaining of confusion, abdominal pain, nausea, and vomiting. Your evaluation reveals a serum calcium level of 17 mg/dL as the cause for her symptoms. Which of the following is TRUE regarding hypercalcemia of malignancy? A Clinical presentation is more attributable to the calcium level than to the rate of rise of the calcium level. B Furosemide is the first-line treatment of hypercalcemia of malignancy. C Hypercalcemia is a rare complication of cancer. D Hypovolemia and dehydration account for many of the nonspecific symptoms. E Steroids are contraindicated in the treatment of cancer-related hypercalcemia.

The correct answer is A. Grave's disease Explanation: The most common cause of hyperthyroidism is Grave's disease. In general, hypothyroidism is 10 times more common in women than in men. Grave disease is most common in the third and fourth decades of life. It is caused by an autoimmune thyroid-stimulating antibody. Toxic multinodular and toxic nodular goiters are the next most frequent etiologies; however, these usually present in older patients, often with a history of an uncomplicated goiter. Other less common causes are thyroiditis, pituitary adenomas, metastatic thyroid cancer, and dermoid tumors. Lithium, iodine, and thyroid medications can also cause hyperthyroidism. 66% of users answered correctly.

A 37-year-old female presents to the ED complaining of palpitations and dyspnea. Her vital signs reveal tachycardia, hypertension, and a fever of 104.1°F. She is breathing rapidly and her O2 saturation is 99% on room air. She is thin and anxious. Her ECG shows atrial fibrillation. She denies drug use and has not had any recent illnesses. Her neck is shown below (Figure 4-1). What is the most likely cause of the symptoms? A Grave's disease B Lithium therapy C Pituitary tumors D Thyroiditis E Toxic nodular goiters

The correct answer is D. PTU, iodine, propranolol, dexamethasone Explanation: The treatment of thyroid storm is complicated because the physician must take into account the neogenesis of thyroid hormone. Stopping the thyroid gland from synthesizing new thyroid hormone needs to occur prior to starting iodine therapy as this will prevent the stimulation of new thyroid hormone. After supportive care, thionamides (PTU or methimazole) should be given to decrease the new hormone production. PTU is preferred over methimazole because it also can inhibit the conversion of T4 to T3. At least 1 hour later, iodine therapy may be started. This will block the release of thyroid hormone. If there is concern for either amiodarone-induced or iodine overload-induced hyperthyroidism, lithium should be used instead of iodine. Propranolol is given to help block the direct effects of the thyroid hormone. It is contraindicated in reactive airway disease, CHF, pregnancy, and diabetes. Glucocorticoid therapy, either dexamethasone or hydrocortisone, block peripheral conversion of T4 to T3 and improve survival rates from thyroid storm. 30% of users answered correctly.

A 37-year-old female presents to the ED complaining of palpitations and dyspnea. Her vital signs reveal tachycardia, hypertension, and a fever of 104.1°F. She is breathing rapidly and her O2 saturation is 99% on room air. She is thin and anxious. Her ECG shows atrial fibrillation. She denies drug use and has not had any recent illnesses. Her neck is shown below (Figure 4-1). After initiating supportive care, which of the following describes the appropriate treatment priorities for this patient? A Dexamethasone, PTU, iodine, propranolol B Iodine, propranolol, dexamethasone, PTU C Propranolol, dexamethasone, PTU, iodine D PTU, iodine, propranolol, dexamethasone E Propranolol, PTU, iodine, dexamethasone

The correct answer is E. IV saline Explanation: In patients with diabetes mellitus, it is very important to prioritize therapeutic interventions. The order of therapeutic priorities is volume resuscitation first and foremost. Patients often have a fluid deficit of 5-10 L. Potassium deficits should be addressed next. Diabetic ketoacidosis (DKA) patients often have profound total-body potassium deficits. For an initial potassium level between 3.3 and 5.3 mEq/L, with established urine output, potassium should be replaced at a rate of 10 mEq KCL per hour for 4 hours. Insulin may be given, but only after volume resuscitation and potassium deficits have been addressed. It may be administered at a bolus of 0.1 units/kg and followed by a drip of 0.1 units/kg per hour. In general, IV phosphate therapy should only be initiated if the serum level is <1.0 mg/dL. Routine use of IV bicarbonate is not recommended in the treatment of DKA. 79% of users answered correctly.

A 37-year-old male presents to the ED with altered mental status. He was found unconscious in the bathroom at work. On exam, he is arousable to painful stimulus, muttering incoherently. His airway is intact and he has bilateral breath sounds. His initial vital signs are blood pressure (BP) 95/47, P 110, respiratory rate (RR) 14, O2% 97% on room air, T 99.4. He has dry mucus membranes. Fingerstick glucose is 396. Lab work reveals a normal CBC, 3+ acetone, Na 121, Cl- 97, HCO3 9, K 3.0, Mg 2.9, Phos 1.5, AG 29. Which of the following is the first priority in caring for this patient? A IV bicarbonate B IV lactated ringers C IV phosphate D IV potassium E IV saline

The correct answer is B. Compartment syndrome Explanation: Patients struck by lightning may sustain cardiac and/or respiratory arrest but have a good prognosis with resuscitation. Prolonged CPR may be indicated. Lightning injury is associated with cardiac arrhythmias, ST segment abnormalities, and other ECG changes. Lower extremity paralysis following lightning injury is termed keraunoparalysis and is usually temporary. Cataracts are a common ocular complication of lightning injury, even in patients without evidence of direct injury to head or eyes. Development of cataracts may be delayed for weeks to years. Tympanic membrane perforation can be seen due to blast injury. Unlike high-voltage electrical injuries, electrical injury from lightning tends to be superficial and skeletal muscle injury is rare in the absence of associated blunt trauma. Hence, compartment syndrome is not associated with lightning injuries. 31% of users answered correctly.

A 37-year-old man is brought in by EMS from a golf course after a lightening strike. He was rapidly resuscitated on the scene. Which of the following is LEAST likely to be observed in this patient? A Cataracts B Compartment syndrome C ECG abnormalities D Lower extremity paralysis E Tympanic membrane rupture

The correct answer is D. This man presents with postextraction alveolar osteitis and should have a panorex radiograph to rule out retained root tip or foreign body and then be given an inferior alveolar nerve block, irrigation and packing with oil of clove-soaked gauze, and be seen by his dentist in 24 hours. Explanation: Although third molar extraction can be complicated by deep masticator space infection, this is unlikely in the afebrile patient without trismus or gingival edema and thus a CT and intravenous antibiotics are not indicated. Third molar infection is not usually associated with the sublingual/submandibular space infections. Ludwig angina typically arises from more anterior periodontal infections. This patient does have postoperative pain and could probably be managed with local anesthesia and oral analgesics, but this patient's presentation is most consistent with postextraction alveolar osteitis or "dry socket" and is most properly managed with a panorex radiograph to rule out retained root tip or foreign body and then be given an inferior alveolar nerve block, irrigation and packing with oil of clove-soaked gauze, and be seen by his dentist in 24 hours. Thrombin-impregnated gauze is not indicated in the absence of bleeding. 38% of users answered correctly. Clove oil and eugenol, one of the chemicals it contains, have long been applied to the teeth and gums for toothache, but the U.S. Food and Drug Administration (FDA) has reclassified eugenol, downgrading its effectiveness rating. The FDA now believes there is not enough evidence to rate eugenol as effective for toothache pain.

A 37-year-old smoker presents with severe pain at the site of his third mandibular molar extraction 2 days ago. He is afebrile and has no trismus, facial, or gingival swelling. The space at the site of his third molar is without bleeding or exudate but is exquisitely tender. Which of the following is TRUE? A This man presents with common postextraction pain and should be given an inferior alveolar nerve block and analgesics and be seen by his dentist in 24 hours. B This man presents with early postextraction infectious complication and should have a CT scan to rule out masticator space infection and then should be treated with IV antibiotics and analgesics and be seen by his dentist in 24 hours. C This man presents with postextraction alveolar osteitis due to fibrin clot loss at the site of extraction and is at risk for bleeding. He should be given an inferior alveolar nerve block, irrigation and packing with thrombin impregnated gauze, and be seen by his dentist in 24 hours. D This man presents with postextraction alveolar osteitis and should have a panorex radiograph to rule out retained root tip or foreign body and then be given an inferior alveolar nerve block, irrigation and packing with oil of clove-soaked gauze, and be seen by his dentist in 24 hours. E This man presents with postextraction mandibular osteomyelitis and is at risk for developing Ludwig's Angina. He should undergo CT and treated with IV antibiotics and analgesics and urgent dental referral.

The correct answer is A. Hypertension is the most common risk factor for placental abruption. Explanation: Hypertension is the most common risk factor for placental abruption. Trauma, smoking, advanced maternal age, African American race, and cocaine use are other risk factors. Ultrasound is specific but not sensitive for the detection of a retroplacental clot. Antenatal administration of corticosteroids before 34 weeks, not 32 weeks, of gestation speeds lung maturity and decreases the incidence of intraventricular hemorrhage and necrotizing enterocolitis. Tocolytics, like magnesium, may be considered in preterm rupture of membranes but not in placental abruption. 58% of users answered correctly.

A 38-year-old African American female G3P2 at 32 weeks with history of pregnancy-induced hypertension presents with sudden onset of severe lower abdominal pain. Which of the following statements is most correct regarding this patient's condition? A Hypertension is the most common risk factor for placental abruption. B Race is not a risk factor associated with placental abruption. C The patient does not need intravenous steroids for fetal lung maturity since she is 32 weeks pregnant. D Tocolytics such as magnesium should be administered as soon as possible. E Ultrasound is sensitive but not specific for placental abruption.

The correct answer is D. Place ice packs on groin, give analgesia, and attempt reduction Explanation: This patient reveals signs of an incarcerated hernia. If the hernia is incarcerated, but the patient does not yet show signs of strangulation, then try one or two attempts at reduction in the ED. Steps for hernia reduction include (1) NPO status in case reduction attempts are unsuccessful, (2) adequate IV narcotic analgesia, (3) proper positioning with the patient supine in mild Trendelenburg position, and (4) apply cold packs to the hernia site to reduce swelling and make reduction attempts easier. Grasp and elongate the hernia neck with one hand, and with the other hand, apply firm, steady pressure to the distal part of the hernia. Applying pressure on the most distal part of the hernia can cause bulging of the neck and prevent reduction. If the hernia is exquisitely tender and is associated with systemic signs and symptoms, such as intestinal obstruction, toxic appearance, peritonitis, or meets sepsis criteria, then assume hernia strangulation. Consult general surgery immediately. 80% of users answered correctly.

A 38-year-old male presents with scrotal pain after lifting heavy boxes while at work. On exam, you detect an indirect inguinal hernia. He is nontoxic appearing, has normal vitals and a soft abdomen. He is in mild pain and has some tenderness when you examine him. What should be your next step? A Call surgery for admission B Call surgery to take patient to the OR C Order a CT scan D Place ice packs on groin, give analgesia, and attempt reduction E Send a CBC, lipase, and chemistry panel and observe

The correct answer is B. Vitamin B1 Explanation: Wernicke-Korsakoff syndrome is a complication seen most often in malnourished chronic alcoholics who are suffering from a vitamin B1 deficiency. Wernicke encephalopathy is believed to be the result of an acute deficiency thiamine. Korsakoff syndrome is believed to be a chronic long-term sequela of Wernicke encephalopathy. The administration of thiamine prior to administering D50 to avoid precipitation of Wernicke encephalopathy is unfounded. Thiamine is excreted in the urine and therefore toxic levels are relatively unheard of. 54% of users answered correctly.

A 39-year-old alcoholic presents with ataxia, confusion, and confabulations. What vitamin deficiency is causing these symptoms? A Vitamin A B Vitamin B1 C Vitamin B6 D Vitamin B12 E Vitamin C

The correct answer is D. Ocular decongestant drops Explanation: This patient has a classic case of viral conjunctivitis. Supportive care is in order. Ocular decongestants and artificial tears can provide some relief. Symptoms may take weeks to resolve. Antibiotics are not warranted when there is no sign of purulent discharge. Flare, cell and photophobia are signs of iritis, which should trigger a workup for systemic disease and is treated with steroids and cycloplegics. This patient has a clear anterior chamber. 66% of users answered correctly.

A 39-year-old woman comes in complaining of red eyes and watery discharge. On exam, there is conjunctival injection, chemosis, watery discharge, as well as auricular adenopathy. No uptake was noted with fluorescein staining and anterior chamber was clear on slit-lamp exam. Which of the following is recommended? A Antibiotic drops B Cycloplegic drops C Needle biopsy of the lymph node D Ocular decongestant drops E Urgent ophthalmology consult

The correct answer is D. Give intravenous narcotics, clindamycin, and order a CT scan. Explanation: Trismus without swelling suggests pterygomandibular space abscess which can be diagnosed by CT. Constitutional symptoms of masticator space infections include fever, malaise, pain, dehydration, and no visible edema. Airway compromise is rare but should be considered if the trismus does not abate with pain medication; the patient is vomitting; there is a large phlegmon on CT; or other signs of impending airway compromise. Clindamycin or ampicillin-sulbactam would be the antibiotics of choice as the offending organism is usually anaerobic. 45% of users answered correctly.

A 40-year-old homeless man returns to the ED for the third visit in 5 days having been diagnoses with pharyngitis and given penicillin that was then switched to clindamycin on the last visit 2 days ago complaining of persistent and worsening sore throat and having run out of his pain medications. On exam, he has fever and mild tachycardia; there is no facial swelling. He can open his mouth only 2 finger breaths and oropharynx reveals dry erythematous mucous membranes without lesion, normal soft palate, and tonsils except for marked erythema bilaterally; he is handling his secretions but wincing and speaking with a soft voice secondary to "pain." What is the most appropriate course of action? A Change his antibiotics to a macrolide noting concerns for "drug seeking behavior" and discharge home. B Emergently intubate the patient, give intravenous antibiotics, and order a CT scan. C Give intravenous narcotics, cephalexin, and order a CT scan. D Give intravenous narcotics, clindamycin, and order a CT scan. E Perform a 3-point needle aspiration.

The correct answer is E. Relapse of P. vivax Explanation: Malaria caused by P. vivax is typically successfully treated by chloroquine. However, both P. vivax and Plasmodium ovale develop an exoerythrocytic phase known as a hypnozoite that lays dormant in the liver. Chloroquine is not active against the hypnozoite form, and therefore most patients diagnosed with P. vivax are also treated with primaquine. Without primaquine treatment, patients may have a relapse of infection after reactivation of the hypnozoite form. P. falciparum does not have a hypnozoite form. Recent illness with malaria should heighten suspicion, even after treatment, and symptoms should not be quickly attributed to an influenza-type illness. Fever in a returning traveler after 14 days essentially rules out the diagnosis of dengue fever. Recrudescence refers to a reappearance of malaria in days or weeks after apparently successful treatment, typically due to treatment failure of P. falciparum. 45% of users answered correctly.

A 40-year-old male presents complaining of intermittent fever, myalgias, and malaise for 8 days. The patient informs you that he had returned from India 6 months ago and had a similar illness diagnosed as malaria that was treated with chloroquine. What process is MOST likely? A Dengue fever B Influenza virus after successful treatment of malaria C Recrudescence of Plasmodium falciparum D Relapse of P. falciparum E Relapse of P. vivax

The correct answer is E. Vibrio vulnificus Explanation: V. vulnificus and Vibrio parahaemolyticus are the major bacteria associated with soft-tissue infections after exposure to seawater. Cases are predominantly found along the Gulf Coast, but can be seen on both the East and West Coasts as well. V. vulnificus often involves bullous lesions and should be suspected when the presentation suggests necrotizing fasciitis. It is often an aggressive process, especially in patients with liver disease, end-stage renal disease, or other immunosuppression. Aggressive resuscitation is warranted as patients tend to have cardiovascular collapse. History of exposure to seawater is important, as the antibiotic of choice, doxycycline, is not typically used for necrotizing fasciitis and septicemia. As with all necrotizing wounds, consultation for surgical debridement is prudent. Aeromonas is associated with freshwater exposure. Wounds due to Clostridium are less common due to improved hygiene. E. rhusiopathiae is related to contaminated meat or fish exposure. Immunocompromised patients are at risk for S. epidermidis deep space infections. 61% of users answered correctly.

A 40-year-old male with a history of end-stage liver disease presents with fever, swelling, and extreme leg pain. He sustained a minor abrasions to the area 2 days prior while swimming in the Gulf of Mexico. His temperature is 39.0°C, blood pressure is 94/50, and his heart rate is 122. His right leg is severely swollen with bullous skin lesions (Figure 10-3). What is the MOST likely pathogen? A Aeromonas species B Clostridium species C Erysipelothrix rhusiopathiae D Staphylococcus epidermidis E Vibrio vulnificus

The correct answer is C. Immediate surgical consultation Explanation: This patient has a presentation suggestive for streptococcal TSS with necrotizing fasciitis. Both diabetes and chronic alcoholism are associated with more severe disease. Immediate surgical consultation is prudent for both diagnosis and treatment. Necrotizing soft-tissue infections carry an extremely high mortality. Earlier operative treatment involving debridement, fasciotomy, and even amputation improves outcomes. Broad-spectrum intravenous antibiotics and fluid resuscitation are indicated. Imaging with a CT scan or MRI may be useful in patients who are not critically ill and in whom the diagnosis is not certain. It would only delay definitive care in this case. The patient is showing signs of sepsis and cannot be treated with bedside incision and drainage. Streptococcal TSS is often associated with severe infection and blood cultures are often positive. However, intravenous antibiotics cannot substitute for emergent surgical debridement. Alcohol withdrawal may eventually be a concern, but is not currently. 60% of users answered correctly.

A 40-year-old male with diabetes and chronic alcoholism presents complaining of extreme pain in his left shoulder. He states he may have bumped into a wall 2 days ago, but it did not start hurting until today. His temperature is 39.0°C, blood pressure 88/50, and heart rate 120. He has only mild erythema and swelling on his left shoulder. However, his exam is significant for severe pain of his left shoulder with decreased range of motion. The pain appears out of proportion to your exam. He has a noticeably diffuse erythroderma. His white blood cell count is 20,000 with 40% bands. What is the MOST critical step in treatment? A Bedside incision and drainage and discharge on oral antibiotics B CT scan with intravenous contrast of the left upper extremity C Immediate surgical consultation D Intravenous antibiotics E Intravenous benzodiazepines

The correct answer is B. Fluconazole Explanation: Odynophagia with a CD4 count fewer than 100 and oral thrust strongly suggests esophageal candidiasis. If the patient can tolerate oral therapy, fluconazole is the first-line treatment. Endoscopy is reserved for treatment failure or atypical presentation. Cytomegalovirus and HSV are diagnosed by endoscopy, and are treated with ganciclovir and acyclovir, respectively. Patients with HIV can have esophagitis due to reflux, but in this setting, empiric treatment for candidiasis is the most appropriate. 74% of users answered correctly.

A 44-year-old female with AIDS presents complaining of pain when she swallows, but is able to tolerate liquids and solids. The pain radiates down her chest. She appears nontoxic. On exam, you notice a white plaque on her tongue and oropharynx that is easily scraped off. Her CD4 count is 100. What is the MOST appropriate treatment? A Acyclovir B Fluconazole C Ganciclovir D Immediate endoscopy E Proton pump inhibitor

The correct answer is E. Volume expansion with 500-1000 mL of NS. Explanation: This patient has cardiac tamponade, as indicated by the narrow pulse pressure, pulsus paradoxus, electrical alternans, and RV collapse in diastole on echocardiogram. However, she is hemodynamically stable. Pericardiocentesis is the definitive therapy for cardiac tamponade and should be performed either emergently or urgently with echocardiographic guidance. Fluid samples should also be sent for studies to rule out malignancy, which is the most common cause of nontraumatic pericardial tamponade. Patients with pericardial tamponade have intrapericardial pressures, which exceed normal filling pressures, as seen by RA and RV diastolic collapse. Preload must be maintained to support filling pressure, so the EP can temporize with a 500-1000 mL of NS. Therapies that would decrease preload, such as diuretics or vasodilators, would worsen cardiac tamponade and should be judiciously avoided. 65% of users answered correctly.

A 44-year-old woman presents with dyspnea at rest and with exertion for the past 3 months. Vital signs are BP 1080/80, HR 95, RR 22, T 98.6, room air saturation of 97%. The patient is alert and not in extremis. Physical exam findings include jugular vein distention, soft heart sounds, clear lungs, and a pulsus paradoxus of 25 mm Hg. An ECG shows low-voltage QRS complexes and electrical alternans. While arrangements are made for admission and an echocardiography-guided pericardiocentesis with pigtail catheter insertion, what would be an appropriate temporizing ED treatment. A Furosemide intravenously B Nitroglycerin intravenously C Nitroglycerin sublingual D Nitroprusside intravenously E Volume expansion with 500-1000 mL of NS.

The correct answer is E. Trimethoprim-sulfamethoxazole can trigger a flare of this disease. Explanation: The picture shows Heinz bodies, often seen in patients with glucose-6-phosphate dehydrogenase (G6PD) deficiency. Several medications, including trimethoprim-sulfamethoxazole, can induce acute hemolysis. G6PD deficiency is the most common enzymopathy of red blood cells in humans, affecting more than 400 million people worldwide. All races are affected; however, this disease has mostly been found in persons of African, Asian, and Mediterranean descent. G6PD deficiency is an X-linked disease, affecting mostly boys and men. There are five variants of the disease, as defined by the World Health Organization. 54% of users answered correctly.

A 45-year-old Asian man presents to the emergency department complaining of progressive shortness of breath and fatigue. A smear of his blood is shown below (Figure 9-1). Which of the following is TRUE regarding the patient's likely diagnosis? A Caucasians are predominantly affected. B It is inherited in an autosomal-dominant fashion. C It is caused by a rare enzyme deficiency. D There are three clinical variants of this disease. E Trimethoprim-sulfamethoxazole can trigger a flare of this disease.

The correct answer is C. Offer a prescription for nabumetone (Relafen). Explanation: The patient exhibits several features of drug-seeking behavior, but he may have true pain. Offering a nonnarcotic oral medication would be the best thing. A dental block would also be appropriate, but lidocaine is too short acting to be adequate. Refilling a narcotic medication is not the best choice in this type of patient; additionally, the patient states that he is allergic to acetaminophen (a component of Vicodin). One study showed that patients who were refused opioids at one facility were successful in obtaining opioids at another facility 93% of the time and were later successful at obtaining opioids from the same facility 71% of the time; thus, giving a "one time" shot of narcotics is probably not a good strategy.

A 45-year-old male presents to the emergency department on a Friday night with a complaint of dental pain requesting a refill of his acetaminophen and hydrocodone (Vicodin) prescription. He has had several visits for the same problem over the past 3 months. He states he is allergic to ibuprofen (Motrin), acetaminophen (Tylenol), codeine, and aspirin, all of which cause nausea. His exam shows normal vital signs but multiple dental caries without clear abscess. Which is the MOST appropriate treatment for this patient from among those listed? A Give him an injection of hydromorphone (Dilaudid) if he promises that he will not return to your facility for narcotics in the future. B Offer a dental block with lidocaine and refer him to a dentist in the morning. C Offer a prescription for nabumetone (Relafen). D Refill his prescription for acetaminophen and hydrocodone (Vicodin) as requested. E Refuse any pain medicine; the patient is drug seeking.

The correct answer is A. Catheter removal Explanation: This patient has characteristics suggestive of severe sepsis. Source control is a key component of successful therapy in sepsis. Definitive treatment of sepsis would be removal of the nidus of infection if feasible. Patients receiving dialysis through temporary external catheters are at higher risk for bacteremia than those with AV fistulas. Coagulase-negative staphylococcus and S. aureus comprise the majority of infections. Enterococcus and enteric gram-negative organisms must also be considered. While this patient should receive fluid resuscitation and antimicrobial therapy, this patient is unlikely to recover without removal of the source of infection. It would be prudent to assess this patient's chemistry panel, but it is more likely that he is suffering from sepsis than dialyzable conditions such as uremia or hyperkalemia. 73% of users answered correctly.

A 45-year-old male with end-stage renal disease presents to the ED with 2 days of fever up to 39.0°C. His initial blood pressure is 74/40 and his heart rate is 130. There is a dialysis catheter in his right internal jugular vein. There is mild erythema around the site but no frank pus. His last dialysis was 3 days ago. What intervention is necessary for definitive treatment of this infection? A Catheter removal B Emergent dialysis C Intravenous fluids D Levofloxacin 750 mg intravenously E Vancomycin 1 g intravenously

The correct answer is B. Vitamin B3 Explanation: Vitamin B3, niacin, is both a prescribed and over-the-counter therapy for high cholesterol. In doses greater than 100 mg, niacin ingestion triggers a histamine release with resultant vasodilation causing uncomfortable face, neck, and chest flushing and itching. Antihistamines can be used to ameliorate the symptoms. Vitamin B1, thiamine, is not generally stored in the body in significant quantities and thus does not generally cause problems with large dose ingestions. Vitamin D toxicity generally requires massive doses (1000-3000 IU/kg) with symptoms manifesting in 2-8 days. Patients can present with anorexia, nausea, abdominal pain, lethargy, weight loss, polyuria, constipation, confusion, and coma. Treatment involves discontinuing vitamin D supplements and the reduction in calcium intake. Vitamin E is composed of eight different fat-soluble compounds. At high doses, vitamin E increases vitamin K requirements and inhibits platelet aggregation. Fatigue, nausea, and blurry vision may also be encountered in adults who ingest toxic doses of vitamin E. Vitamin K is important in several steps of the coagulation pathway. Toxic doses can cause hemolytic anemia, kernicterus, and hemoglobinuria in premature infants. In adults, it may cause renal and liver damage. 49% of users answered correctly.

A 46-year-old male with high cholesterol presents with face, neck, and chest burning, itching and redness after taking something from an herb shop to help his cholesterol. Which of the following is the etiology for this patient's symptoms? A Vitamin B1 B Vitamin B3 C Vitamin D D Vitamin E E Vitamin K

The correct answer is E. Singed nasal hair, soot in mouth, and carbonaceous sputum are useful indicators of smoke inhalation Explanation: Inhalation injury is suspected based on history and presence of physical signs of airway involvement including facial burns, singed nasal hair, soot in mouth, carbonaceous sputum, hoarseness, and wheezing. Upper airway edema can occur rapidly and presents early. Stridor may be present. In contrast, lower airway edema may not be evident for up to 24 hours and respiratory signs may be absent initially. Accordingly, the initial chest radiograph may be normal. Up to 50% of intubated burn patients will go on to develop acute respiratory distress syndrome (ARDS). COHb should be measured in all patients suspected of inhalation injury and may help confirm prolonged smoke exposure. Circumferential neck burns are an indication for early intubation due to the potential for airway edema. Other indications for early intubation include full-thickness burns of the face or perioral region, respiratory distress, progressive hoarseness, stridor or air hunger, respiratory depression or AMS, supraglottic edema, and inflammation on laryngoscopy. 91% of users answered correctly.

A 48-year-old alcoholic woman is brought in by EMS from a house fire. The patient is awake and on oxygen. Which of the following is true in evaluating and managing her respiratory status? A Absence of stridor or wheezing excludes lower airway edema B Carboxyhemoglobin (COHb) measurement is not helpful C Inhalation injury is excluded if the initial chest radiograph is normal D Intubation should be avoided if she has circumferential neck burns E Singed nasal hair, soot in mouth, and carbonaceous sputum are useful indicators of smoke inhalation

The correct answer is B. Calcium chloride Explanation: While all the answers are therapeutic interventions, it is important to recognize which drug should be given first in a patient with elevated potassium and a wide QRS. Calcium chloride is a cardiac membrane stabilizer and has an onset of action within 1-3 minutes and lasts between 30 and 50 minutes. Calcium carbonate may also be used, but calcium chloride has three times the amount of calcium as calcium carbonate. Albuterol may be given to help shift potassium intracellularly, but its onset of action is approximately 15-30 minutes. It lasts between 2 and 4 hours. Insulin and glucose also work to shift potassium into the cell and has an onset of action of approximately 30 minutes and lasts between 4 and 6 hours. Sodium bicarbonate also shifts potassium intracellularly and works within 5-10 minutes and lasts between 1 and 2 hours. Sodium polystyrene sulfonate acts to increase GI potassium excretion. Its time of onset is 1-2 hours and its effects last 4-6 hours. 71% of users answered correctly.

A 49-year-old male with a history of chronic renal insufficiency is referred to the ED for palpitations and fatigue. His potassium level was found to be 6.1 by his primary doctor. Upon arrival, the patient has a heart rate of 56, a widened QRS on ECG, and a BP of 110/76. He is speaking to you, his lungs are clear bilaterally, and his oxygen saturation is 100% on 2 L NC. He has a large bore IV and is placed on the monitor. Which of the following is the most appropriate initial treatment for this patient with severe hyperkalemia? A Albuterol nebulizer B Calcium chloride C Insulin and glucose D Sodium bicarbonate E Sodium polystyrene sulfonate

The correct answer is A. Extract the loose tooth and follow up with a dentist within 24 hours. Explanation: Avulsed primary teeth are never reimplanted. Severe luxations generally require extraction, as they are a risk for aspiration and risk damage to the underlying teeth. Posttraumatic sequelae are variable and require close dental follow-up. 25% of users answered correctly.

A 5-year-old boy comes in 20 minutes after having tripped into a table edge; his mother brings in one front tooth suspended in milk. His physical exam is normal except for ecchymotic gingiva over his anterior alveolar ridge, which is stable. His one incisor is completely avulsed and suspended in milk. His other incisor is extremely loose but still in the socket. You should: A Extract the loose tooth and follow up with a dentist within 24 hours. B Extract the loose tooth, soak both teeth in Hank's balanced salt solution for 30 minutes, then push the teeth firmly into the tooth sockets, securing them to the surrounding teeth with periodontal dressing, and follow up with a dentist within 24 hours. C Leave the avulsed tooth out, secure the loose incisor to surrounding teeth with periodontal dressing, and follow up with a dentist within 24 hours. D Leave the loose tooth and follow up with a dentist within 24 hours. E Push the avulsed tooth firmly into the tooth socket, securing them to the surrounding teeth with periodontal dressing, and follow up with a dentist within 24 hours.

The correct answer is A. COHb level Explanation: Exposure to propane-fueled heaters is a common source of CO. Frequently whole families may fall ill. COHb level is likely to be abnormal and would explain this child's symptoms. Positive influenza swab is less likely given the exposure history. CT of the brain can show hypoattenuation in the globus pallidus after significant CO exposure, but these lesions may take up to 24 hours to develop. Chest radiograph is typically normal. Pulse oximetry does not reflect the systemic hypoxia of CO poisoning because the device cannot distinguish between oxyhemoglobin and COHb. 84% of users answered correctly.

A 5-year-old child is brought in by parents for headache, vomiting, and dyspnea but no fever. The parents have similar but milder symptoms. They have been staying in their mountain cabin that is heated by a propane heater. Which of the following tests is most likely to be abnormal? A COHb level B Chest radiograph C CT of the brain D Influenza swab E Pulse oximetry

The correct answer is E. Spray the nares with a vasoconstrictor, apply steady pressure for 15 minutes; if the bleeding continues, then place thrombogenic foam to the bleeding surface prior to trying nasal tamponade. Explanation: High blood pressure usually resolves as the epistaxis and patient's anxiety are controlled. Persistent, extremely high blood pressure should be addressed once the epistaxis is brought under control. Current treatment algorithm recommendations for acute anterior epistaxis are have the patient blow out all clots, spray the nares with vasoconstrictor, apply steady pressure for 15 minutes, and if direct pressure fails, chemical cautery is the next appropriate step with a silver nitrate stick. Thrombogenic foams and gels are becoming an increasing good option to be considered when chemical cautery fails. If the above measures fail, nasal tamponade with sponge or balloon is indicated. Nasal sponge and antistaphylococcal antibiotics are appropriate treatments for epistaxis that does not subside with direct pressure. Aspirin and nonsteroidal anti-inflammatory drugs should be avoided for 3-4 days. 43% of users answered correctly.

A 50-year-old man with a history of hypertension comes in complaining of spontaneous copious bleeding from his nose over the past 2 hours despite applying pressure. On physical exam, his blood pressure is 190/100, other vital signs are normal. He has bright red blood oozing from the septum in the right nares. What is the most appropriate next step? A Administer an antihypertensive medication, then spray the nares with a vasoconstrictor, apply steady pressure for 15 minutes; if the bleeding continues, place a 5-cm nasal sponge and discharge with cephalexin and follow up in 2 days. B Administer an antihypertensive medication, then spray the nares with a vasoconstrictor, apply the tip of a silver nitrate stick with gently pressure for 3 minutes. C Spray the nares with vasoconstrictor, apply steady pressure for 15 minutes; if the bleeding continues, place a 5-cm nasal sponge and discharge with cephalexin, ibuprofen, and follow up in 2 days. D Spray the nares with vasoconstrictor, apply steady pressure for 15 minutes; if the bleeding continues, place a 5-cm nasal sponge and discharge with cephalexin, and follow up in 2 days. E Spray the nares with a vasoconstrictor, apply steady pressure for 15 minutes; if the bleeding continues, then place thrombogenic foam to the bleeding surface prior to trying nasal tamponade.

The correct answer is E. Water deprivation test Explanation: The patient has diabetes insipidus (DI), which can be central or nephrogenic. In DI, patients have increased sodium levels, abnormally low urine osmolality, and an increased serum osmolality. His irritability and tremulousness is a result of his elevated serum osmolality. To determine if the DI is central or peripheral, the patient must undergo a water deprivation test. In DI, the urine will fail to show increasing urine osmolality. Once this occurs, aqueous vasopressin is administered subcutaneously. Central DI will respond to the vasopressin causing an increase in urine osmolality. Patients with nephrogenic diabetes will fail to show increases in their urine osmolality. Patients with hypoaldosteronism, for which an aldosterone level might be useful, often present with postural hypotension, anorexia, weight loss, asthenia, hyperpigmentation, and/or elevated potassium levels. Serum cortisol levels can be utilized to detect Addison disease, hypopituitarism, and Cushing syndrome. A corticotrophin stimulation test is used to differentiate between primary or secondary adrenal insufficiency. A thyroid-stimulating hormone level would help differentiate thyroid abnormalities, which do not usually present with elevations of sodium. 39% of users answered correctly.

A 52-year-old male with no past medical history complains of tremulousness and increased irritability. His primary care provider sent him to the ED because on exam, his sodium was 161 mEq/L, urine osmolality was 226, and his serum osmolality was 283. Which of the following is the most appropriate next step in diagnosis? A Aldosterone level B Cortisol level C Corticotrophin stimulation test D Thyroid-stimulating hormone level E Water deprivation test

The correct answer is D. Imipenem-cilastin Explanation: Lower extremity infections in diabetic patients are classified as non-limb-threatening, limb threatening, or life threatening. Non-limb-threatening infections do not involve deep structures or bone and are often from a recent injury. The patient will have no signs of systemic toxicity or leukocytosis and may be treated with cephalexin, clindamycin, dicloxacillin, or amoxicillin-clavulanate. Patients with limb-threatening infections will have a wound with greater than 2 cm of surrounding cellulitis, ascending lymphangitis, an area of large necrotic tissue, involvement of bone or deep structures, full-thickness ulceration or abscess, and gangrene surrounding the ulcer or lower extremity ischemia. Limb-threatening patients may be treated with ampicillin-sulbactam, ticarcillin-clavulanate, second-generation cephalosporin, clindamycin plus ciprofloxacin or ceftriaxone, or an oral fluoroquinolone plus oral clindamycin. Patients with a life-threatening infection will exhibit signs of sepsis such as fever, leukocytosis, hypotension, tachycardia, tachypnea, altered mental status, and metabolic abnormalities. They should be treated with imipenem-cilastatin, ampicillin-sulbactam plus tobramycin, or vancomycin plus metronidazole plus aztreonam. 43% of users answered correctly.

A 52-year-old noninsulin-dependent diabetic male presents to the ED complaining of a 4-cm ulcer to his foot. He states that the pain has gotten much worse. It has become progressively red and warm, and he has had fevers up to 103.1°F today. He states that for the last 2 days his blood sugars have been ranging between 300 and 400. His vital signs are BP 101/69, P 122, RR 16, normal oxygen saturation, and an oral temperature of 102.9°F. On exam, the patient has a 4-cm deep ulcer to the medial calf with significant surrounding erythema and 3-4 cm of streaking up the calf. What is the antibiotic of choice for this patient? A Amoxicillin-clavulanate B Ampicillin-sulbactam C Cephalexin D Imipenem-cilastin E Ticarcillin-clavulanate

The correct answer is B. Babesiosis Explanation: Erythema migrans, a well-defined macular rash, often with central clearing, is usually diagnostic of Lyme disease. It is the most common vector-borne zoonotic disease in the United States and occurs predominantly in the northeastern and north central United States, but has been reported in all continental 48 states. Lyme disease is transmitted by the Ixodes scapularis deer tick, which also is the vector for both babesiosis and anaplasmosis. The patient's symptoms are most likely the result of a coinfection with Babesia that occurred at the same time the patient was infected with Lyme disease. While anaplasmosis, which may present as an influenza-type illness, is treated by the same regimen of doxycycline, babesiosis is not. Babesiosis is often asymptomatic, but when present symptoms closely mimic malaria. It is diagnosed by a Wright or Giemsa-stained peripheral blood smear that demonstrates intraerythrocytic parasites (ring forms with a central pallor), known as the Maltese cross. Treatment is atovaquone plus azithromycin. The vector for Colorado tick fever is the wood tick, Dermacentor andersoni, and treatment is mainly supportive. Ehrlichiosis is transmitted by the lone star tick Amblyomma americanum. It typically presents as an influenza-type illness 1-2 weeks after tick bite, but in a minority of patients can progress to renal failure, respiratory failure, and encephalitis. Treatment is with doxycycline. Secondary syphilis presents with nonpruritic, rough, red or reddish brown macules on the trunk and flexor areas and often extends to the palms and soles. 46% of users answered correctly.

A 53-year-old patient is seen in the ED complaining of fever and headache for 2 days. One month ago, he took his family on vacation to eastern Massachusetts. Two weeks ago, he was seen by his primary care provider for the rash depicted in Figure 10-6. He was treated with doxycycline for 21 days. His temperature is 38.5°C. There is no rash. Which of the following is the MOST likely explanation for his current symptoms? A Anaplasmosis B Babesiosis C Colorado tick fever D Ehrlichiosis E Secondary syphilis

The correct answer is D. Place a purse string suture, apply a dry dressing, and recheck in 30 minutes Explanation: Large-volume therapeutic paracentesis is a lengthy procedure associated with complications such as hyponatremia, renal impairment, persistent leakage, and encephalopathy. A purse-string suture can be placed to minimize leakage. Recheck the patient in 30 minutes to identify persistent leakage or an increase in symptoms to suggest a complication. Patients with large-volume paracentesis should be monitored for hypotension after the procedure. Cover the puncture site with a dry dressing for 48 hours. 54% of users answered correctly.

A 54-year-old male with cirrhosis presents with a distended abdomen and shortness of breath. An ultrasound reveals significant peritoneal fluid consistent with ascites. A paracentesis was performed. More than 4 L of fluid was removed from the patient. Once the paracentesis is complete, the patient's nurse notifies you that the paracentesis site is leaking. What is the most appropriate next step in management? A Consult a surgeon B Perform another paracentesis to remove more peritoneal fluid C Place a dry dressing and recheck in 1 hour D Place a purse string suture, apply a dry dressing, and recheck in 30 minutes E Send a chemistry pane

The correct answer is B. Primary PCI in <90 minutes Explanation: Both the AHA and the ACC recommend primary coronary intervention within 90 minutes if possible. If the time is greater than 3 hours from symptom onset, PCI is even more preferred. The door-to-balloon time may be as long as 120 minutes if the patient is transferred to a PCI capable center. 87% of users answered correctly.

A 54-year-old man presents with 6 hours of acute substernal chest pain. He has an ECG within the first 10 minutes of arrival. He has 4 mm acute ST-segment elevation in leads V2-V5. What is the American College of Cardiology (ACC) and American Heart Association (AHA) recommended reperfusion strategy? A Pain control then percutaneous coronary intervention (PCI) once patient is pain free B Primary PCI in <90 minutes C Primary PCI in <120 minutes D Thrombolysis in <90 minutes E Thrombolysis in <120 minutes

The correct answer is D. Right coronary artery Explanation: The presence of ST elevation in lead III greater than in lead II predicts an right coronary artery (RCA) lesion. When this is accompanied by ST elevation in V1 or V4R, it predicts proximal RCA lesions with accompanying right ventricular infarction. 77% of users answered correctly.

A 54-year-old man presents with acute substernal chest pain. His ECG has 4 mm acute ST-segment elevation in lead III and 2 mm ST-segment elevation in lead II. Which of the following is the most likely vessel occluded? A Left anterior descending artery B Left circumflex artery C Left main coronary artery D Right coronary artery E Sinoatrial node artery

The correct answer is B. Erysipelas Explanation: This patient presents with the classic appearance of erysipelas, rapidly spreading raised red puffy appearance sharply defined borders. Cellulitis, chemical or infectious, is more diffuse in appearance. Impetigo presents with amber crusting, as it is more of a superficial epidermal infection. Whether this patient had erysipelas or cellulitis, intravenous ampicillin-sulbactam would be the treatment of choice. The most common pathogen in erysipelas is Streptococcus pyogenes and in cellulitis is S. aureus and S. pyogenes. Facial abscess can be diagnosed with clinical exam or bedside ultrasound and is best treated with incision and drainage. 59% of users answered correctly.

A 54-year-old woman, with a history of alcohol abuse, presents with a well-demarcated 4 cm raised, puffy, warm, erythematous patch without fluctuance on her right cheek that started as a bug bite 6 hours ago. What is the most likely diagnosis? A Cellulitis B Erysipelas C Facial abscess D Impetigo E Panniculitis

The correct answer is E. Administer broad-spectrum antibiotics and drain the submandibular abscess. Explanation: This patient presents with classic sublingual, submandibular space infection known as Ludwig's angina. The tongue can be rapidly displaced posteriorly occluding the airway. The airway of choice is awake fiberoptic endotracheal intubation or awake tracheotomy. A CT of neck and intravenous antibiotics (penicillin, ampicillin-sulbactam and clindamycin) are the preferred choice, and incision and drainage by head and neck surgeon is the appropriate treatment. 56% of users answered correctly.

A 55-year-old diabetic woman complains of 1 week of lower anterior molar pain and now with 24 hours of sore throat, difficulty swallowing, and sweats. On physical exam (see Figure 6-6), she is anxious and having difficulty breathing. She has significant trismus and so the pharynx is not visualized. Her tongue appears elevated and she is unable to protrude it beyond her teeth, and the sublingual space is indurated and elevated. What is the next best course of action? A Administer broad-spectrum antibiotics and drain the submandibular abscess. B Perform a cricothyroidotomy, administer intravenous fluoroquinolone, order a CT scan, and emergent head and neck surgery consultation. C Perform rapid sequence endotracheal intubation, administer intravenous ampicillin-sulbactam, order a CT scan, and emergent head and neck surgery consultation. D Perform rapid sequence endotracheal intubation, administer intravenous fluoroquinolone, order a CT scan, and emergent head and neck surgery consultation. E Perform awake fiberoptic endotracheal intubation, administer intravenous ampicillin-sulbactam, order a CT scan, and emergent head and neck surgery consultation.

The correct answer is E. Zenker diverticulum Explanation: This patient has symptoms of transfer dysphagia (difficulty in initiating a swallow) associated with halitosis and a sensation of fullness in the neck, most consistent with Zenker diverticula. Zenker diverticula typically develop in middle-aged men and result from a progressive out-pouching of the pharyngeal mucosa just above the upper esophageal sphincter. Esophageal strictures occur in the distal esophagus and interfere with lower sphincter function. Patients typically complain of solid food dysphagia a few seconds after swallowing is initiated (transport dysphagia). Esophageal strictures develop over time as a consequence of GERD or other chronic inflammatory conditions. Esophageal neoplasm must be considered in any patient older than 40 years who develops new onset dysphagia. However, given the constellation of symptoms, neoplasm is not the most likely diagnosis. This patient should be referred to a gastroenterologist to rule out a neoplastic process. Plummer-Vinson syndrome is a disorder associated with intermittent solid food dysphagia, esophageal webs, and iron-deficiency anemia. A Schatzki ring is a fibrous, diaphragm-like structure near the gastroesophageal junction that is present in up to 15% of the population. It is the most common cause of intermittent dysphagia with solids. A frequent presenting sign is the esophageal impaction of poorly chewed meat. Related Topics:zenker diverticulum 68% of users answered correctly.

A 55-year-old man presents to the ED complaining of new difficulty with initiation of swallowing. His symptoms are associated with halitosis and a sensation of fullness in his neck. He denies pain with swallowing or a sensation of food "getting stuck." He denies a prior history of heartburn or constitutional symptoms. He does not smoke or drink alcohol. His exam and basic labs, including a CBC, are normal. What is the MOST likely diagnosis? A Esophageal stricture B Neoplasm C Plummer-Vinson syndrome D Schatzki ring E Zenker diverticulum

The correct answer is E. Vancomycin intravenously Explanation: The patient is presenting with cellulitis and has failed outpatient therapy with an oral beta-lactam. He is also showing signs of systemic toxicity and should likely be hospitalized for parenteral therapy. A patient that warrants hospitalization should receive therapy for methicillin-resistant S. aureus (MRSA) and Streptococcus pyogenes.Vancomycin would be an ideal agent. Alternative choices may be linezolid or daptomycin. MRSA with resistance to clindamycin is high in some communities and should not be relied on for severe soft-tissue infections. Trimethoprim/sulfamethoxazole would provide adequate coverage for MRSA in a patient stable for outpatient treatment. However, it has minimal activity against S. pyogenes and should not be sole therapy. Cefazolin and other beta-lactam antibiotics are appropriate treatment for streptococcus and methicillin-sensitive S. aureus cellulitis, but treatment of severe disease should cover MRSA. MRSA in many communities develops rapid resistance to ciprofloxacin, and it should not be first-line treatment. 58% of users answered correctly.

A 58-year-old male presents complaining of pain, swelling, and redness to his right lower extremity that has progressed over 4 days. He was seen by his primary care physician 2 days ago and was prescribed cephalexin. His temperature is 38.3°C, blood pressure is 140/82, heart rate is 118, and respiratory rate is 16. A fingerstick glucose is 140. The white blood cell count is 18,000 cells/mm3. His right lower extremity has a strong dorsalis pedis pulse and is depicted below (Figure 10-2). Which of the following is the MOST appropriate antimicrobial treatment? A Cefazolin intravenously B Ciprofloxacin by mouth C Clindamycin intravenously D Trimethoprim/sulfamethoxazole by mouth E Vancomycin intravenously

The correct answer is A. Admit patient Explanation: This patient has strong evidence of a structural cardiac abnormality, most likely aortic stenosis, and should be admitted for further evaluation of cardiac function. A transthoracic echocardiography is essential. Critical aortic stenosis is associated with a classic triad of chest pain, dyspnea on exertion, and syncope. Older patients with aortic stenosis and syncope who are asymptomatic upon ED presentation should still be admitted due to the increases risk of death. Patients with documented cardiac syncope have a 6-month mortality exceeding 10%. Electrophysiologic testing is done for patients with dysrhythmias, preexcitation, or conduction delays. Tilt-table testing is performed on patients with recurrent unexplained syncope, suspected to have a reflex-mediated etiology. EEGs would be reserved for patients with suspected seizures. 76% of users answered correctly.

A 58-year-old man is brought in by paramedics after a witnessed syncopal episode while walking with friends. He denies headache, chest pain, or shortness of breath prior to the event, or now. He has hypertension and takes lisinopril. Vital signs are BP 150/100, HR 78, RR 18, T 98.6, room air saturation 97%. Physical examination is notable for a harsh, systolic murmur at the right base, which radiates into the neck. His lungs are clear to auscultation. A transthoracic echocardiogram is pending. The ECG shows normal sinus rhythm with left atrial enlargement but is otherwise normal. What is the MOST appropriate next step in management? A Admit patient B Arrange computed tomography (CT) of chest C Arrange EEG D Arrange outpatient electrophysiologic testing E Arrange outpatient tilt-table testing

The correct answer is E. Admit for IV antibiotics and surgery consultation. Explanation: The majority of uncomplicated diverticulitis improves with bowel rest (liquid diet) and antibiotics (Table 7-1). It is estimated that conservative treatment in this group of patients has a 70-100% success rate. In cases where uncomplicated diverticulitis is confirmed with CT, the success rate is 97%. Most patients should be able to follow this regimen as an outpatient. Complicated diverticulitis generally requires admission. In addition to the standard regimen of bowel rest and IV antibiotics, patients will need specific treatments directed at the complications. Complicated diverticulitis is often referred to by the Hinchey Classification scheme: Stage 1 refers to small, confined pericolic or mesenteric abscesses; Stage 2 refers to larger abscesses, often confined to the pelvis; Stage 3 refers to perforated diverticulitis where a ruptured abscess causes purulent diverticulitis; and Stage 4 refers to free perforation with fecal contamination of the peritoneal cavity. 85% of users answered correctly.

A 58-year-old obese female presents with abdominal pain localized to the left lower quadrant. A CT scan of the abdomen and pelvis reveals sigmoid diverticulitis with mesenteric abscess measuring 5 cm. Appropriate disposition of this patient is: A Discharge home with PO antibiotics for 2 weeks B Discharge home with PO antibiotics for 2 weeks with a 48-hour recheck C Discharge home after surgery consultation with PO antibiotics D Admit for 24-hour observation E Admit for IV antibiotics and surgery consultation.

The correct answer is B. Ethacrynic acid 50 mg IV Explanation: Ethacrynic acid, a loop-diuretic, is indicated for treatment of CHF, in patients with a significant sulfonamide allergy. Furosemide, bumetanide, and torsemide are all sulfonamide-containing loop-diuretics. Parenteral dosing of furosemide is preferable in fluid-overloaded patients, since bowel wall edema may diminish gastrointestinal absorption of an oral dose. 39% of users answered correctly.

A 58-year-old woman presents to the ED with a 4 days of increasing shortness of breath, orthopnea, and dyspnea on exertion. She denies chest pain and claims compliance with her hypertension medication, enalapril. She had Stevens Johnson after taking Septra. Vital signs are BP 160/94, HR 90, RR 24, T 98.8, room air saturation 96%. Physical examination finings include bibasilar rales, JVD, and marked peripheral edema. An IV is established. She received sublingual nitroglycerin. What is the next appropriate step in treatment? A Bumetanide 1 mg IV B Ethacrynic acid 50 mg IV C Furosemide 40 mg IV D Furosemide 40 mg PO E Torsemide 10 mg IV

The correct answer is C. Rocky Mountain spotted fever (RMSF) Explanation: RMSF is a disseminated infection with R. rickettsii. The presentation of RMSF may closely resemble disseminated meningococcemia and the untreated mortality rate of RMSF is 5-10%. The petechial rash of RMSF tends to begin on the wrists and ankles and spreads centrally while the converse is typical for meningococcemia. There are unfortunately no rapid tests that can aid in diagnosis in the ED. Diagnosis is by clinical suspicion and treatment should not be delayed. Any organ system can be affected. Factors suggesting RMSF may include rash (although absent in 10-15% of cases), history of tick bite, laboratory abnormalities as in this patient, or travel to areas populated by the Ixodes deer tick (most cases are in the southeastern United States, but are occasionally seen in California and Arizona). Babesiosis is transmitted by the Ixodes tick (same as Lyme disease), and is a protozoal illness that clinically resembles malaria. Brucellosis is a difficult diagnosis. The most common symptoms are fever and malaise, and it can infect any organ system. Exposure to unpasteurized milk or animals, especially cattle, or a prolonged febrile illness should heighten suspicion. Tick-borne relapsing fever is a Borrelia spirochete-mediated disease that typically involves an eschar at the site of the tick bite and presents as alternating febrile and afebrile episodes along with myalgia, arthralgia, abdominal pain, and malaise. Tularemia is transmitted by the same Dermacentor tick species as RMSF. The most common form of the disease involves ulceration at the site of tick bite along with painful regional adenopathy. 73% of users answered correctly.

A 60-year-old female is brought by EMS from home complaining of fever and malaise for 2 days. Upon further questioning, she also complains of feeling confused with a severe headache, muscle aches, and nausea with vomiting. She visited her sister in South Carolina 4 days ago. On exam, she does not appear to have nuchal rigidity; her lungs are clear to auscultation, and her abdomen nontender. However, you notice a petechial rash on her wrists and ankles. Her temperature is 39.0°C, blood pressure is 90/50, heart rate is 120. Preliminary lab results show neutropenia, thrombocytopenia, and hyponatremia. You consider meningococcemia, but what else should you suspect and treat? A Babesiosis B Brucellosis C Rocky Mountain spotted fever (RMSF) D Tick-borne relapsing fever E Tularemia

The correct answer is B. Amitriptyline (Elavil) Explanation: Patients with neuropathic pain such as postherpetic neuralgia frequently do not respond well to standard analgesics and may be resistant to short-acting opioids. Tricyclics such as amitriptyline (Elavil) as well as some anticonvulsants and long-acting opioids have been found to be effective for neuropathic pain. Most need to be titrated in the outpatient setting.

A 60-year-old female presents to the emergency department complaining of pain from postherpetic neuralgia. Acetaminophen (Tylenol) has been ineffective. Which would be the BEST choice for chronic pain control? A Acetaminophen with Codeine No. 3 (Tyco no. 3) B Amitriptyline (Elavil) C Fentanyl (Sublimaze) transmucosal lozenge D Ibuprofen (Motrin) E Ketorolac (Toradol)

The correct answer is B. Intravenous access, fluid resuscitation, administer intravenous antibiotics, remove the nasal packing, and admission. Explanation: The presentation is strongly suggestive for toxic shock syndrome (TSS). Menstrual causes of TSS have decreased since the withdrawal of highly absorbent tampons. Nonmenstrual causes of TSS include wound infections, mastitis, respiratory infections following viral pneumonia, and enterocolitis. Additionally, nasal packing has been associated with S. aureus invasion. The diffuse rash is known as erythroderma. Patients may experience massive vasodilation and cardiac dysfunction, requiring profound fluid resuscitation. The history and the erythroderma are not consistent with anaphylaxis. Vague, diffuse abdominal pain is characteristic of the disease, and while investigation may be indicated, the nasal packing is a likely source and a CT is not initially indicated. Erythroderma is not consistent with a viral exanthem. Vancomycin plus clindamycin is an appropriate antibiotic regimen for staphylococcal TSS. Although patients may develop bleeding complications due to thrombocytopenia, the nasal packing must be removed, as it is the source of the disease. 85% of users answered correctly.

A 60-year-old female returns to the ED 3 days after anterior nasal packing for epistaxis. She now complains of fevers, chills, myalgias, and diffuse abdominal pain. Her temperature is 39.2°C, blood pressure is 82/44, and heart rate is 132. She has a diffuse, raised, blanching, erythematous rash that resembles severe sunburn. What is the MOST beneficial course of treatment? A Intravenous access, blood transfusion, leave the nasal packing in place due to likely disseminated intravascular coagulation, and admission. B Intravenous access, fluid resuscitation, administer intravenous antibiotics, remove the nasal packing, and admission. C Intravenous access, fluid resuscitation, epinephrine for anaphylaxis, and admission. D Order a CT scan of the abdomen to evaluate for abscess. E Reassure the patient she has sinusitis and a viral exanthema and discharge home.

The correct answer is A. Ensure close follow-up to recheck her prothrombin time while on warfarin. Explanation: An awareness of drug-drug interactions is critical to limiting iatrogenic injury. Antibiotics are among the most common medications prescribed by emergency physicians, and so knowledge of the potential complications is prudent. Ciprofloxacin, a commonly used fluoroquinolone for urinary tract infections, is typically a well-tolerated medication, but it has many interactions to consider. Ciprofloxacin is a known inhibitor of cytochrome P450 and can enhance the activity of many other medications. This list includes some that have a narrow therapeutic index, such as warfarin and theophylline. The interaction with warfarin is not consistent or predictable. Doses do not need to be initially adjusted, but the prothrombin time should be followed closely. Fluoroquinolones also can cause QT prolongation that is usually clinically insignificant. However, patients with prolonged QT, or who also take amiodarone, procainamide, sotalol, or another drug that may prolong the QT interval, have a small but increased risk of torsades de pointes when also taking a fluoroquinolone. There have been case reports detailing prolonged hypoglycemia due to concomitant use of a fluoroquinolone while taking an oral sulfonylurea, such as glyburide. Absorption of fluoroquinolones, as well as the tetracyclines, is inhibited by coingestion of divalent cations. Patients should avoid taking iron, calcium, or antacids at the same time as fluoroquinolones. Furosemide has an increased rate of ototoxicity when taken along with aminoglycosides. 59% of users answered correctly.

A 60-year-old female with diabetes, congestive heart failure, and atrial fibrillation is diagnosed with a urinary tract infection. She is on multiple medications. You plan to prescribe ciprofloxacin, but due to drug-drug interactions, what else should you do? A Ensure close follow-up to recheck her prothrombin time while on warfarin. B Hold her digoxin due to prolonged QT and risk for torsades de pointes. C Hold her furosemide to decrease the risk of ototoxicity. D Increase her glyburide due to hyperglycemia. E Recommend that she continues her antacids to decrease the risk of gastritis.

The correct answer is E. Small bowel obstruction Explanation: Pain preceding nausea and vomiting is associated with an obstructive process. The content of the emesis is helpful in determining if an obstruction is present. Bilious emesis is associated with small bowel obstruction. Esophageal disorders such as Achalasia result in regurgitation of swallowed food particles. Gastric outlet obstruction results in emesis composed of food particles, but devoid of bile. Large bowel obstruction is associated with emesis of feculent material. While it is important to consider acute coronary syndrome in elderly patients with epigastric abdominal pain, a normal ECG and the presence of bile in the emesis point to small bowel obstruction as the most likely cause. Related Topics:small bowel obstruction 75% of users answered correctly.

A 60-year-old male presents to your ED complaining of nausea and vomiting. Two hours ago he had the onset of epigastric abdominal pain. About 1 hour ago he started having intense nausea and multiple episodes of bilious emesis. He denies any history of coronary artery disease or NSAID use. His ECG done at the time of presentation is normal. What is the most likely cause of his emesis? A Achalasia B Gastric outlet obstruction C Large bowel obstruction D Myocardial infarction E Small bowel obstruction

The correct answer is C. Mitral regurgitation Explanation: Acute mitral regurgitation due to papillary muscle dysfunction causes a harsh apical systolic murmur that starts with the first heart sound but ends before the second. Acute mitral regurgitation due to chordae tendineae rupture causes a soft apical holosystolic murmur that radiates to the axilla, but is often obscured by rales, or ambient noise in the ED. Acute ventral septal defect is characterized by a new loud holosystolic left parasternal murmur, usually with a palpable thrill that diminishes as the interventricular pressures equilibrate. Acute aortic insufficiency produces a high-pitched blowing diastolic murmur immediately after S2 and heard best at the left parasternal border. 69% of users answered correctly.

A 60-year-old man presents to the ED with acute onset of chest pain and dyspnea for the past 20 minutes. He has a past medical history of adult onset diabetes mellitus. ECG indicates acute STEMI. Vital signs are BP 90/74, HR 123, RR 26, and room air SaO2 94%. Physical examination shows an ashen, diaphoretic, tachypneic man. He has cool mottled skin, JVD, rales, and a new harsh apical systolic murmur that begins with the first heart sound but ends before the second. Prior medical records do not document any murmurs. Which of the following is the MOST likely diagnosis? A Aortic insufficiency B Aortic stenosis C Mitral regurgitation D Pulmonary embolism E Ventral septal defect

The correct answer is B. Diabetic neuropathy Explanation: The symptoms of diabetic neuropathy are symmetrical pain and abnormal sensation in the legs. Gentle touch may provoke pain in some patients. Postherpetic neuralgia and complex regional pain syndrome are unlikely to be symmetrical as they follow episodes of herpes zoster or injury, respectively. Sciatic is also more commonly unilateral and follows the S1 dermatome. Fibromyalgia is defined by the presence of 11 of 18 specific tender points, none of which are below the knee, nonrestorative sleep, muscle stiffness, and generalized aching pain, with symptom duration of more than 3 months.

A 62-year-old man with hypertension, diabetes, and hyperlipidemia complains of symmetric numbness and burning pain to bilateral feet. Pain is provoked by gentle touch. What is the MOST likely diagnosis? A Complex regional pain syndrome B Diabetic neuropathy C Fibromyalgia D Postherpetic neuralgia E Sciatica

The correct answer is B. Congestive heart failure Explanation: Patients with chronic lung disease can have elevated BNP levels due to right ventricular strain. The low and high cutoff points for BNP are 100 and 500 pg/mL, respectively. A BNP below 100 has a high negative likelihood ratio for CHF in patients with lung disease. In this patient, the clinical picture of prior MI, rales, JVD, lower extremity edema, and Q waves raise concern for possible congestive heart failure. However, the BNP of 80 pg/mL makes the diagnosis less likely. Echocardiography would be helpful if clinical suspicion remained high for CHF or PE. This patient should have serial troponins to evaluate possible acute coronary syndrome. Possible pneumonia remains high on the differential for this patient. 47% of users answered correctly.

A 62-year-old woman with a history of chronic obstructive pulmonary disease (COPD) and a prior history of inferior MI presents to the ED with 4 days of progressive dyspnea, nonproductive cough, and peripheral edema. She denies chest pain or fever. Vital signs are BP 150/90, HR 115, RR 30, T 100.2, room air saturation 94%. Physical exam findings include JVD, bilateral diffuse rales, wheezing, hepatojugular reflex, and bilateral pitting edema. The ECG shows normal intervals and sinus tachycardia without ST-segment elevation or depression. The chest x-ray shows hyperinflation. The electrolyte panel and first troponin are normal. The brain natriuretic peptide (BNP) is 80 pg/mL. What is the LEAST likely diagnosis for this patient? A Acute coronary syndrome B Congestive heart failure C COPD exacerbation D Pneumonia E Pulmonary embolism (PE)

The correct answer is B. CT scan Explanation: In stable patients with a history of confirmed diverticulitis and a similar acute presentation, no further diagnostic evaluation is necessary unless the patient fails to improve with conservative medical treatment. If a prior diagnosis has not been confirmed or the current episode differs from the past episode, diagnostic imaging is required to rule out other intra-abdominal pathology and evaluate for complications. CT is the preferred imaging modality given its ability to evaluate the severity of disease and the presence of complications. 65% of users answered correctly.

A 64-year-old Caucasian male presents with new left-sided abdominal pain for 1 day. He has no prior episodes of this pain. He also complains of constipation in addition to his pain. On exam, he has reproducible tenderness in the left lower quadrant. His urinalysis is negative. A CBC reveals a mild leukocytosis with a white cell count of 12,000 no bands or left shift. After receiving pain medication, the symptoms improve. What is the most appropriate next step? A Admit to the hospital for observation B CT scan C Discharge home with antibiotics D Surgery consultation E Ultrasound

The correct answer is A. Admission for observation Explanation: Hypoglycemia is a known complication in the treatment of diabetes. In patients who are taking sulfonylureas, a search for underlying causes should be sought. This patient has no evidence of an infection, so treatment with antibiotics is not indicated. Patients on sulfonylureas, medium- to long-acting insulins, or meglitinides should be admitted for serial glucose monitoring and treatment. 68% of users answered correctly.

A 64-year-old female is brought in by her family for right-sided paralysis. The family states the patient was watching TV and had sudden onset of slurred speech 35 minutes ago. When she tried to get up and walk, she was unable to move her right side. The family states that she thinks she has had the flu for a few days but no fever, cough, or abdominal pain. She has a past medical history of diabetes and hypertension. She takes metformin, glyburide, and metoprolol. She has not had any changes in her medications. Her vital signs show a BP of 169/93, heart rate of 56, RR of 12, and her pulse oximetry of 99% on 2 L. She is speaking to you but has a right-sided facial droop and a right-sided motor deficit. The patient's right-sided paralysis resolves after intervention. She is sitting comfortably with her family, eating lunch, and asking when she can go home. Her CBC, chemistry panel, chest x-ray, and urinalysis are all normal. Which of the following is the most appropriate next step in the management of this patient? A Admission for observation B Broad-spectrum antibiotics and admission C Discharge home with oral antibiotics D Discharge home with precautions to return to ED. E Observe for another 2 hours and if normal finger stick, discharge home.

The correct answer is C. Glucose level Explanation: Although neurological causes such as cerebrovascular events and transient ischemic attacks are high on the differential diagnosis, it is important to recognize that hypoglycemia may manifest in a stroke-like syndrome. In any patient where stroke is suspected, a bedside glucose level should be attained prior to the patient receiving further evaluation. 82% of users answered correctly.

A 64-year-old female is brought in by her family for right-sided paralysis. The family states the patient was watching TV and had sudden onset of slurred speech 35 minutes ago. When she tried to get up and walk, she was unable to move her right side. The family states that she thinks she has had the flu for a few days but no fever, cough, or abdominal pain. She has a past medical history of diabetes and hypertension. She takes metformin, glyburide, and metoprolol. She has not had any changes in her medications. Her vital signs show a BP of 169/93, heart rate of 56, RR of 12, and her pulse oximetry of 99% on 2 L. She is speaking to you but has a right-sided facial droop and a right-sided motor deficit. Which of the following is the first step in this patient's management? A CT angiogram of head B ECG C Glucose level D Lactate level E MRI of head

The correct answer is E. Referral to a gynecologist for endometrial biopsy Explanation: Endometrial cancer must be considered in any woman with abnormal vaginal bleeding who is older than 35, or any woman younger than 35 with risk factors for endometrial cancer. Referral for endometrial biopsy is warranted. Abdominal CT scan is not indicated since she has no abdominal pain. A routine physical exam would not be appropriate follow-up given the risk for cancer. The patient does not need progesterone since she has minimal bleeding and is hemodynamically stable. 78% of users answered correctly.

A 64-year-old female presents to the ED with vaginal bleeding for 4 days. She denies abdominal pain and describes the bleeding as light. Her vital signs are normal and exam confirms trace blood in the vaginal vault. Her hematocrit is stable and an ultrasound does not show a cause for her symptoms. Which of the following is the most appropriate next step in management? A Abdominal CT scan B Follow up with her primary care doctor for her routine physical exam C No further care is warranted D Progesterone 10 mg/day for 10 days E Referral to a gynecologist for endometrial biopsy

The correct answer is B. Infuse 250-500 mL of normal saline Explanation: This patient is in cardiogenic shock due to an RV infarction complicating her acute inferior STEMI. This acutely ill patient needs an emergency bedside transthoracic echocardiogram to assess cardiac contractility and to rule out any mechanical cause of shock such as acute pulmonary embolus or pericardial effusion. If she can be stabilized, transesophageal echocardiography would be appropriate. Since the patient does not have signs of pulmonary congestion, a judicious trial of crystalloid is indicated. Patients with RV infarcts may require several boluses of IV fluid. Early revascularization, either by PCI or coronary artery bypass graft (CABG), has been shown to increase survival of cardiogenic shock compared with the use of an intra-aortic balloon pump (IABP) along with thrombolytics or with thrombolytics alone. Mortality remains high (∼50%) despite appropriate treatment. Beta-blockers should be avoided in cardiac shock because of their negative inotropy and chronotropy. Pure α1-adrenergic receptor agonists such as phenylephrine should be avoided since they increase cardiac afterload. If crystalloid therapy does not improve perfusion, vasopressors or inotropes should be considered. Dobutamine is indicated if the patient is not profoundly hypotensive. However, dobutamine should be avoided if the systolic BP remains below 90 mm Hg, due to its vasodilator potential. Dopamine is the preferred single agent if the systolic BP is below 70 mm Hg. Dopamine in combination with dobutamine can be more effective than either agent alone but pharmacologic therapy is only a temporizing measure until early revascularization is arranged. 75% of users answered correctly.

A 64-year-old woman arrives by ambulance with crushing substernal chest pain for the past 40 minutes. She received aspirin and a large bore IV en route. Several tubes of blood are available. Her past medical history includes adult onset diabetes mellitus and hypertension, controlled by metformin and lisinopril, respectively. Vital signs include blood pressure (BP) 90/74, heart rate (HR) 115, respiratory rate (RR) 28, temperature (T) 37°C, room air SaO2 96%. The patient appears ashen and diaphoretic, with cool, mottled extremities. She is awake but answers questions slowly. Jugular venous distention (JVD) is present. Heart sounds are regular with an S4 gallop but without murmurs or rubs. Lungs sounds are equal bilaterally and clear to auscultation. Her abdomen is soft, nontender, without palpable masses. Femoral pulses are equal. Extremities are cool, but without edema. Peripheral pulses are faint but equal. Despite her slow responses, the patient follows commands and moves all four extremities. The initial ECG showed sinus tachycardia, with ST-segment elevation in leads II, III, and aVF. The second ECG is shown here (Figure 2-1). While arrangements for echocardiography and early revascularization are made with the cardiologist, which of the following is the next MOST appropriate action? A Administer propranolol B Infuse 250-500 mL of normal saline C Initiate thrombolytic therapy D Start a dobutamine drip E Start a phenylephrine drip

The correct answer is E. Rescue PCI Explanation: Rescue PCI is indicated in patients in the following groups after thrombolysis: patients who are in cardiogenic shock and younger than 75 years, patients with severe heart failure or pulmonary edema, patients with hemodynamically compromising ventricular arrhythmias, patients with a large area of myocardium at risk, and for whom fibrinolytic therapy has failed. There is no role for repeat thrombolytic administration in this type of patient. A balloon pump might help temporarily, but rescue PCI is the procedure of choice. Nitroglycerin is a poor choice as the patient's BP is too low. Dopamine infusion will increase myocardial oxygen demand. 75% of users answered correctly.

A 65-year-old male presents with 2 hours of chest pain. He has ST elevations across his precordial leads and is diagnosed with an AMI. He is given thrombolytics in the ED. One hour after administration, he is still having chest pain and his BP is 80/60. The next best course of action would be A Dopamine infusion B Nitroglycerin drip C Placement of an aortic balloon pump D Repeat thrombolytic administration E Rescue PCI

The correct answer is D. Order an arterial duplex and ankle-brachial indices of the leg. Explanation: In a diabetic patient with a history of cardiovascular disease presenting with a leg ulcer, with recent hair loss, increased pain, and shiny skin, vascular insufficiency must be considered. Measurement of ankle-brachial indices as well as performing an arterial duplex are of utmost importance in this case, as severe vascular insufficiency can require emergent intervention by a vascular surgeon. This takes precedence over determining if there is a coexistent deep vein thrombosis, local wound care, or arranging for outpatient wound management, although all of these interventions may be necessary for this patient eventually. Wound cultures of leg ulcers have been shown to be of questionable value and generally are not helpful unless performed in the operating room during wound debridement. 73% of users answered correctly.

A 65-year-old male with a history of diabetes and cardiac disease presents complaining of a painful ulcer on his foot which is shown in Figure 3-15. He reports pain in the distal leg when supine and a new appearance of shiny skin with hair loss. What is the MOST important next step in his evaluation and management? A Apply a wet-to-dry dressing with a topical antimicrobial and start oral antibiotics. B Arrange for outpatient wound management and regular checkups. C Obtain a wound culture to determine the best antibiotic management. D Order an arterial duplex and ankle-brachial indices of the leg. E Order a venous duplex ultrasound.

The correct answer is B. Endotracheal intubation Explanation: This patient requires immediate endotracheal intubation because he now has impaired consciousness and severe respiratory distress and has failed a trial of CPAP, a form of noninvasive ventilation. Readjusting his mask or increasing the pressure would be ineffective since he is now hypoxic and has a depressed sensorium. Although treatment with nitroglycerin followed by furosemide is indicated in this patient with probable pulmonary edema, control of airway takes precedence. Bioimpedance monitoring, which can noninvasively provide accurate information about cardiac output and thoracic water content, is a useful bedside adjunct for diagnosing and managing suspected heart failure in the ED and could be implemented after the airway is secured. 74% of users answered correctly.

A 65-year-old man with a history of hypertension is brought in by paramedics with 3 hours of dyspnea without chest pain. Paramedics state he became more fatigued and obtunded on the way into the ED. Vital signs are BP 172/108, HR 114, RR 30, T 98.6, O2 saturation 88% on continuous positive airway pressure (CPAP). The patient is unresponsive to voice, with clammy skin. Physical exam findings include JVD, diffuse rales, an S3 gallop, and mild pitting edema. What is the next most appropriate treatment for this patient? A Begin bioimpedance monitoring B Endotracheal intubation C Furosemide IV D Increase CPAP pressure from 5 to 10 cm of water and readjust mask E Start nitroglycerin IV drip

The correct answer is D. Give epinephrine 0.3 mg subcutaneously, diphenhydramine 50 mg, and methylprednisolone 125 mg intravenously. Perform nasopharyngoscopy; if there is no laryngeal edema, the patient can be discharged after observing for several hours if clinically improved. Explanation: Angioedema can occur as an adverse effect to angiotensin-converting enzyme inhibitor therapy, usually within the first week of beginning therapy. Other etiologies are IgE-mediated allergic reaction, congenital loss of C1 esterase inhibitor (presents at an earlier age), and idiopathic. Subcutaneous epinephrine and racemic epinephrine nebulizer are both appropriate. Nasopharyngoscopy is indicated and admission is indicated if there is any laryngeal edema or the patient does not improve clinically. 37% of users answered correctly.

A 65-year-old woman recently started on lisinopril presents with 2 hours of swelling of her lips and tongue. She has no stridor or wheezing and her sublingual space is normal. What is the best course of action? A Give diphenhydramine 50 mg and methylprednisolone 125 mg intravenously. Admit to the intensive care unit. B Give epinephrine 0.3 mg subcutaneously, diphenhydramine 50 mg, and methylprednisolone 125 mg intravenously. Avoid nasopharyngoscopy; the patient can be discharged after several hours of observation if clinically improved. C Give epinephrine 0.3 mg subcutaneously, diphenhydramine 50 mg, and methylprednisolone 125 mg intravenously. Perform nasopharyngoscopy; if there is laryngeal edema, perform a cricothyroidotomy and admit to the intensive care unit. D Give epinephrine 0.3 mg subcutaneously, diphenhydramine 50 mg, and methylprednisolone 125 mg intravenously. Perform nasopharyngoscopy; if there is no laryngeal edema, the patient can be discharged after observing for several hours if clinically improved. E Give racemic epinephrine nebulizer, diphenhydramine 50 mg, and methylprednisolone 125 mg intravenously. Admit to the intensive care unit.

The correct answer is C. Malignancy Explanation: The most common causes of hypercalcemia in the ED are malignancy, particularly lung, breast, kidney, myeloma, and leukemia. Primary hyperparathyroidism is also a frequent cause of elevated calcium levels, but is not as common in the ED. Drugs such as lithium and thiazides can also raise calcium levels. Granulomatous diseases are also known to increase serum calcium levels. Related Topics:hypercalcemia 65% of users answered correctly.

A 66-year-old female with no past medical history presents to the emergency department with increasing lethargy. The family states that she is normally healthy, but has been losing weight (unintentionally) over the past few months. The last week, she has been complaining of abdominal pain, frequent urination, and pain in her arms and legs. She has had no cough, headache, fevers, or chills. On exam, the patient is arousable to stimulus but is confused, is afebrile, and has normal vital signs. She is moving all extremities and her exam is unremarkable. Her ECG shows ST segment depressions, widened T waves, and a shortened ST and QT interval. Her glucose on finger stick is 98. Her CBC is within normal limits. The laboratory notifies you because her calcium level is 14.2 mg/dL, magnesium level is 1.1 mEq/L, and phosphate level is 1.9 mEq/L. Which of the following is the most likely explanation for this patient's hypercalcemia? A Hyperparathyroidism B Lithium toxicity C Malignancy D Paget's disease E Sarcoidosis

The correct answer is D. Normal saline Explanation: Treatment of hypercalcemia should begin in the ED if patients are symptomatic or if the calcium level is greater than 14 mg/dL. The first component of treatment is for volume repletion with NS. Once volume repletion has been accomplished, furosemide may be added to increase the renal calcium excretion. To decrease absorption of calcium from the bones, calcitonin may be given. Hydrocortisone has been shown to be useful in patients with hypercalcemia caused by sarcoidosis, vitamin D toxicity, multiple myeloma, leukemia, or breast cancer. IV phosphates are no longer used in patients with elevated calcium levels. 64% of users answered correctly.

A 66-year-old female with no past medical history presents to the emergency department with increasing lethargy. The family states that she is normally healthy, but has been losing weight (unintentionally) over the past few months. The last week, she has been complaining of abdominal pain, frequent urination, and pain in her arms and legs. She has had no cough, headache, fevers, or chills. On exam, the patient is arousable to stimulus but is confused, is afebrile, and has normal vital signs. She is moving all extremities and her exam is unremarkable. Her ECG shows ST segment depressions, widened T waves, and a shortened ST and QT interval. Her glucose on finger stick is 98. Her CBC is within normal limits. The laboratory notifies you because her calcium level is 14.2 mg/dL, magnesium level is 1.1 mEq/L, and phosphate level is 1.9 mEq/L. Which of the following is the most appropriate medication for this patient? A Calcitonin B Hydrocortisone C Magnesium D Normal saline E Phosphate

The correct answer is D. Patients with HIT are prone to thromboembolic events. Explanation: Despite the low platelet count associated with HIT, patients are more prone to clotting as opposed to bleeding. HIT occurs in 0.5-5% of patients treated with heparin products. The diagnosis of HIT should only be considered in patients exposed to heparin products and in whom the platelet count has dropped at least 50%. All heparin products, including Lovenox®, should be stopped in patients suspected of having HIT. Definitive laboratory testing is challenging. Test with high specificity are complex and only done in a few centers. Widely available immunologic assays have a lower specificity. 49% of users answered correctly.

A 66-year-old man presents to the emergency department complaining of unilateral leg swelling and pain. On review of his hospital records, you discover that the patient was treated with prophylactic subcutaneous heparin during his recent stay for pneumonia. His vital signs are normal, and the physical examination is only notable for a swelling and tenderness of the left lower leg. Laboratory studies reveal a platelet count of 43,000/mm3. At discharge 2 weeks ago, the platelet count was 209,000/mm3. You suspect that the patient may have heparin-induced thrombocytopenia (HIT). Which of the following is TRUE? A Clinicians should suspect HIT when the platelet count has dropped 25% after exposure to heparin products. B Definitive laboratory testing for HIT is easy. C HIT occurs in 10% of patients treated with heparin products. D Patients with HIT are prone to thromboembolic events. E Treatment with low-molecular-weight heparin should be continued in patients suspected of having HIT.

The correct answer is A. Acetazolamide Explanation: The proper management of acute angle glaucoma is intravenous analgesics, antiemetics, mannitol and acetazolamide, topical beta-blockers, and steroids to lower IOP, followed by a miotic. However, in this patient, acetazolamide is contraindicated with her allergy to sulfonamides. Miotics (pupillary constrictors) not mydriatics (pupillary dilators) are effective when the IOP has been brought under control by the other measures and pull the iris away from the cornea to avoid recurrence. There is no role of antibiotics. Emergent ophthalmic consultation is needed for definitive treatment. 50% of users answered correctly.

A 66-year-old woman with a history of hypertension and allergy to "sulfa" complains of acute onset of left eye pain, headache, and vomiting. She has photophobia. Her eye is pictured below (Figure 6-1). Her intraocular pressure (IOP) is 60 mm Hg. What medication is contraindicated? A Acetazolamide B Ciprofloxacin C Morphine sulfate D Pilocarpine E Timolol

The correct answer is D. Vitamin B12 Explanation: Vitamin B12 deficiency causes pernicious anemia. Vitamin B12 takes several forms: cyanocobalamin, hydroxocobalamin, aquocobalamin, nitritocobalamin, coenzyme B12, and methylcobalamin. Deficiencies are usually a result of absorption problems rather than dietary deficiencies. It is a potent molecule and thus small doses are needed to correct the deficiency. 78% of users answered correctly.

A 67-year-old female presents with bilateral hand and feet paresthesias, decreased proprioception, anemia, and an elevated lactate dehydrogenase level. What vitamin deficiency is causing her symptoms? A Vitamin A B Vitamin B1 C Vitamin B2 D Vitamin B12 E Vitamin K

The correct answer is E. Infusion of packed red blood cells (PRBCs) Explanation: Although all of these interventions may be appropriate at some point in the care of this patient, infusion of PRBCs is the first best step. The patient described in this scenario has a hemodynamically unstable gastrointestinal bleed and requires prompt transfusion of blood to address this and his acute anemia. 84% of users answered correctly.

A 67-year-old man presents to the emergency department complaining of weakness. On examination, his blood pressure is 88/46 mm Hg and his heart rate is 89 beats/min. He is pale, and his rectal examination reveals melena. Laboratory studies are remarkable for a hemoglobin of 5.5g/dL. What is the most appropriate initial step in the management of this patient? A Infusion of an inotrope B Infusion of a proton pump inhibitor C Infusion of erythropoietin D Infusion of iron E Infusion of packed red blood cells (PRBCs)

The correct answer is C. Direct pressure, holding warfarin and oral vitamin K Explanation: This patient presents with a greatly elevated INR. Despite this, the patient presents with only minor bleeding, and therefore, he does not require factor replacement or intravenous vitamin K. As is always the case, direct pressure to the site of bleeding is necessitated. In addition, the offending agent, in this case warfarin, should be held for at least one or two doses. And it is also recommended that warfarin-treated patients with an INR greater than 9 without significant bleeding receive oral vitamin K therapy. 60% of users answered correctly.

A 67-year-old man with a history of atrial fibrillation for which he takes warfarin presents to the emergency department with a small bleeding abrasion after shaving. His vital signs are normal. International normalized ratio (INR) is 10. Which of the following represents proper management of this patient? A Direct pressure alone B Direct pressure and holding warfarin C Direct pressure, holding warfarin and oral vitamin K D Direct pressure, holding warfarin, oral vitamin K and FFP E Direct pressure, holding warfarin, intravenous vitamin K and FFP

The correct answer is C. Start quinidine and doxycycline intravenously and contact the CDC for possible artesunate therapy. Explanation: Suspicion for malaria must be high in a febrile patient who has recently traveled to a malarial zone, even if chemoprophylaxis was used. Chloroquine is inappropriate chemoprophylaxis for travel to Africa. The manifestation of P. falciparum is often more severe and the progression of the disease is much more rapid in a nonimmune host. The early disease process can mirror an influenza-type illness with fever, malaise, myalgia, and headache. The classic cyclical pattern of fever is often absent in P. falciparum infection. As the disease progresses, it may manifest in any organ system. Cerebral malaria is marked by coma, seizures, or focal neurologic signs. Jaundice develops through hemolysis, hepatocellular injury, and cholestasis. Respiratory distress, renal failure, lactic acidosis, and disseminated intravascular coagulation can also all develop. The diagnosis of P. falciparum is made by observing small ring forms with double-chromatin knobs within the erythrocyte on thin and thick blood smears. Absence in one sample does not exclude the disease and should not preclude immediate treatment when suspicion is high. Smears should be continued twice daily for 2-3 days. Ideal treatment for P. falciparum is with an artemesin-based derivative. The only intravenous derivative available in the United States is artesunate, which is only available through the CDC. Therapy should begin immediately with quinidine and doxycycline. The CDC should be contacted for artesunate therapy for severe disease or if antimalarial agents are not available at one's hospital. HIV infection can lead to more severe manifestation of malaria, but immediate testing is not necessary. This patient should receive a lumbar puncture after a CT scan to rule out bacterial meningitis and viral encephalitis, but it is not the critical step in the management of this patient. Chemoprophylaxis is typically given for exposure to respiratory droplets from a patient with infection due to Neisseria meningitidis or Haemophilus influenzae. 59% of users answered correctly.

A 68-year-old female is brought in by EMS for fever for 4 days. She was found altered by her husband today who called 911. You learn that she just returned from a trip to Africa, and went on a safari in Tanzania. She had used chloroquine for prophylaxis. She was seen in the ED 2 days earlier and diagnosed with a viral syndrome. She is hypoglycemic but her mental status does not improve with therapy. You intubate her due to altered mental status and respiratory distress. She is pale and jaundiced. She does not have a rash. In addition to treating for possible bacterial meningitis, what else is crucial to her care? A Await results of a single thin and thick smear before initiating additional therapy. B Ensure health care workers treating the patient receive prophylaxis for meningococcemia. C Start quinidine and doxycycline intravenously and contact the CDC for possible artesunate therapy. D Send the cerebrospinal fluid for encephalitis panel. E Test for HIV.

The correct answer is B. Carotid sinus hypersensitivity Explanation: The patient's prior history of syncope during head turning and the current scenario suggest carotid sinus hypersensitivity syndrome. Direct pressure on baroreceptors in the carotid body may trigger an abnormal vagal response of bradycardia and asystole or a drop in BP without a decrease in heart rate. Both reactions may also occur simultaneously. This patient's sitting position at the time of syncope precludes a diagnosis of orthostatic hypotension. Neither micturition nor postprandial syncope are suggested by this scenario. Aortic stenosis should always be considered in the differential diagnosis of syncope in an elderly patient, but physical exam findings such as delayed carotid pulse and a harsh systolic murmur would be evident.

A 68-year-old man presents to the ED after an episode of syncope at home witnessed by his grandson. The patient had allowed his grandson, who was practicing for a first-aid certificate, attempt to locate the "pulse in his neck." The grandson states the patient slumped over after about 5 seconds of neck pressure and was unresponsive. The patient woke up spontaneously after his grandson placed him supine on the floor and 911 was called. The patient has a history of hypertension and takes enalapril. He reports having a similar fainting episode 6 months ago while trimming his sideburns. He denies headache, chest pain, shortness of breath, abdominal pain, or weakness. Vital signs are BP 138/90, HR 70, RR 18, T 98.6, room air saturation 96%. The physical examination, ECG, and bedside transthoracic echocardiogram are unremarkable. Which of the following is the MOST likely etiology of this presentation? A Aortic stenosis B Carotid sinus hypersensitivity C Orthostatic hypotension D Postprandial hypotension E Micturition syncope

The correct answer is C. Homoatropine drops, erythromycin ointment, pain medication, and ophthalmology follow-up in 24 hours Explanation: Cycloplegics help considerably decrease the pain from corneal abrasion, though adjunct oral pain medications are often needed. Homatropine is recommended rather than atropine, as the effects of atropine can last up to 2 weeks. Both tobramycin and erythromycin are adequate topical antimicrobial agents, but abrasions from organic sources have potential for fungal infections and should not be patched. There is no role for steroids in the treatment of corneal abrasion. 45% of users answered correctly.

A 7-year-old girl scraped her cornea with an apple stem. Her visual acuity is 20/20. She has moderate photophobia and is rubbing her eye and crying. On slit-lamp exam, she has a 6-mm superficial linear abrasion of her cornea in the temporal inferior quadrant. Seidel test is negative. Anterior chamber is quiet. What is the most appropriate treatment? A Erythromycin ointment, an eye patch, pain medication, and ophthalmology follow-up in 24 hours B Homoatropine drops, tobramycin drops, an eye patch, pain medication, and ophthalmology follow-up in 24 hours C Homoatropine drops, erythromycin ointment, pain medication, and ophthalmology follow-up in 24 hours D Tobramycin eye drops, topical steroids, pain medication, and ophthalmology follow-up in 24 hours E Tobramycin drops, atropine drops, pain meds, and ophthalmology follow-up in 24 hours

The correct answer is B. Enterococcus spp. Explanation: Enterococcus endocarditis is associated with manipulation of the lower gastrointestinal or genitourinary tract, and is increased among diabetics. It has a propensity for infecting abnormal left-sided valves. Underlying senile aortic stenosis develops with age and can be a risk factor for endocarditis during bacteremia. 37% of users answered correctly.

A 74-year-old diabetic patient is brought in by ambulance because of intermittent fever and malaise for the previous 2 weeks. Upon further questioning, you learn he underwent a transurethral resection of the prostate 1 month ago. He has a diastolic decrescendo murmur on exam. You suspect bacterial endocarditis. Which pathogen is classically associated with this history? A Cardiobacterium spp. B Enterococcus spp. C Escherichia coli D S. aureus E S. viridans

The correct answer is E. Surgical consult Explanation: The most likely diagnosis is mesenteric ischemia secondary to low arterial flow or thromboembolism given the history of congestive heart failure, clinical findings consistent with atrial fibrillation, and pain out of proportion to exam. Mesenteric ischemia is a surgical emergency that will often lead to bowel necrosis and overwhelming sepsis if left untreated. Despite aggressive treatment, prognosis is poor with a survival of 50% if diagnosed within 24 hours. Immediate surgical consultation is required as soon as the diagnosis is suspected to increase the patient's chance of survival. CT scan of the abdomen should be obtained after surgical consultation, but should not delay taking the patient to the operating room. Mesenteric ischemia is a more likely diagnosis than a GI bleed. Therefore, GI consultation for endoscopy is not appropriate prior to surgical consultation. Every attempt should be made to make the diagnosis in the ED prior to admission. A delay in diagnosis by admitting the patient to a nonsurgical service will lead an adverse outcome. Right upper quadrant ultrasound looking for cholecystitis may be appropriate, but should not delay surgical consultation for possible mesenteric ischemia. 57% of users answered correctly.

A 74-year-old woman with a history of congestive heart failure presents with diffuse abdominal pain that started 2 hours ago. She describes the pain as severe and she is in significant distress secondary to pain. She has a temperature of 100.5, a heart rate of 110, and a blood pressure of 95/60 mm Hg. Her cardiovascular exam is remarkable for an irregular heart beat. She has a distended abdomen with hypoactive bowel sounds and diffuse tenderness to palpation on abdominal exam. There is no focal tenderness, rebound, or guarding. On rectal examination, there is profuse soft, dark brown stool that is guaiac positive. Lab results are remarkable for WBC of 16 and a lactic acid of 4.0. After fluid resuscitation and antibiotics, what is the MOST appropriate next step in the management of this patient? A CT scan of the abdomen B GI consult for endoscopy C Medical admission for observation D Right upper quadrant ultrasound E Surgical consult

The correct answer is E. T4 plus T3 Explanation: After supportive care has been initiated, the next steps involve replacing thyroid hormones and identifying and treating the precipitating factors. In this case, the patient was noncompliant with medications. It is important not to wait for lab results before initiating treatment. IV T4 should be given in combination with T3 because conversion of T4 to T3 in myxedema coma is often reduced. T4 alone has a slow and steady onset of action. T3 alone has a more rapid onset but levels fluctuate rapidly. Causes of hypothyroidism include autoimmune (Hashimoto's), ablation, thyroiditis, infiltrative disease, congenital, panhypopituitarism, neoplasms, drugs such as amiodarone, lithium, or iodine, and idiopathic. 52% of users answered correctly.

A 75-year-old female is brought to the ED by her family. Her family states that over the last few months, she has been complaining of generalized weakness and fatigue. The family became concerned because she had not wanted to get out of bed the last 2 days despite the fact that she is normally very active. On exam, the patient answers questions slowly, stating that she stopped taking her thyroid medicine and her BP medicine several months ago. Her vital signs are BP 92/61, HR 57, RR 14, 100% RA, T 95.9. Her finger stick glucose is 99. The patient receives warm blankets, IV fluids, and oxygen. She also receives a dose of steroids. Which of the following is the most appropriate next step in the management of this patient? A Atropine B D5NS C Inotropes D T4 E T4 plus T3

The correct answer is A. She has acute central retinal artery occlusion and she should be treated with acetazolamide, topical beta-blockers, and ocular massage. Emergent ophthalmologic consultation is recommended and tPA infusion considered. Explanation: Central retinal artery occlusion is commonly caused by an embolus originating from a thrombus in the heart as occurs in atrial fibrillation or a cholesterol plaque from an arterial source. There is no good evidence-based treatment for acute retinal artery occlusion, but tPA can be considered if administered within 20 hours of symptoms. Central vein thrombosis presents with a "blood and thunder" retina. A vitreal hemorrhage would usually obscure visualization of the retina or appear as a pre-retinal bleed, not a cherry red macula. Optic neuritis presents with decreased color perception before visual loss, and a normal fundus with normal or edematous optic disk. Occipital strokes manifest as hemianopsia not monocular visual loss. 82% of users answered correctly.

A 75-year-old woman with new onset atrial fibrillation presents with acute visual loss in the right eye for 20 minutes. She has a pale retina with a cherry red macula. What is the appropriate diagnosis and corresponding treatment? A She has acute central retinal artery occlusion and she should be treated with acetazolamide, topical beta-blockers, and ocular massage. Emergent ophthalmologic consultation is recommended and tPA infusion considered. B She has acute central retinal vein thrombosis and she should be treated with aspirin and emergent ophthalmologic consultation. C She has an acute occipital stroke and should receive tPA infusion. D She has acute optic neuritis and should be treated with intravenous steroids and immediate ophthalmologic consultation is recommended. E She has a vitreal hemorrhage and needs to be upright, and should avoid aspirin and nonsteroidal anti-inflammatory drugs and follow up with the ophthalmologist in 24 hours.

The correct answer is C. Administer unfractionated heparin at 80 units/kg bolus followed by an 18 units/kg/hour infusion Explanation: The clinical evaluation is the EP's most important diagnostic tool for assessing potential occlusive arterial disease. Clinically, thrombus has most likely subacutely caused limb-threatening ischemia in a patient with underlying peripheral vascular disease. Pain can be the sole symptom of ischemia in a patient with chronic peripheral vascular disease, but sometimes, total occlusion in a severely diseased artery is silent. An ankle-brachial index <0.25 suggests possibly limb-threatening ischemia. Values between 0.41 and .90 suggest mild-to-moderate peripheral arterial disease. Normal values are 0.91-1.3. See Figure 64-1. Once acute limb ischemia is suspected, the EP should obtain a vascular surgery consultation, before ordering confirmatory imaging. Current practice calls for immediate administration of IV unfractionated heparin: an 80 units/kg followed by an infusion of 18 units/kg/hour. Conservative medical therapy with heparin, without surgery, is generally done in cases such as this one with thrombotic occlusion in the setting of advance atherosclerosis. Aspirin can also be given. Patients with suspected occlusive arterial disease are at risk for reperfusion injury, which is characterized by myoglobinemia, renal failure, and peripheral muscle infarction. About one-third of deaths from occlusive arterial disease are secondary to the metabolic sequelae of revascularization. Baseline electrolytes, myoglobin, and creatine kinase should be ordered, as well as a coagulation panel. If an embolic source, rather than thrombotic source, is suspected, an echocardiogram can rule out a mural thrombus. Bedside studies are usually more available than transthoracic echocardiograms. An ECG can evaluate the presence of atrial fibrillation or ischemia. Serial cardiac biomarkers are indicated to establish acute coronary syndrome. The EP can order a duplex ultrasound to determine complete or incomplete obstruction in the common femoral, superficial femoral, and popliteal vessels, or in bypass grafts. An arteriogram is done to confirm diagnosis and define vascular anatomy and perfusion. Arteriosclerosis is usually evident in patients with a thrombus. An embolus would show an abrupt cutoff of blood flow in a disease-free or minimally diseased vessel. CTA or MRA of the extremity could also be ordered. If an aortic dissection, or aneurysm in the aortoiliac or femoral vessels, is suspected, an aortogram, CT, or MR can be ordered. Definitive treatment of an arterial clot requires a vascular surgeon. Options include percutaneous mechanical thrombectomy and revascularization with percutaneous transluminal angioplasty (PTA) or standard surgery. Timely embolectomy is indicated for acute embolic occlusions. 57% of users answered correctly.

A 78-year-old man presents with gradual onset of severe pain in his left lower leg over 4 days. His past medical history is peripheral artery disease and intermittent claudication with calf pain, relieved by rest. The patient continues to smoke tobacco. He currently denies chest pain, shortness of breath, or headache. Vital signs include BP 140/85, HR 80, RR 20, T 98.6, room air saturation 97%. Physical exam findings include shiny, hyperpigmented skin and absent hair on both the mid-lower legs. The left lower leg is cool, mottled, and cyanotic, with diminished posterior tibial and dorsalis pedis pulses, and decreased sensation to vibration and proprioception but intact deep sensation. The left ankle-brachial index is 0.20. The right leg is slightly warmer with normal color and faintly palpable pulses and right ankle-brachial index of 0.7. An ECG shows sinus rhythm without ST-segment elevation or depression. An IV has been started and blood is available for laboratory studies. A vascular surgeon has been paged but has not yet responded. What is the next most appropriate action? A Arrange CTA of lower extremities B Arrange transthoracic echocardiogram C Administer unfractionated heparin at 80 units/kg bolus followed by an 18 units/kg/hour infusion D Obtain cardiac markers E Order electrolytes, myoglobin, creatine kinase, and coagulation panel

The correct answer is A. Borrelia burgdorferi Explanation: Lyme disease, caused by B. burgdorferi can infect anyone, but it has a significant bimodal expression from 5 to 9 years and 50 to 54 years of age. The second stage of Lyme disease usually occurs days to 6 months after the primary infection. It involves fever, adenopathy, arthritis, atrioventricular nodal block, malaise, and cranial nerve palsies. The most common cranial nerve affected is the facial nerve (7th), and it can be bilateral. The rash of erythema migrans is not always present, so a careful travel history and antecedent rash is prudent with any child presenting with a peripheral facial nerve (7th) palsy. C. psittaci is the agent that causes psittacosis, an avian to human zoonosis that most commonly presents with fever, malaise, and pulmonary symptoms. P. multocida is implicated in soft tissue infections developing from cat and dog bites. R. rickettsii is the pathogen responsible for RMSF. Y. pestis causes both pneumonic and bubonic plague. 73% of users answered correctly.

A 9-year-old male from Connecticut presents to the ED with a 1-day history of bilateral facial nerve palsy that involves the forehead. Upon further questioning, he recalls a nonpruritic, nonpainful rash 2 months ago for which he did not tell his mother or seek treatment. What is the MOST likely pathogen? A Borrelia burgdorferi B Chlamydophila psittaci C Pasteurella multocida D Rickettsia rickettsii E Yersinia pestis

The correct answer is D. Neuralgia and osteoarthritis are the most common causes of chronic pain in the elderly. Explanation: Opioids may cause debilitating constipation in the elderly. However, when properly titrated, opioids may produce fewer serious side effects than NSAIDs, as the NSAIDs cause higher rates of gastrointestinal bleeding and renal disease in the elderly. Neuralgia and osteoarthritis are the most common causes of chronic pain in the elderly. The elderly and others are frequently undertreated for pain, and although side effects must be considered, increasing the dosage of a narcotic may be indicated. Follow-up with a regular doctor would be important in this case.

A 90-year-old female presents to the emergency department complaining of pain unrelieved by her home medications. When treating her, which of the following is MOST correct? A Constipation is a minor side effect of opioids. B Her regular doctor need not be involved. C Increasing the dosage of a previously used opioid is inappropriate. D Neuralgia and osteoarthritis are the most common causes of chronic pain in the elderly. E NSAIDs will have fewer side effects than opioids in this population.

The correct answer is B. Incarcerated Explanation: A hernia is reducible when the hernia sac itself is soft and easy to replace back through the hernia neck defect. A hernia is incarcerated when it is firm, often painful, and nonreducible by direct manual pressure. Strangulation develops as a consequence of incarceration and implies impairment of blood flow (arterial, venous, or both). A strangulated hernia presents as severe, exquisite pain at the hernia site, often with signs and symptoms of intestinal obstruction, toxic appearance, and, possibly, skin changes overlying the hernia sac. A strangulated hernia is an acute surgical emergency. 82% of users answered correctly.

A hernia that is firm, painful, and nonreducible in a stable patient without signs of toxicity is which type of hernia? A Femoral B Incarcerated C Indirect D Reducible E Strangulated

The correct answer is E. This is the most common cause of preventable acute renal failure in childhood. Explanation: This 7-year-old boy has hemolytic uremic syndrome (HUS)—characterized by microangiopathic hemolytic anemia, acute renal failure/insufficiency, and low platelets. HUS is the most common preventable cause of acute renal failure in children. It occurs in a typical and atypical form. The typical form is usually caused by Escherichia coli O157:H7. HUS typically manifests itself after about 1 week into a case of infectious diarrhea—not immediately after ingestion of the causative organism. As mentioned earlier, there is also an atypical form of HUS—sometimes caused by bone marrow transplantation or administration of chemotherapy drugs or immunosuppressants. Finally, the use of antibiotics in the treatment of the underlying infection is controversial and should be addressed on a case-by-case basis. 49% of users answered correctly.

A mother brings her 7-year-old boy into the emergency department for evaluation of fatigue and easy bruising. The mother also reports that her child has had diarrhea (sometimes bloody) for about 1 week. Her vital signs are normal, except for a mild tachycardia. The physical examination is notable for three medium-sized bruises over her lower extremities. Laboratory data reveals a platelet count of 62,000/mm3, a hematocrit of 20%, and a serum creatinine of 1.2. Which of the following is TRUE regarding her likely diagnosis? A Administration of antibiotics in the treatment of the causative organism is absolutely contraindicated. B Atypical forms of this disease are only caused by infections. C Onset of this disease typically occurs immediately after ingestion of the causative bacterium. D The typical form of this disease is most commonly caused by Salmonella species. E This is the most common cause of preventable acute renal failure in childhood.

The correct answer is A. Airborne Explanation: Varicella, or chickenpox, is a very common illness of childhood (although rates are decreasing with vaccine utilization), and is highly contagious. It is spread through respiratory secretions and disease control is actually through airborne precautions, as the virus is able to remain suspended in air and can be dispersed over a long distance. This requires placing the patient in a monitored negative air pressure room with at least 6 air changes per hour with air discharged outdoors or through a filtration system. Other diseases that mandate airborne precautions include measles and tuberculosis. Droplet precautions are for diseases also spread through the respiratory route, but spread is usually limited to direct contact with nasal or respiratory secretions. This requires personnel to use special fit masks when within 3 feet of the patient when exposed to secretions. A simple mask may be placed on the patient during transport to decrease droplet spread. Notable diseases spread by droplets include respiratory viruses, such as novel H1N1, H. influenzae, Neisseria, and Streptococcus. Universal precautions, also known as standard precautions, include hand washing after patient care, use of gloves, masks and/or gowns when handling fluids or exposed to secretions, and proper disposal of contaminated equipment. 36% of users answered correctly.

A nurse informs you that a mother has brought her child in with the following rash (Figure 10-7). What kind of precautions should be taken to decrease the risk of disease spread? A Airborne B Contact C Droplet D None E Universal

The correct answer is A. Atelectasis Explanation: Fever is a common postoperative complaint. The "five W's": wind (atelectasis or pneumonia), water (urinary tract infection), wound, walking (deep vein thrombosis), and wonder drugs (drug fever or pseudomembranous colitis) are a common way to think about the potential etiologies of postoperative fever. Atelectasis is a common cause of fever during the initial 24 hours. Other early postoperative causes of fever are necrotizing streptococcal and clostridial infections. 61% of users answered correctly.

A nurse notifies you that an inpatient who had a cholecystectomy 14 hours ago has developed a fever of 101. He is not complaining of shortness of breath, chest pain, or abdominal pain. The rest of his vitals are BP 120/80, RR 18, P 74, and oxygen saturation of 98% on room air. His lungs are clear, heart is regular, abdomen is nontender, and surgical scar appears intact. What is the most likely cause of his fever? A Atelectasis B Deep vein thrombosis C Intra-abdominal infection D Pneumonia E Urinary tract infection

The correct answer is C. Dumping syndrome Explanation: A common complication of the Roux-en-Y gastric bypass is dumping syndrome, which can occur either right after the meal (early) or 2-4 hours later (late). Dumping symptoms occur when the pylorus is bypassed or removed. The hyperosmolar chyme contents of the stomach are dumped into the jejunum, resulting in rapid influx of extracellular fluid and an autonomic response. Patients experience nausea, epigastric discomfort, palpitations, colicky abdominal pain, diaphoresis, and, in some cases, dizziness and syncope. Patients with early dumping symptoms experience diarrhea, whereas those with late dumping symptoms, 2-4 hours postprandially, usually do not. The late dumping syndrome is believed to be due to a reactive hypoglycemia. The mainstay of treatment is dietary modification; eating small, dry meals; and separating solids from liquids. In refractory cases, pyloroplasty can be tried. Most patients with dumping syndrome do not require admission. 78% of users answered correctly.

A patient presents complaining of nausea, diarrhea, dizziness, and epigastric discomfort immediately after eating a meal. The symptoms recur after each meal with a near syncopal episode today. She had a gastric bypass procedure 3 months ago. What is the most likely cause of her symptoms? A Anastomotic leak B Cholecystitis C Dumping syndrome D Gastroesophageal reflux disease (GERD) E Enterocutaneous fistula

The correct answer is E. Repeat biomarkers Explanation: It is important NOT to be swayed by response to therapy in patients with chest pain. Antacids provide pain relief in 18-45% of patients with acute coronary syndrome. Lack of pain relief with nitroglycerin should also not be convincing. In one study, 65% of patients with acute coronary syndrome did not respond to nitroglycerin. This type of patient, with an atypical story of chest pain and some risk factors, should at least get a second set of biomarkers. 79% of users answered correctly.

A patient presents with a history of hypertension and epigastric and chest pain for 3 hours with negative biomarkers. His electrocardiogram (ECG) is normal. He responds to antacids and feels better. What is your next appropriate response? A A prescription for the proton pump inhibitor B Admit for cardiac workup C An emergent cardiac catheterization D Nitroglycerin E Repeat biomarkers

The correct answer is A. A "holding dose" of low-molecular-weight heparin and arrange for a confirmatory ultrasound on Monday Explanation: D-dimer test would not be sufficient in this patient as he is a high-risk patient based on the Wells criteria, so a "holding dose" of low-molecular-weight heparin and arrangements for a confirmatory ultrasound on Monday are appropriate. Several reports document the safety of a single dose of low-molecular-weight heparin in patients without contraindications while confirmatory tests are arranged. Admission is not warranted. 51% of users answered correctly.

A patient with a recent immobilization presents with unilateral swelling of his entire right leg, pitting edema, and calf swelling of greater than 3 cm. His vitals are normal and he has no shortness of breath. It is Sunday morning 4 am and you do a compression ultrasound of his right leg. It is nondiagnostic showing neither complete compression nor a clot present. There is no on-call vascular tech this weekend. The next most appropriate step in your workup is A A "holding dose" of low-molecular-weight heparin and arrange for a confirmatory ultrasound on Monday B Admission and IV heparin C D-dimer D No treatment and arrangements for a confirmatory ultrasound on Monday E Thrombolysis

The correct answer is E. Thrombolysis Explanation: The condition is phlegmasia cerulea dolens and was first described in the sixteenth century. It is a condition in which the entire venous system is blocked by VTE and limb ischemia is the result. This is limb threatening and should be considered extremely dangerous and a true emergency. The appropriate treatment for this is thrombolysis and possibly surgical thrombectomy. Heparin or a CT will not help this patient, as rapid clot removal or destruction is needed. 40% of users answered correctly.

A patient with lung cancer presents with an extremely painful swollen, bluish colored left lower leg. What is the next step in management? A CT scan of chest with venous runoff study B D-dimer C IV heparin D Low-molecular-weight heparin E Thrombolysis

The correct answer is B. Myocarditis Explanation: This patient, with a pericardial friction rub, elevated cardiac biomarkers, and radiographic and clinical signs of CHF, has myocarditis, a common cause of dilated cardiomyopathy. Etiologies of myocarditis include viral, bacterial, fungal causes, as well as malignant, drug-induced, and systemic diseases. Echocardiography is nonspecific for myocarditis but can show myocardial depression (global hypokinesis) and wall motion abnormalities in severe cases. Diagnosis of myocarditis may require a nuclear scan, heart biopsy, or MRI. Cardiac enzymes are elevated with myocarditis and should be followed serially. Acute coronary syndrome is unlikely in a young, previously well patient. The pericardium can be inflamed in myopericarditis. Pulmonary embolus belongs to the differential diagnosis for acute dyspnea, but other elements in this presentation suggest myocarditis. Most patients with idiopathic or presumed viral myocarditis respond well to nonsteroidal anti-inflammatory medications and can be treated as outpatients, with echocardiographic follow-up. Patients who require admission may have one or more of the following poor prognostic indicators: temperature >38°C, subacute onset over weeks, immunosuppression, oral anticoagulant use, associated myocarditis, and a large pericardial effusion. Pericardial tamponade can occur with very large effusions in acute pericarditis.

A previously well 25-year-old man presents to the ED with 3 days of dyspnea, and orthopnea, and a recent febrile illness associated with myalgias, dyspnea, and precordial chest pain. Vital signs are BP 124/82, HR 120, RR 26, T 100.6, room air saturation 96%. Physical exam findings include a supple neck, faint bibasilar rales, and a friction rub. His chest x-ray (CXR) is shown here (Figure 2-2). A CBC, serum electrolytes, blood urea nitrogen, and creatinine are normal. The initial troponin is 1 ng/mL and myoglobin is 100 ng/mL. His ECG shows sinus tachycardia with nonspecific ST-T-wave changes. Echocardiography shows a very small pericardial effusion. What is his most likely diagnosis? A Acute coronary syndrome B Myocarditis C Pericarditis D Pneumonia E Pulmonary embolus

The correct answer is D. Guaiac positive stool Explanation: The acronym CHESS summarizes the San Francisco Syncope Rules' five predictors of adverse events, including arrhythmia and mortality: history of CHF, hematocrit less than 30, abnormal ECG, shortness of breath, and systolic BP less than 90. An abnormal ECG would include any non-sinus rhythm, conduction delays, new changes to the QRS, or ST segment not previously seen on prior tracings. A new prolonged QT interval would meet the criteria and would warrant admission, due to risk of torsades des pointes. Patients with a clear cause of syncope, such as vasovagal, hypovolemia, situational (cough or micturition), medication-related events, which improved with treatment, and are stable during an observation period, may be discharged. Unfortunately, the etiology of syncope remains unclear in over half of the patients who presented to the ED. 58% of users answered correctly.

According to the San Francisco Syncope Rules, which of the following are predictors for an adverse event in a patient with syncope? A Age >65 years B Elevated TSH level C Guaiac positive stool D History of congestive heart failure (CHF) E Positive pregnancy test

The correct answer is E. Treatment involves placement of a Word catheter for 4-6 weeks. Explanation: Bartholin glands are located in the labia minora and drain into the posterior vestibule at 4 and 8 o'clock positions. Obstruction of a gland can lead to abscess formation. Bartholin gland abscess should be treated with incision and drainage with a Word catheter left in place for 4-6 weeks to prevent reoccurrence. Intravenous antibiotics and hospital admission are usually not indicated once the abscess had been drained. The infection is usually polymicrobial although has been associated with gonorrhea and chlamydia. 56% of users answered correctly.

An 18-year-old female presents with pain and swelling on her labia. Physical exam findings are remarkable for a tender, fluctuant mass at the 8 o'clock position of her labia minora near the introitus. Which of the following statements is true regarding her diagnosis? A Incision and drainage with packing placed for 2 days is the best treatment option. B Patient should be given intravenous antibiotics and admitted to the hospital. C The culprit glands are normally positioned at 2 and 10 o'clock position. D The infection is usually caused by gonorrhea. E Treatment involves placement of a Word catheter for 4-6 weeks.

The correct answer is D. Surgical consultation Explanation: Surgical consultation is recommended for any patient with a classic history for appendicitis. However, surgeons may request imaging before they will take the patient to the operating room. Related Topics:appendicitis 57% of users answered correctly.

An 18-year-old male presents with abdominal pain that began approximately 8 hours ago. Initially, he experienced nausea with periumbilical pain that is now primarily in the right lower quadrant. He has never been hospitalized or had any surgeries. On exam, his temperature is 100.6°F, and his abdominal exam reveals tenderness at McBurney's point. What should be the next step? A CBC and urinalysis B CT scan of the abdomen/pelvis C Plain films of the abdomen D Surgical consultation E Ultrasound of the abdomen

The correct answer is A. Acetaminophen (Tylenol) 650 mg orally Explanation: Acetaminophen (Tylenol) is effective for mild to moderate pain. No change in dosage is needed for mild hepatic impairment. Elderly patients who are not on opioids regularly are more sensitive to both the analgesic effects and the side effects of these medications. Propoxyphene (Darvon), in particular, has few analgesic effects and high potential for addiction and overdose. In this elderly patient with moderate pain and a problem that will be treated as an outpatient, 10 mg of IV morphine would be an inappropriate choice as well. Transdermal fentanyl has a slow onset as well as other potential contraindications. Adverse effects of nonsteroidal anti-inflammatory drugs (NSAIDs) include platelet dysfunction and thus increase the risk of bleeding with warfarin (Coumadin), so Ibuprofen (Motrin) would not be a good choice in this case either. 61% of users answered correctly.

An 80-year-old female arrives in the emergency department with a chief complaint of moderate ankle pain after twisting her ankle. Radiographs are negative for fracture, and she will be discharged home. The patient has a history of mild hepatic impairment and atrial fibrillation. She is on warfarin (Coumadin) and has a history a meperidine (Demerol) allergy. What is the BEST analgesic choice for this patient? A Acetaminophen (Tylenol) 650 mg orally B Fentanyl (Duragesic patch) 25 μg/hour transdermally C Ibuprofen (Motrin) 600 mg orally D Morphine 10 mg intravenously E Propoxyphene hydrochloride (Darvon) 65 mg orally

The correct answer is D. Imipenem/cilastatin Explanation: Imipenem/cilastatin is indicated as monotherapy for cancer patients with neutropenic fever. Amikacin is indicated only as a second antimicrobial in the treatment of febrile neutropenia. Ceftriaxone is not indicated as monotherapy in the treatment of cancer patients with febrile neutropenia, as it does not cover pseudomonas. Vancomycin may be added to another antimicrobial in the treatment of neutropenic fever in patients at increased risked for gram-positive sepsis (e.g., those with indwelling catheters). Amoxicillin/clavulanate, when combined with ciprofloxacin, can be used to treat cancer patients with neutropenic fever in the outpatient setting. 62% of users answered correctly. UpToDate Neutropenia — The definition of neutropenia may vary from institution to institution - neutropenia is an absolute neutrophil count (ANC) <1500 or 1000 cells/microL - severe neutropenia as an ANC <500 cells/microL or an ANC that is expected to decrease to <500 cells/microL over the next 48 hours - profound neutropenia as an ANC <100 cells/microL

An 81-year-old man with metastatic prostate cancer presents to the emergency department complaining of fever. He tells you that he last received chemotherapy 6 days ago at another facility. He takes no medications other than the chemotherapy. His vital signs are normal with the exception of his temperature that measures 39.1°C. The physical examination is unremarkable, except for chronic vertebral tenderness. He has no indwelling catheters. Laboratory studies reveal an absolute neutrophil count of 400/mm3. His chest radiograph and urine analysis are normal. After obtaining blood and urine cultures, what is the best empiric antibiotic regimen? A Amikacin B Amoxicillin/clavulanate C Ceftriaxone D Imipenem/cilastatin E Vancomycin

The correct answer is C. Evaporative cooling and fluid resuscitation Explanation: The likely diagnosis is heat stroke. CT scan of the brain, toxicological screens, endocrine evaluation, and search for infection may be important in this patient to rule out other causes of hyperthermia. However, the initial goal is rapid cooling and fluid resuscitation with normal saline. The goal temperature should be 39°C to avoid overshoot hypothermia, and is best accomplished by evaporative cooling. Immersion cooling makes patient airway management and cardiac monitoring difficult and is not advised. Cooling blankets work too slowly for a patient with heat stroke. Ice packs to groin, neck, and axilla may be useful adjunctive cooling measures. Acetaminophen and dantrolene have no role in treatment of heat stroke. 71% of users answered correctly.

An 82-year-old man is brought in by ambulance during a summer heat wave unresponsive with a rectal temperature of 41°C and dry skin. He lives alone in a trailer without air conditioning. No medications were found by emergency medical services (EMS) on the scene. Which of the following is the most appropriate initial management? A Cooling to body temperature of 37°C by immersion bath B CT scan of brain, serum and urine toxicological screens, and cooling blanket C Evaporative cooling and fluid resuscitation D Rectal acetaminophen, blood cultures, and broad-spectrum antibiotics E Thyroid function tests and administration of dantrolene sodium

The correct answer is B. Intestinal pseudoobstruction (Ogilvie syndrome) Explanation: Intestinal pseudoobstruction is a clinical disorder with the signs and symptoms, and radiographic appearance of acute, large bowel obstruction with no evidence of distal colonic obstruction. The colon may become massively dilated. If not decompressed, the patient risks perforation, peritonitis, and death. The exact mechanism is unknown, but thought to be secondary to an imbalance in the colonic autonomic innervation. Predisposing factors include advanced age, recent surgery, underlying neurologic disorders, and critical illness. Mechanical colonic obstruction is unlikely given the results of the CT scan. Plain radiographs in mesenteric ischemia are generally normal or nonspecific. Toxic megacolon is a complication of inflammatory or infectious colitis. Patients with toxic megacolon are acutely ill with signs of systemic toxicity. 60% of users answered correctly.

An 82-year-old man with dementia is transported from his nursing home secondary to abdominal distention for the past 24 hours. His vital signs are unremarkable except for a low-grade temperature of 99.5. On abdominal exam, he has hypoactive bowel sounds. His abdomen is distended and diffusely tender. He has soft, brown stool in his rectal vault. Plain abdominal films show a dilated colon from the cecum to the splenic flexure. A subsequent CT scan of the abdomen does not reveal a mechanical cause for the condition. What is the MOST likely diagnosis? A Hirschsprung disease B Intestinal pseudoobstruction (Ogilvie syndrome) C Mechanical colonic obstruction D Mesenteric ischemia E Toxic megacolon

The correct answer is C. Echocardiogram Explanation: This patient is likely having an aortic dissection. Aortic dissections are difficult because of the dynamic nature of their presentations. As the dissection travels down the aorta, it can cut off circulation to the brachiocephalic artery, the carotids, the renal vessels, etc. This patient's hypotension was most likely due to his dissection spreading anterograde and causing a pericardial tamponade. Chest x-ray, CT scans, etc., are less helpful than a bedside cardiac echo. The echo can help in the patient with undifferentiated hypotension. It can be done at the bedside, without the need for transporting the critically ill patient to the radiology suite. 53% of users answered correctly.

An 85-year-old male with a history of chronic hypertension presents with 2 hours of chest pain, which radiates to his back. It is the worst pain he has ever had and feels it ripping down his back. His initial BP in the ED is 180/110. He has a widened mediastinum on his chest x-ray, and his ECG is unchanged. In approximately 45 minutes, you notice that he has become altered and his BP is now 75/50. What is the most appropriate diagnostic modality for this patient? A CT scan B Central venous pressure monitoring C Echocardiogram D Repeat chest x-ray E Repeat ECG

The correct answer is D. Minimal sedation Explanation: Moderate sedation patients generally have their eyes closed and respond slowly to vocal commands. Dissociative sedation is a type of moderate sedation where cortical centers do not receive sensory stimuli. Deep sedation is characterized by a profoundly depressed level of consciousness, with a purposeful motor response elicited only after repeated or painful stimuli. Minimal sedation is characterized by anxiolysis but with normal arousal to verbal stimuli. With general anesthesia, the patient is unarousable.

An anxious 50-year-old female drug user with a 10-cm thigh abscess from skin-popping is given midazolam (Versed) to facilitate incision and drainage. She remains calm but with her eyes open and cooperative and verbal throughout the procedure. What type of sedation is this called? A Deep sedation B Dissociation C General anesthesia D Minimal sedation E Moderate sedation

The correct answer is D. Test the intern for anti-HBs. If inadequate, treat with HBIG and vaccine booster. Explanation: Management for possible hepatitis B after bodily fluid exposure is guided by the hepatitis B status of the source and the response to vaccination in the person exposed. Documentation of antibody to hepatitis B surface antigen (anti-HBs) >10 mIU/mL demonstrates a proper response to the vaccine and signifies immunity. No intervention is necessary if exposed. However, if the exposed is unvaccinated or the response to vaccine unknown, then testing begins with the source. Ability to transmit hepatitis B is demonstrated by the presence of hepatitis B surface antigen (HBsAg). Exposure to an HBsAg positive source without proper vaccination requires both HBIG and the primary vaccination series. Exposure without proper response to the vaccine requires HBIG and vaccination booster or a second dose of HBIG in follow-up. Treatment should be within the first 24 hours. 52% of users answered correctly.

An intern experiences a needlestick after attempting to place a central line in a septic patient. The source patient has liver cancer from chronic hepatitis B. The intern received the vaccination series for hepatitis B, but there are no records of antibody titers. What is the proper course of action? A If the source is positive for hepatitis B core antigen (HBcAg), test the intern for HBsAg. If inadequate, treat with vaccine booster. B No treatment is necessary; the intern has received proper vaccination. C Test the intern for anti-HBs. If adequate, give only hepatitis B immune globulin (HBIG). D Test the intern for anti-HBs. If inadequate, treat with HBIG and vaccine booster. E Treat the intern with HBIG.

The correct answer is E. Ultrasound guidance or a peripheral nerve stimulator should be used if available. Explanation: Femoral nerve blocks are appropriate for hip fractures and are especially useful in the elderly. Femoral nerve blocks are less likely to exacerbate hypotension or alter the patient's mental status if appropriate anesthetic doses are used when compared with parenteral opioid analgesics. The anesthetic should not be injected if resistance is felt, as this indicates the possibility of intraneural injection. Injection should be done at the level of the inguinal crease. In obese patients, placement of a pillow underneath the affected hip can help expose the area. Ten to thirty minutes are required to obtain optimal analgesia. Use of ultrasound or a peripheral nerve stimulator is recommended when available.

An obese elderly female presents with a head injury and a hip fracture. Her blood pressure is low and she is moaning in pain. You decide to perform a femoral nerve block on this patient, which of the following is correct about that procedure? A Inject after resistance is felt. B Injection should be done 3 cm below the inguinal crease. C Pillow should be placed under the contralateral hip. D Relief is almost immediate. E Ultrasound guidance or a peripheral nerve stimulator should be used if available.

The correct answer is A. Aneurysm Explanation: This patient is presenting with a third nerve palsy and pupil dilation. Acute CN III palsy with ipsilateral pupillary dilatation is a posterior circulation aneurysm until proven otherwise. The expanding aneurysm causes compression on the pupillomotor fibers that ride on the outside of the third verve. Emergent blood pressure management, neuroimaging, and neurosurgical consultation are indicated. Although diabetes and hypertension are the most frequent causes of third nerve palsy, they are usually papillary sparing as the mechanism is secondary to insufficiency of the vasa vasorum (penetrating feeding vessels into the nerve), thus causing ischemia to the central fibers first. Carotid dissection would cause a Horner syndrome, which presents as miosis not dilation along with ptosis and anhydrosis and does not affect the third nerve. Werneike encephalopathy affects the abductors or CN VI rather than CN III and is associated with altered mental status. CN palsy of IV is most common after acceleration/deceleration injury and presents with subtle diplopia. 65% of users answered correctly.

An otherwise healthy 45-year-old woman with no significant past medical history comes in complaining of 12 hours of diplopia. On exam she has a dilated right pupil and can only abduct and look up with her right eye when range of motion is tested. What is her likely diagnosis? A Aneurysm B Diabetes mellitus C Carotid dissection D Werneike encephalopathy E Vertebral artery dissection

The correct answer is A. <10-20,000/mm3 Explanation: The risk for spontaneous bleeding, particularly within the brain, is much increased as the platelet count falls below the 10,000-20,000/mm3 range. Clinicians should consider transfusion of platelets, even in the asymptomatic patient, when the platelet level falls below 10,000/mm3. 74% of users answered correctly.

At what platelet count level does the risk for spontaneous bleeding become significant? A <10-20,000/mm3 B 20-30,000/mm3 C 30-40,000/mm3 D 40-50,000/mm3 E 50-60,000/mm3

The correct answer is D. 50-year-old man with syncope and a harsh systolic ejection murmur Explanation: The 55-year-old man with syncope and a harsh systolic ejection murmur should be admitted for suspected aortic stenosis, despite being asymptomatic at rest, since he is at risk for sudden death and heart failure. When assessing a patient with a new cardiac murmur, the EP should always consider the patient's underlying medical condition. Fever, pregnancy, anemia, renal failure with chronic volume overload, thyrotoxicosis, arteriovenous fistula, and sepsis all cause increased cardiac output and systolic murmurs in patients with normal cardiac anatomy. If the new murmur is diastolic, or if symptoms occur at rest, the murmur may be pathologic. Patients with suspected aortic stenosis and syncope, who look well at rest, are the exception to this rule and should be admitted, with cardiology referral and an echocardiographic study. 76% of users answered correctly.

During their ED workups, all of the following patients are found to have murmurs, but each person is asymptomatic at rest and currently stable. Based solely on the information below, which patient MOST warrants admission, cardiology referral, and an echocardiographic study? A 17-year-old boy with fever, pharyngitis, and a systolic ejection murmur B 20-year-old woman, 10 weeks pregnant, with a systolic ejection murmur C 40-year-old woman with fibroids, a hemoglobin of 10, and a systolic ejection murmur D 50-year-old man with syncope and a harsh systolic ejection murmur E 60-year-old man with chronic renal failure, a patent AV shunt, and a systolic ejection murmur

The correct answer is D. 1st week after delivery by cesarean Explanation: The risk peaks in the first week with an incidence of 1 in 500 women. The immobility and increase risk after surgery are likely contributors. Exogenous estrogen, whether for contraception or hormone replacement therapy, increases the risk as well. 66% of users answered correctly.

During which period is a pregnant woman at most risk for VTE? A 1st trimester B 2nd trimester C 3rd trimester D 1st week after delivery by cesarean E 1st week after delivery by normal spontaneous vaginal delivery

The correct answer is D. Postherpetic neuralgia Explanation: In general, tricyclic antidepressants are an effective therapy in patients with neuropathic pain, which includes postherpetic neuralgia; however, antidepressants are not effective for spinal cord injury pain, phantom limb pain, and HIV-related neuropathy. Intranasal calcitonin (but no other therapy studied) was recommended by a meta-analysis for complex regional pain syndrome, type 1.

For which of the following conditions is a tricyclic antidepressant considered first-line therapy? A Complex regional pain syndrome, type I, also known as reflex sympathetic dystrophy B HIV-related neuropathy C Phantom limb pain D Postherpetic neuralgia E Spinal cord injury pain

The correct answer is A. Lack of hand washing Explanation: Hepatitis A is transmitted by the fecal-oral route. Although it is popularly associated with improper food handling or oyster consumption, the most common transmission occurs from asymptomatic children to adults. Hepatitis A infection has an incubation period of 15-50 days, followed by a prodrome of nausea, vomiting, and malaise. About a week into the illness, patients may note dark urine (bilirubinuria). A few days later, they develop clay-colored stools and jaundice. Hepatitis A does not have a chronic component, and death from hepatic failure is rare. 27% of users answered correctly.

How is hepatitis A most commonly transmitted? A Asymptomatic children to adults B Contaminated food C Improper food handling D Lack of hand washing E Oyster consumption

The correct answer is C. 6 Explanation: Troponins leak out of damaged myocardium cells after prolonged ischemia. Thus, it takes 6 hours for cellular death and breakdown for these biomarkers to leak into the bloodstream. Myoglobin elevations may be seen earlier (in the 2-4 hours range). Patients who present soon after symptoms may have NO elevation in biomarkers. 64% of users answered correctly.

How many hours after a cardiac ischemia event would one expect to reliably find a positive troponin? A 2 B 4 C 6 D 8 E 10

The correct answer is D. CT scan Explanation: In the ED, flat and upright abdominal radiographs and upright chest x-ray or a lateral decubitus view can be used to screen for bowel obstruction, severe constipation, or free air. Plain x-rays can also localize the site to large or small bowel. However, if clinical suspicion for obstruction is strong, a CT scan is the diagnostic method of choice in the ED. If intravenous contrast cannot be used because of renal insufficiency or contrast allergy, oral contrast alone may provide sufficient diagnostic information. Oral and intravenous contrast-enhanced CT can delineate partial or complete bowel obstruction, distinguish partial small bowel obstruction from ileus, and differentiate strangulated from simple small bowel obstruction. 59% of users answered correctly.

If you suspect a patient has a small or large bowel obstruction, the diagnostic study of choice is: A Abdominal series (plain films) B Barium enema C Barium swallow D CT scan E MRI

The correct answer is C. Knee and hip Explanation: The immobilization of the knee and hip confers the greatest risk to the patient. The risk increases as follows: elbow, shoulder, ankle, knee, and hip. Bed rest also confers an increased risk after 48-72 hours of immobilization. 92% of users answered correctly.

Immobilization of which of the following contributes the most risk to the formation of a venous thromboembolism (VTE)? A Ankle B Elbow C Knee and hip D Shoulder E Wrist

The correct answer is A. Cancer patients with back pain should undergo spinal imaging. Explanation: Because approximately 80% of patients with malignant spinal cord compression have a prior diagnosis of cancer, back pain in those with a known history of cancer should undergo imaging. Said another way, these patients have spinal column metastases until proven otherwise. Fluid restriction—not demeclocycline—is the mainstay of treatment in nonseizing patients with hyponatremia caused by inappropriate antidiuretic hormone secretion. Although electrical alternans is a classic electrocardiographic finding in patients with malignant pericardial effusion, it is seen rarely. Tumor lysis syndrome is characterized by hyperuricemia, hyperkalemia, hyperphosphatemia, and hypocalcemia. Thromboembolism is the second leading cause of death in cancer patients. 62% of users answered correctly.

In considering emergency complications of cancer, which of the following is TRUE? A Cancer patients with back pain should undergo spinal imaging. B Demeclocycline is the treatment of choice for cancer patients with hyponatremia from inappropriate antidiuretic hormone secretion. C Electrical alternans is a common electrocardiographic finding in patients with malignant pericardial effusion. D Hypokalemia is frequent laboratory abnormality associated with tumor lysis syndrome. E Thromboembolism is the leading cause of death in cancer patients.

The correct answer is B. Cryoprecipitate is the treatment of choice for types II and III vWD. Explanation: Cryoprecipitate is the treatment of choice for patients with types II and III vWD. Patients with type I vWD are treated initially with desmopressin. Patients with vWD often present with mucocutaneous bleeding, including easy bruising. Hemophilia A and B are X-linked diseases, therefore affects boys and men more. Hemophilia A is caused by a deficiency in factor VIII, and hemophilia B is caused by a deficiency of factor IX. Hemophilia A and B can cause prolongation of the activated partial thromboplastin time—a measure of intrinsic pathway function. They do not affect the prothrombin time. 44% of users answered correctly.

In terms of the hemophilia A and B and von Willebrand disease (vWD), which of the following is TRUE? A Bruising is an uncommon manifestation of vWD. B Cryoprecipitate is the treatment of choice for types II and III vWD. C Hemophilia A and B predominantly affects girls and women. D Hemophilia B is caused by a deficiency of factor VIII. E Prothrombin time is elevated in patients with hemophilia A.

The correct answer is E. Unknown or indeterminate Explanation: The longitudinal Framingham study described 822 reports of syncope in the 7814 patients followed for 17 years. The causes of syncope were vasovagal (21%), cardiac (10%), orthostatic (9%), seizure (5%), neurologic (4.1%), and unknown (37%). Other studies also report an unknown cause in about 40% of cases, despite extensive workups. Limited ED evaluations may not determine a specific cause of syncope in 50-60% of cases. The ED focus is to identify patients at increased risk of immediate decompensation or future serious morbidity or sudden death. 60% of users answered correctly.

In the Framingham Heart Study, which of the following was the MOST common etiology of syncope? A Cardiac B Orthostatic C Seizure D Vasovagal E Unknown or indeterminate

The correct answer is E. Viral gastroenteritis Explanation: Acute nausea and vomiting can be a symptom of many disorders. It is important to consider a broad differential when evaluating a patient with acute nausea and vomiting. While all of the answers are known to cause acute nausea and vomiting, viral gastroenteritis is the most common cause in the United States. 80% of users answered correctly.

In the United States, what is the MOST common cause of acute nausea and vomiting? A Central nervous system tumor B Diabetic ketoacidosis C Nonsteroidal anti-inflammatory drugs (NSAIDs) D Pregnancy E Viral gastroenteritis

The correct answer is D. Patients with slowly developing anemia may have no complaints even with a hemoglobin concentration as low as 5g/dL. Explanation: Anemia that develops over a long period of time may present with almost no symptoms. Rapidly developing, severe anemia can present with altered sensorium. Folate and vitamin B12 deficiencies cause macrocytic anemia. The MCHC is a measure of hemoglobin concentration in the average red blood cell. Serum ferritin is the most useful test for the diagnosis of iron deficiency anemia. 81% of users answered correctly.

In the evaluation of a patient with anemia, which of the following is TRUE? A Altered mental status is never attributable to anemia. B Folate deficiency causes microcytic anemia. C Mean corpuscular hemoglobin concentration (MCHC) is a direct measure of bone marrow activity. D Patients with slowly developing anemia may have no complaints even with a hemoglobin concentration as low as 5g/dL. E Serum ferritin is the most useful test in the diagnosis of thalassemia.

The correct answer is A. Diarrhea in less than 8 hours of symptom onset Explanation: There are three stages in the presentation of mushroom poisoning. In the first stage, 6-24 hours after ingestion, patients experience abdominal pain, fever, nausea, vomiting, pronounced diarrhea, tachycardia, hyperglycemia, hypotension, and electrolyte imbalance. From 24 to 48 hours after ingestion, the second stage of poisoning brings reduction in symptoms, even as the liver deteriorates precipitously. Fulminant liver failure marks the third stage, and the patient will experience coagulopathy, hepatic coma, shock, renal failure, extreme electrolyte imbalance, and seizures. The onset of diarrhea at less than 8 hours after ingestion has been demonstrated to be a sensitive marker for fatality (i.e., death or liver transplantation). In addition, all patients with international normalized ratio (INR) ≥6 at 4 days from ingestion have required liver transplantation. Overall, various case series have demonstrated that 10-30% of patients who ingest A. phalloides will develop hepatic failure and need transplantation. The rapid progression of initial symptoms to full hepatic failure makes transplantation difficult. 33% of users answered correctly.

In the setting of liver failure secondary to Amanita phalloides, what has been shown to be the most sensitive clinical marker for fatality? A Diarrhea in less than 8 hours of symptom onset B Electrolyte imbalance C Fever of 104°F D Hypotension E Tachycardia

The correct answer is E. 60-year-old man with chest pain and ST segment elevations on ECG Explanation: Although the benefits of hyperbaric oxygen for carbon monoxide poisoning remain controversial, AMS or coma, focal neurological deficits, pregnancy with COHb >15%, COHb level >25%, and evidence of acute myocardial ischemia are all accepted indications for HBO. Children and elderly may be more susceptible to the toxic effects of CO poisoning, but the indications for HBO are the same as in adults. 40% of users answered correctly.

In which of the following patients is hyperbaric oxygen treatment indicated after a carbon monoxide (CO) exposure? A Asymptomatic 85-year-old woman with history of dementia whose mental status is at baseline according to family B Asymptomatic 30-year-old man with COHb of 17% C Asymptomatic woman who is 36 weeks pregnant with COHb of 5% D 5-year-old girl with a headache and vomiting E 60-year-old man with chest pain and ST segment elevations on ECG

The correct answer is A. Within 1 month . Explanation: The image demonstrates scabies. Once exposed, it typically takes the immune system approximately 30 days to respond with the development of the intensely pruritic eruptions, which are an immune response to the scabies mite and its excrement within the skin. All of the other options are too short of a timeline for adequate development of immune response and subsequent symptoms. Related Topics: scabies 37% of users answered correctly.

Infestation by this organism produces symptoms approximately how long after initial exposure (Figure 3-18)? A Within 1 month B Within 2 weeks C Within 24 hours D Within 3 hours E Within 5 days

The correct answer is C. Jimsonweed Explanation: Jimsonweed is commonly ingested for its hallucinogenic properties. It can cause an anticholinergic toxidrome that is not usually life threatening. Castor beans contain ricin toxin, one of the most potent naturally occurring toxins. Oleander and foxglove both contain cardiac glycosides resembling digitalis and can cause fatal cardiac dysrhythmias. Water hemlock contains cicutoxin that may be rapidly fatal. 36% of users answered correctly.

Ingestion of which of the following plants is NOT associated with life-threatening toxicity? A Castor bean B Foxglove C Jimsonweed D Oleander E Water hemlock

The correct answer is B. High or ultrahigh potency topical corticosteroids Explanation: The figure demonstrates a silver scaly eruption on a salmon-colored base typical of psoriasis. Initial treatment of psoriasis, especially without the intervention of a dermatologist, is high or ultra-high potency topical corticosteroids, including fluocinonide, clobetasol propionate, or betamethasone dipropionate ointment. Tar-containing soaks followed with application of topical corticosteroids are reserved for areas that are difficult to treat, such as the palms and soles, due to the thickness of the skin. Other treatments employed under the care of a dermatologist, often when initial attempts at topical corticosteroids are insufficient, include systemic biologics, such as methotrexate or cyclosporine, acitretin, and psoralen-ultraviolet light A. Related Topics: psoriasis 51% of users answered correctly.

Initial ED treatment for a patient with these skin findings includes which of the following (Figure 3-16)? A Acitretin B High or ultrahigh potency topical corticosteroids C Methotrexate D Psoralen-ultraviolet light A treatments E Tar-containing solutions

The correct answer is B. H2 receptor antagonists—inhibit histamine receptor of parietal cells. Explanation: Histamine receptor antagonists inhibit the H2 receptors of the gastric parietal cells, thereby suppressing acid secretion. Antacids buffer gastric acid, and are mainly used as needed for breakthrough ulcer pain in patients taking concurrent proton pump inhibitor or H2 blockers until healing occurs. Misoprostol is a prostaglandin analog that increases mucous bicarbonate production by increasing mucosal blood flow. Misoprostol may prevent ulcer formation in patients on concurrent NSAID therapy. Proton pump inhibitors decrease acid by blocking its secretion at the proton pump on gastric parietal cells. When compared with H2 blockers, proton pump inhibitors heal ulcers faster. Sucralfate protects ulcer from acid exposure by forming a sticky gel to allow healing. 75% of users answered correctly.

Match the CORRECT drug used to treat peptic ulcer disease and its associated mechanism of action. A Antacids—block secretion of acid at the proton pump. B H2 receptor antagonists—inhibit histamine receptor of parietal cells. C Misoprostol—protects ulcer from acid exposure by forming a sticky gel. D Proton pump inhibitor—buffer gastric acid. E Sucralfate—prostaglandin analog that increases bicarbonate production.

The correct answer is B. Albuterol nebulizer, antibiotics, and immediately referral for bronchoscopy Explanation: This patient shows classic history of foreign body aspiration. There is choking episode followed by asymptomatic period then recurrent unilateral obstructive pneumonia and/or wheezing. The patient's choking and coughing believed to be secondary to laryngeal exposure to the acidic orange juice was actually from aspiration of a seed. Children usually present with a history of choking or dysphagia, but 60% exhibit mild or absent signs and symptoms at initial presentation. Food is the most common aspired foreign body in children. All suspected foreign body aspirations require bronchoscopy. 45% of users answered correctly.

One week prior to presentation, an 18-month-old girl was being fed a half orange by squeezing it into her mouth and she began coughing and gasping. She was admitted to the intensive care unit overnight for trace vocal cord edema that resolved. She returns now with 3 days of cough and fever (101°F) and a focal right-sided wheeze. She has no stridor, her voice and cry are normal. Chest x-ray shows a right lower lobe infiltrate. What is the most appropriate next step? A Albuterol nebulizer and antibiotics to cover aspiration pneumonia and 24 hours follow-up B Albuterol nebulizer, antibiotics, and immediately referral for bronchoscopy C Albuterol nebulizer and a chest CT to rule out pulmonary embolism D Albuterol nebulizer, chest percussion, antibiotics to cover aspiration pneumonia, and admission for observation E Albuterol nebulizer, oral steroids, and antibiotics to cover aspiration pneumonia and 24 hours follow-up

The correct answer is A. Drugs that stimulate ovulation are a risk factor for ovarian torsion. Explanation: Risk factors for developing ovarian torsion include pregnancy, the presence of a large ovarian cyst or mass, and chemical induction of ovulation. Ultrasound is often the first imaging modality to evaluate torsion but should not be relied upon to rule out the diagnosis when clinical suspicion is high. Although patients classically present with severe unilateral pain, atypical presentations of ovarian torsion are common and patients may present with bilateral, mild, or intermittent pain. 44% of users answered correctly.

Regarding ovarian torsion, which of the following statements is correct? A Drugs that stimulate ovulation are a risk factor for ovarian torsion. B If a patient has vague, bilateral lower abdominal pain, ovarian torsion is extremely unlikely. C Ovarian torsion most commonly occurs on the left side. D Pelvic ultrasound is highly reliable for the evaluation of ovarian torsion. E Pregnancy is not a risk factor for ovarian torsion.

The correct answer is B. Labetalol and nifedipine are safe in pregnancy for chronic hypertension. Explanation: During pregnancy, labetalol and nifedipine are safe to use in the management of chronic hypertension. ACE-Is should be avoided in pregnancy because of teratogenic effects and intrauterine growth retardation. Ketosis occurs more rapidly and at lower glucose level in pregnancy compared with nonpregnancy; therefore, any pregnant woman with elevated blood sugar and who is ill appearing should be evaluated for diabetic ketoacidosis. Propylthiouracil is the first-line treatment for hyperthyroidism in pregnancy; methimazole may be used as an alternative if patients develop a pruritic rash. Sulfonamide should not be used in the third trimester because of concern of kernicterus in the infant. 69% of users answered correctly.

Regarding the management of chronic medical illness during pregnancy, which of the following statements is true? A Angiotensin-converting enzyme inhibiters (ACE-Is) are first-line therapy for chronic hypertension in diabetic pregnant women. B Labetalol and nifedipine are safe in pregnancy for chronic hypertension. C Pregnant women are at less risk for diabetic ketoacidosis than nonpregnant women because of a higher basal metabolic rate. D Methimazole is the drug of choice for hyperthyroidism in pregnancy. E Trimethoprim-sulfonamide is the best choice for treatment of urinary tract infection in pregnant women in their third trimester.

The correct answer is A. Methotrexate has a higher failure rate with ectopic pregnancies with fetal cardiac activity. Explanation: Factors associated with a higher failure rate for methotrexate treatment include larger tubal diameter, higher initial serum hCG level, severe abdominal pain, and fetal cardiac activity. The most common side effect of methotrexate is abdominal pain followed by flatulence and stomatitis. Lower abdominal pain lasting up to 12 hours is common 3-7 days after methotrexate treatment and is thought to be secondary to methotrexate-induced tubal abortion or tubal distention due to hematoma formation. Moderate-to-severe pain warrants evaluation with an ultrasound and blood count for concern of ongoing rupture or hemoperitoneum. Methotrexate should not be used in a hemodynamically unstable patient. If the hCG is greater than 5000 mIU/mL, multiple dose methotrexate may be needed, and there is a higher failure rate of the therapy. 42% of users answered correctly.

Regarding the use of methotrexate for ectopic pregnancy, which of the following statements is true? A Methotrexate has a higher failure rate with ectopic pregnancies with fetal cardiac activity. B Patients should be advised to expect moderate-to-severe abdominal pain after methotrexate. C Patients should undergo immediate laparoscopy if there is no resolution of the ectopic pregnancy following the initial dose of methotrexate. D There are no absolute contraindications to methotrexate and should be considered first-line therapy for all patients with ectopic pregnancy. E Vaginal bleeding is the most common adverse effect of methotrexate.

The correct answer is A. 2% Explanation: The Pulmonary Embolism Rule-Out Criteria (PERC) is a well-validated rule to aid the clinician predicts which patients have a very low risk of PE even before any diagnostic testing is done. This allows the clinician, if all criteria are met, to eliminate the diagnosis of PE from the differential diagnosis, thus saving resources and potential complications from unnecessary diagnostic tests. The PERC criteria are used to help the clinician do a bedside assessment to determine if a patient is at "very low risk" for pulmonary embolism and does not warrant additional diagnostic evaluation, including a D-dimer. The PERC criteria includes: - Age < 50 yrs - Pulse < 100 bpm - SaO2 > 94% - No unilateral swelling - No hemoptysis - No recent trauma or surgery - No prior PE or DVT - No hormone use

The Pulmonary Embolism Rule-Out Criteria (PERC) states that if all the criteria are met, then the risk of PE falls to less than what percent? A 2% B 5% C 10% D 15% E 20%

The correct answer is C. The infection can be treated with rifampicin or clotrimazole. Explanation: The picture shows fish tank granuloma caused by Mycobacterium marinum. This infection is distinguished from cat scratch fever and primary tuberculosis by the absence of lymphadenopathy. Treatment may include sulfamethoxazole, trimethoprim, tetracyclines, and rifampicin. It is seen rarely and is associated with frequent exposure to bodies of water or who are fish tank cleaners. The infection requires exposure to abraded or traumatized skin. Related Topics: mycobacterium marinum infection 37% of users answered correctly.

The individual pictured here gives a history of working at a local pet store (Figure 3-6). His job is to maintain the fish tanks. Which of the following is NOT true concerning his infection? A All aquatic environments can support the causative organism of this infection: fresh, salt, and brackish water. B Infection requires exposure to traumatized skin in a contaminated environment. C The infection can be treated with rifampicin or clotrimazole. D There is the development of significant lymphadenopathy with this infection. E Two to three weeks after exposure to this organism, an initial nodule or pustule forms, which then breaks down to form an abscess or ulcer.

The correct answer is D. Selenium sulfide lotion or shampoo Explanation: The slide shows a KOH prep obtained from a patient with pityriasis versicolor. This condition is caused by an overgrowth of yeast called Pityrosporum ovale, otherwise known as M. furfur. It is most common in young adults in the summer months. The lesions are asymptomatic, may be hypo- or hyperpigmented, occurring most commonly on the chest or upper back. Selenium sulfide shampoo may be used daily for a week to treat the lesions. Other treatments may include topical ketoconazole, econazole, miconazole, or clotrimazole. Extensive or refractory pityriasis versicolor can be treated with oral ketoconazole 400 mg single dose, repeated at monthly intervals. Fluconazole 400 mg single dose or itraconazole 200 mg/d/7 days or 400 mg single dose are also effective treatments. Permethrin cream is used to treat scabies lesions. Vaseline may be used to treat psoriatic lesions and may be used in hydrocortisone preparations with Pityriasis rosea. Related Topics: tinea versicolor 43% of users answered correctly.

The most appropriate treatment for this condition is (Figure 3-2). A Ketoconazole shampoo B Oral antihistamines and topical steroids C Permethrin cream D Selenium sulfide lotion or shampoo E Vaseline

The correct answer is C. 35% Explanation: A variety of benefits have been shown when nitroglycerin is used in patients with AMI. Several trials have demonstrated a reduced infarct size, improved regional function, and decreased cardiovascular complications. Mortality rate appears to be reduced to 35% with the use of nitrates. IV nitroglycerin was titrated to a mean arterial pressure reduction of 10% in normotensive patients and 30% in hypertensive patients. Nitroglycerin was NOT titrated to symptom resolution. In addition, nitroglycerin should be used VERY cautiously in patients with inferior wall ischemia, as these patients are preload dependent and do not tolerate much of a BP decrease. 27% of users answered correctly.

The use of nitroglycerin in AMI patients is associated with what degree of decrease in mortality rates? A 10% B 25% C 35% D 50% E No decrease

The correct answer is C. Esophageal coin impaction Explanation: Esophageal coin impactions appear as a radio-opaque, circular object on AP films. Conversely, tracheal coin impactions appear as a radio-opaque, circular object on lateral films. 79% of users answered correctly.

The x-ray is MOST consistent with which diagnosis (Figure 7-2)? A Aortic knob dilation B Artifact C Esophageal coin impaction D Subcutaneous coin impaction E Tracheal coin impaction

The correct answer is A. Ceftriaxone Explanation: The picture shows an individual with the painful ulceration of a chancroid, caused by Haemophilus ducreyi. Local, painful adenopathy is often present. The diagnosis is made clinically. The disease is sexually transmitted. Treatment may include ceftriaxone, azithromycin, and ciprofloxacin. Other painless ulcerations in the differential diagnosis include HSV that may resemble this disease if the vesicles are already broken. Related Topics: ceftriaxone 42% of users answered correctly.

This patient presents with a painful, penile lesion with associated tender swelling in the groin (Figure 3-5). What medication would be useful in treating the lesion? A Ceftriaxone B Doxycycline C Penicillin D Permethrin cream E Trimethoprim sulfamethoxazole

The correct answer is B. Differential diagnosis includes disseminated fungal infections such as cryptococcosis, histoplasmosis, coccidioidomycosis, and penicilliosis. Explanation: The picture shows a Kaposi sarcoma (KS) on the face of an HIV-infected individual. There are several types of KS including classic or European, non-HIV-related African endemic, HIV-related, and iatrogenic immunosuppressive-associated KS. The lesion is a vascular neoplasia characterized by multisystem involvement. Herpes human virus-8 has been identified in all variants of the lesions. Differential diagnosis includes other vascular neoplasms such as hemangioma, pyogenic granuloma, and bacillary angiomatosis, as well as ecchymosis, insect bites, stasis dermatitis, and pigmented lesions. Treatment does depend on the extent and type of KS. HIV-related cases have improved with highly active antiretroviral therapy. Intralesional chemotherapy with vinblastine, cryotherapy, radiotherapy, laser surgery, or electrosurgery can be used on individual lesions. More aggressive systemic chemotherapy may be required for extensive mucocutaneous or visceral involvement. Related Topics: kaposi sarcoma 34% of users answered correctly.

This picture shows a lesion on the tongue of a HIV-infected individual (Figure 3-4). Which of the following statements about this disease is NOT correct? A Diagnosis is confirmed by skin biopsy. B Differential diagnosis includes disseminated fungal infections such as cryptococcosis, histoplasmosis, coccidioidomycosis, and penicilliosis. C Human herpes virus has been identified in these lesions. D There are several different types of this disease affecting very different populations. E Treatments may include intralesional or systemic chemotherapy.

The correct answer is E. Varicella zoster virus Explanation: The photo demonstrates Hutchinson's sign, which is the presence of a vesicular eruption on the tip of the nose in an outbreak of varicella zoster virus known as shingles. Zoster lesions present on the nose indicate involvement of the skin innervated by the anterior ethmoidal branch of the nasociliary nerve. Because the nasociliary nerve also innervates the cornea, such skin lesions may indicate ocular involvement. Although HSV is also from the herpes family of viruses, it does not cause zoster; therefore, it is not associated with Hutchinson's sign. Group A strep on the face typically causes erysipelas, which presents with a well-demarcated erythematous plaque involving the cheeks. S. aureus often causes impetigo, both bullous and nonbullous forms, but does not present with the eruption seen in the photo. Human papilloma virus eruptions typically do not involve the face or eyes. Related Topics: herpes zoster disease human herpesvirus 3 81% of users answered correctly.

This rash is caused by which of the following (Figure 3-14)? A Group A streptococcus B Herpes simplex virus (HSV) C Human papilloma virus D S. aureus E Varicella zoster virus

The correct answer is B. Grade II Explanation: This patient has a Mallampati grade II airway. The Mallampati grade (I-IV) predicts the technical ease of intubation. The higher the Mallampati score, the more technically difficult the intubation will likely be.

What Mallampati grade is the airway below? A Grade I B Grade II C Grade III D Grade IV

The correct answer is B. Diverticular disease Explanation: The most common source for GI bleeding is from a site proximal to the ligament of Treitz (UGI bleed). Most UGI bleeds are secondary to peptic ulcer disease. However, in patients with an established lower GI source of bleeding, the most common etiology is diverticular disease, followed by colitis, adenomatous polyps, and malignancy. Related Topics:gastrointestinal hemorrhagelower gastrointestinal hemorrhagediverticulosis 85% of users answered correctly.

What is the MOST common cause of LOWER GI bleeding? A Adenomatous polyps B Diverticular disease C Infectious colitis D Inflammatory colitis E Malignancy

The correct answer is D. Acute myocardial infarction (AMI) Explanation: Cardiogenic shock is mostly commonly due to extensive MI with suppressed myocardial contractility. Pump failure is the underlying factor in most causes of cardiogenic shock (Table 54-2). Hypoperfusion, with or without hypotension, is the unifying feature of cardiogenic shock, regardless of etiology. During an AMI, several mechanical complications can precipitate cardiogenic shock, including acute myocardial regurgitation due to papillary muscle rupture, ventricular septal defect (VSD), and free-wall rupture. Mechanical complications cause one-fourth of the cardiogenic shock following AMI. Right ventricular infarction can also cause cardiogenic shock due to loss of preload. Cardiac contractility can also be severely depressed due to sepsis, myocarditis, myocardial contusion, and cardiomyopathy. Mechanical obstruction to forward blood flow can also lead to cardiogenic shock, including aortic stenosis, HCM, mitral stenosis, left atrial myxoma, and pericardial tamponade. Regurgitation of LV output due to chordal rupture or aortic insufficiency can cause profound cardiogenic shock. 85% of users answered correctly.

What is the MOST common cause of cardiogenic shock? A Acute aortic insufficiency B Aortic stenosis C Hypertrophic cardiomyopathy D Acute myocardial infarction (AMI) E Pericardial tamponade

The correct answer is A. Cytomegalovirus Explanation: While HSV, human papillomavirus, and varicella-zoster virus all have a higher prevalence and a tendency for disseminated disease in HIV-infected individuals, cytomegalovirus is the most common. It can establish itself in many organ systems and becomes a serious risk with CD4+ T cell counts below 50. It is the most serious and common ocular opportunistic infection causing blindness. Gastrointestinal, pulmonary, and central nervous system disseminated disease also occur. 73% of users answered correctly.

What is the MOST common cause of serious opportunistic viral disease in patients with advanced AIDS? A Cytomegalovirus B Herpes simple virus C Influenza virus D Human papillomavirus E Varicella zoster virus

The correct answer is B. Escherichia coli Explanation: Diarrhea is a common occurrence in travelers returning from developing countries. Toxin and nontoxin-producing strains of E. coli account for most identifiable cases in Mexico and South America. The invasive bacteria are more commonly seen in travelers to southern Asia. In the United States, the most common cause of infectious diarrhea is the norovirus, accounting for 50-80% of cases. C. difficile and protozoa are known to cause infectious diarrhea, but they are not the most common cause of traveler's diarrhea in patients returning from Central and South America. 67% of users answered correctly.

What is the MOST common cause of traveler's diarrhea in patients returning from Latin America? A Clostridium difficile B Escherichia coli C Invasive bacteria (Campylobacter jejuni, shigella, salmonella) D Norovirus E Protozoa

The correct answer is B. Right lower quadrant Explanation: It is important to remember that appendectomy is the most common nonobstetrical surgical procedure in pregnant women. The right lower quadrant is the most common location of pain in patients with an acute appendicitis. Pregnant women are no exception. However, one must consider appendicitis in other areas of the abdomen especially right upper quadrant because the uterus can displace the intestines into the abdominal cavity as the uterus grows. 63% of users answered correctly.

What is the MOST common location of pain in a pregnant woman with appendicitis? A Right upper quadrant B Right lower quadrant C Left upper quadrant D Left lower quadrant E Umbilicus

The correct answer is C. Gallstones Explanation: Gallstones are the leading cause of acute pancreatitis, accounting for at least 35-40% of cases. Alcohol is the second most frequent cause. Hypertriglyceridemia accounts for 1-4% of cases. Drugs account for <2% of cases. 62% of users answered correctly.

What is the MOST frequent cause of acute pancreatitis? A Alcohol B Drug induced C Gallstones D Hypercalcemia E Hypertriglyceridemia

The correct answer is E. Ultrasound Explanation: Right upper quadrant ultrasound is the imaging modality of choice for diagnosing acute cholecystitis in the ED. It has a sensitivity of 94% and a specificity of 78% for detecting acute cholecystitis. A sonographic Murphy sign has a high PPV for acute cholecystitis. Ultrasonography is readily available in the ED. CT scan of the abdomen is useful in diagnosing acute cholecystitis when ultrasound results are equivocal. ERCP and MRCP are useful for the evaluation of the biliary tree to delineate the cause of obstruction, but are not readily available from the ED. ERCP has the advantage of being both diagnostic and therapeutic. MRI of the abdomen is useful when ultrasonography and CT findings are inconclusive. While it has the highest sensitivity for signs of cholecystitis, it is not as readily available. Related Topics:cholecystitis, acutecholecystitis 87% of users answered correctly.

What is the imaging modality of choice in the ED for diagnosing acute cholecystitis? A CT of the abdomen B Endoscopic retrograde cholangiography (ERCP) C Magnetic resonance cholangiopancreatography (MRCP) D Plain radiographs E Ultrasound

The correct answer is D. Hypertrophic cardiomyopathy Explanation: Hypertrophic cardiomyopathy is the leading cause of sudden death in competitive athletes and in adolescents. Commotio cordis and congenital coronary artery anomalies are, respectively, the second and third leading causes of sudden death in adolescents. Commotio cordis is a term used when a physical blow to the chest wall causes ventricular fibrillation or sudden death without evident structural damage to the thorax or heart. Idiopathic dilated cardiomyopathy is the most common cardiomyopathy and the cause of about one-fourth of all cases of congestive heart failure. Heat stroke is a preventable cause of injury or death in athletes. 75% of users answered correctly.

What is the leading cause of death in competitive athletes? A Congenital coronary artery anomalies B Commotio cordis C Heat stroke D Hypertrophic cardiomyopathy E Idiopathic dilated cardiomyopathy

The correct answer is A. 1 in 500 Explanation: A CT scan gives about 10-20 mSv of radiation and increases the lifetime risk to at least 1 in 500. The risk in young women may be higher given breast radiation. Other life-threatening reactions to CT scans of the chest include anaphylactoid reactions and pulmonary edema, which together occur in about 1 in 1000 patients. 32% of users answered correctly.

What is the lifetime risk of a fatal cancer or leukemia after a CT scan of the chest? A 1 in 500 B 1 in 1000 C 1 in 2000 D 1 in 10,000 E 1 in 20,000

The correct answer is D. Sinus tachycardia Explanation: Sinus tachycardia is a common finding on ECG. As the severity of the PE increases, the right heart begins to work harder and the right-sided pressures increase. Once the RV systolic pressures exceed 40 mm Hg, the T-wave inversions, incomplete or complete RBB, and the classic S1 Q3 T3 patterns may emerge. 79% of users answered correctly.

What is the most common abnormal ECG finding in patients with a PE? A Normal ECG B Right bundle branch block C S1 Q3 T3 (McGinn-White sign) D Sinus tachycardia E T-wave inversions in leads V1-V4

The correct answer is D. Neoplasm Explanation: Neoplasms are by far the most common cause of large bowel obstruction. Colonic obstruction is almost never caused by hernia or surgical adhesions. Adhesions and hernias are common causes for small bowel obstruction in the elderly, whereas carcinoma is the most likely cause of large bowel obstruction in the elderly because of the increased likelihood of cancer as people age. 76% of users answered correctly.

What is the most common cause of large bowel obstruction in a 65-year-old patient? A Adhesions B Hernia C Mesenteric ischemia D Neoplasm E Volvulus

The correct answer is A. Autoimmune disorders Explanation: Approximately 70% of primary AI is caused by autoimmune disorders. Adrenal hemorrhage and thrombosis are usually caused by meningococcal sepsis, coagulation disorders (including anticoagulation therapy), Waterhouse-Frederickson syndrome, trauma, and antiphospholipids syndrome. Drugs such as ketoconazole and adrenolytic agents such as metyrapone, aminoglutethimide, and mitotane can also cause primary AI. Worldwide, the most common infectious agent causing primary AI is tuberculosis. In the United States, the most common infectious agent is HIV/AIDS. 44% of users answered correctly.

What is the most common cause of primary adrenal insufficiency (AI) in the United States? A Autoimmune disorders B Adrenal hemorrhage C Drugs D Idiopathic E Infectious

The correct answer is D. Hemorrhoids Explanation: Hemorrhoidal bleeding is usually limited, with the blood being found on the surface of the stool, on the toilet tissue, or noted at the end of defecation, dripping into the toilet bowl. When patients describe the passage of blood clots, colonic lesions should be suspected and investigated. Although the most common cause of rectal bleeding is hemorrhoids, other, more serious causes should be investigated. 69% of users answered correctly.

What is the most common cause of rectal bleeding? A Anal fissures B Colon cancer C Diverticulosis D Hemorrhoids E Ulcerative colitis

The correct answer is A. Hemorrhage Explanation: Potential complications of colonoscopy include hemorrhage, perforation, retroperitoneal abscess, pneumoscrotum, pneumothorax, volvulus, postcolonoscopy distention, splenic rupture, appendicitis, bacteremia, and infection. Hemorrhage is the most common complication and can be secondary to polypectomy, biopsies, laceration of the mucosa by the instrument, or tearing of the mesentery or spleen. If the bleeding is intraluminal, the patient will develop rectal bleeding. Patients with mesenteric or splenic injury will present with signs of intra-abdominal bleeding. Treatment of intraluminal bleeding depends on the magnitude of hemorrhage. Intra-abdominal bleeding requires emergency laparotomy. Colon perforation with pneumoperitoneum usually is evident immediately but can take several hours to manifest. Perforation is usually secondary to intrinsic disease of the colon (e.g., diverticulitis) or to vigorous manipulation during the procedure. Most patients will require immediate laparotomy, but in some patients presenting late (1-2 days later) without signs of peritonitis, hospital observation may be appropriate. 58% of users answered correctly.

What is the most common complication after colonoscopy? A Hemorrhage B Infection C Perforation D Splenic rupture E Volvulus

The correct answer is E. Hypotension Explanation: Enalaprilat is an intravenous ACE inhibitor that may be indicated in the treatment of patients with heart failure or acute coronary syndrome. Hypotension is the most common side effect with a first dose, so a preliminary test dose of 0.625 mg is recommended. Angioedema, cough, and hyperkalemia are all potential side effects of ACE inhibitors and angiotensin II receptor blockers. Bradycardia is a potential side effect associated with beta-blockers and calcium channel blockers (see Table 61-9). 54% of users answered correctly.

What is the most common first-dose side effect of enalaprilat? A Angioedema B Bradycardia C Cough D Hyperkalemia E Hypotension

The correct answer is C. Chest pain Explanation: In a case series of 464 patients with aortic dissections, 60% presented with chest pain, 64% stated the pain was "sharp," and 50% stated it was tearing or ripping. Syncope occurred 10% of the time. In all 90% of the patients stated that the pain associated with their dissection was the worst they had ever felt. 59% of users answered correctly.

What is the most common location of pain in patients with aortic dissection? A Abdominal pain B Back pain C Chest pain D Left arm pain E Neck pain

The correct answer is E. Trichophyton Explanation: The picture shows tinea capitis with a kerion. The most common cause of kerion development is an inflammatory response with infection of tinea capitis. The most common cause of tinea capitis is Trichophyton. Kerion is an inflamed, boggy plaque with pustules and alopecia. It may cause permanent scarring. Diagnosis is based on positive KOH or fungal culture. Trichophyton invade the hair shaft, so may not have a positive KOH. Wood's light may also be used to help diagnose, but Trichophyton does not fluoresce. First-line treatment is oral griseofulvin. Related Topics: tinea capitis 62% of users answered correctly.

What is the most common organism responsible for this lesion (Figure 3-7)? A Malassezia furfur B Microsporum C Streptococcus pyogenes D Staphylococcus aureus E Trichophyton

The correct answer is A. Acute limb ischemia Explanation: Limb ischemia can be caused by either arterial thrombosis or embolization from the aneurysm. It is the most common serious complication. Rupture can occur, but it is exceedingly rare. Discomfort, pain, or swelling may be present, but they are precursors to the more serious complications.

What is the most common serious complication of a popliteal artery aneurysm? A Acute limb ischemia B Bruising C Deep VTE D Pain E Rupture of aneurysm

The correct answer is E. Transient hypotension Explanation: Transient hypotension is clinically the most important complication of nitroglycerin use, although headache is a very common side effect that usually responds to acetaminophen. Nitroglycerin should be avoided or used with extreme caution in preload-dependent conditions such as aortic stenosis, HCM, and right ventricular infarct. Nitroglycerin has caused methemoglobinemia in very rare instances. Nitroprusside can cause thiocyanate toxicity, especially when used in patients with renal insufficiency. 57% of users answered correctly.

What is the most important clinical complication of nitroglycerin use? A Headache B Persistent hypotension C Methemoglobinemia D Thiocyanate toxicity E Transient hypotension

The correct answer is B. A primary infection with herpes simplex virus (HSV) type 1 Explanation: Gingivostomatitis and pharyngitis are the typical primary manifestations of HSV type 1. Although HSV-2 can cause similar disease, HSV-1 is much more likely to cause oral lesions. Reactivation of HSV-1 from the trigeminal ganglion typically presents as orolabial lesions. HSV-2 is believed to reside in the sacral ganglion and represents as genital lesions. EBV causes infectious mononucleosis. Related Topics: human herpesvirus 1 58% of users answered correctly.

What is the most likely cause of this lesion (Figure 10-8)? A A primary infection with coxsackie A virus B A primary infection with herpes simplex virus (HSV) type 1 C A primary infection with HSV type 2 D A reactivation of HSV type 1 or type 2 from the trigeminal ganglion E Infectious mononucleosis caused by the Epstein-Barr virus (EBV)

The correct answer is B. 10 minutes Explanation: The current recommendations are for ECGs to be performed as soon as possible and preferably within the first 10 minutes. This short door-to-ECG time is an effort by the ACC and AHA to shorten the door to intervention time in patients with an AMI. 82% of users answered correctly.

What is the recommended door-to-ECG time in patients presenting with chest pain? A 1 minute B 10 minutes C 15 minutes D 20 minutes E 30 minutes

The correct answer is C. Presence suggests an elevated pulmonary capillary wedge pressure Explanation: The presence of third heart sound, or ventricular filling gallop, has a specificity of 99% and highly suggests increased pulmonary capillary wedge pressure and helps rule in CHF. Detection by stethoscope can be challenging and has a sensitivity of only 20%. Use of digital microphone detection during ECG testing improves sensitivity to 40.2%; specificity is 88.5%. The presence of an S3 is associated with notably worse outcomes, including a doubling of 90-day mortality. JVD is another excellent physical finding for an elevated pulmonary capillary wedge pressure, with a specificity of 94% and a sensitivity of 39%. 87% of users answered correctly.

What is the significance of a third heart sound (S3) in the diagnosis of CHF? A Absence suggests increased 90-day mortality B Absence suggests worse outcome C Presence suggests an elevated pulmonary capillary wedge pressure D Presence suggests improved outcome E Presence rules out CHF

The correct answer is B. 50% Explanation: Dyspnea is the most common chief complaint in patients with a PE. Chest pain is the second most common but is only present in 50% of patients who are diagnosed with a PE. The skilled clinician must not be complacent in the patient with no complaint of chest pain. A PE must still be in the differential. 62% of users answered correctly.

What percent of patients who are diagnosed with PE in the ED have a complaint of chest pain? A 25% B 50% C 75% D 90% E 100%

The correct answer is E. 50% Explanation: Women and the elderly are more likely to have atypical symptoms. Up to 50% of patients with atypical symptoms are found to have unstable angina. In addition, the prognosis for patients who present with atypical symptoms is worse. 42% of users answered correctly.

What percent of patients with unstable angina present with atypical features? A 10% B 20% C 30% D 40% E 50%

The correct answer is E. >35% Explanation: It has been reported that as many as 37.5% of women and 27.4% of men presenting with ischemia may present without chest pain. Up to 30% of patients with AMI in longitudinal studies are unrecognized. 60% of users answered correctly.

What percent of women with confirmed cardiac ischemia present without chest pain? A 5% B 10% C 20% D 25% E >35%

The correct answer is B. 40% Explanation: Pulmonary emboli form when proximal portions of DVTs break off and lodge in the precapillary pulmonary arteries. In hospitalized patients, 75-80% have image demonstrated DVTs. In ED patients, however, far fewer (40%) have image demonstrated DVTs. 63% of users answered correctly.

What percentage of ambulatory ED patients with a documented PE have a concomitant deep vein thrombosis (DVT)? A 20% B 40% C 60% D 80% E 100%

The correct answer is B. 10 Explanation: Remember that ECG findings alone cannot rule out acute coronary syndrome. Up to 6% of patients with a normal ECG will ultimately be proven to have an NSTEMI. Another 4% will be proven to have unstable angina. Nonspecific changes (T-wave flattening or inversion) can also be concerning if they reverse when the patient is pain free.

What percentage of patients with a normal ECG will ultimately have a diagnosis of NSTEMI or unstable angina? A 5 B 10 C 15 D 20 E 25

The correct answer is B. Family history Explanation: While it is important to recognize all factors that increase cardiovascular disease, a family history of a first-degree relative with an AAA has been shown to be highly predictive. Eighteen percent of patients with an AAA have a first-degree relative who also had an AAA. Diabetes mellitus, hypertension, high cholesterol, and smoking may all be associated with the development of an AAA or cardiovascular disease in general but not as much as a strong family history. 31% of users answered correctly.

What risk factor is highly predictive of developing an abdominal aortic aneurysm (AAA)? A Diabetes B Family history C Hypercholesterolemia D Hypertension E Smoking history

The correct answer is A. Close contact with a bat in a confined space Explanation: The most cases of rabies in the United States are bat related. The risk of rabies following exposure to a bat may be underestimated since the wounds inflicted from a bite may be perceived as insignificant. Current guidelines suggest treatment when a person is not absolutely sure a bite did not occur. Such is the case when a bat is found with a sleeping person, unattended infant, mentally disabled person, or intoxicated person. Raccoons, skunks, and foxes are all considered rabid unless proven otherwise, but the virus is only transmitted through saliva. The virus replicates in the salivary gland of infected animals and is transmitted through nonintact skin or mucous membranes. Dry wounds, such as a scratch from a claw, can be considered noninfectious. 75% of users answered correctly.

What type of nonbite exposure is highest risk and may require PEP for rabies? A Close contact with a bat in a confined space B Dry scratch from a stray cat C Petting a wild dog D Sprayed by a skunk E Stepping in raccoon feces

The correct answer is D. Significant radiation exposure during 8-15 weeks may result in decreased neurodevelopment. Explanation: Significant radiation exposure during 8-15 weeks may result in decreased neurodevelopment including a small head size. Chest CT results in less radiation exposure (0.02-0.1 rad) than ventilation and perfusion scan (0.215 rad) for pulmonary embolism. The first 8 weeks of pregnancy is the period of organogenesis. Fetal exposure to less than 5 rad does not increase the risk of fetal death, mental defect, or growth retardation. 44% of users answered correctly. Explanation: Significant radiation exposure during 8-15 weeks may result in decreased neurodevelopment including a small head size. Chest CT results in less radiation exposure (0.02-0.1 rad) than ventilation and perfusion scan (0.215 rad) for pulmonary embolism. The first 8 weeks of pregnancy is the period of organogenesis. Fetal exposure to less than 5 rad does not increase the risk of fetal death, mental defect, or growth retardation. 44% of users answered correctly.

When discussing the risks of radiation and diagnostic tests with pregnant patients, which of the following statements is true? A Chest CT scanning results in more radiation exposure than ventilation-perfusion scanning for pulmonary embolism. B Fetal exposure to less than 10 rad does not increase the risk of fetal death, mental defect, or growth retardation. C Most of the detrimental effects of radiation occur after 15 weeks of gestation. D Significant radiation exposure during 8-15 weeks may result in decreased neurodevelopment. E The first 2 weeks of pregnancy are the period of organogenesis and therefore the highest risk period.

The correct answer is D. Plain abdominal films Explanation: It is important to recognize that surgical consultation should not be delayed awaiting the results of imaging in the acute abdomen. If imaging is obtained in cases of suspected acute bowel perforation, then the most appropriate initial imaging modality is plain abdominal films as they are relatively easy to obtain, and free air visible on a plain abdominal film mandates immediate surgical evaluation without the need for additional imaging. However, abdominal plain films cannot rule out perforation. Up to 20% of patients with perforation will not demonstrate free air. Ultrasonography is the preferred modality for the evaluation of the biliary tract. It may be a more sensitive test than abdominal plain films for the detection of free air, but it is highly operator dependent, requires specialized equipment, and is limited by obesity and overlying gas. While CT scan of the abdomen is superior in identifying free air when compared with plain abdominal films, it is associated with significant delays due to the administration of contrast and radiologist interpretation. Cholescintigraphy and MRI of the abdomen do not have a role in the initial evaluation of suspected acute bowel perforation. 68% of users answered correctly.

When evaluating a patient for suspected acute bowel perforation, what is the MOST appropriate initial imaging modality? A Cholescintigraphy B Computed tomography (CT) of the abdomen with contrast C Magnetic resonance imaging (MRI) of the abdomen D Plain abdominal films E Ultrasonography of the abdomen

The correct answer is C. Perianal Explanation: An isolated perianal abscess not associated with deeper, perirectal abscesses is generally the only type of anorectal abscess that can be adequately treated in the ED. Treatment of perianal abscesses is incision and drainage. All perirectal abscesses (supralevator, intersphincteric, and complicated ischiorectal) should be drained in the operating room. Related Topics:perianal abscess 80% of users answered correctly.

Which anorectal abscess can be appropriately treated in the ED? A Intersphincteric B Ischiorectal C Perianal D Submucosal E Supralevator

The correct answer is C. Preexisting mild hepatic impairment Explanation: NSAID-induced acute renal failure is more common in patients who are volume depleted, have preexisting renal or cardiac disease, or are taking loop diuretics. Mild hepatic impairment does not affect the metabolism of most analgesics nor is it known to increase the risk of acute renal failure with this class of medications. 72% of users answered correctly.

Which is NOT generally thought to increase the risk of NSAID-induced acute renal failure? A Loop diuretic use B Preexisting cardiac disease C Preexisting mild hepatic impairment D Preexisting mild renal disease E Volume depletion

The correct answer is A. Chronic pain is more common in elderly patients. Explanation: Chronic pain is more common in elderly patients and the most common site of chronic pain is the back. Complete pain relief is an unrealistic goal for chronic pain patients who present to the emergency department. Chronic pain patients who present to the emergency department may be best helped by referral to a pain clinic or specialists; hospital admission is seldom indicated.

Which is TRUE about chronic pain? A Chronic pain is more common in elderly patients. B Less than 25% of patients whose pain lasts 3 months will continue to have chronic pain for several years. C The goal of emergency treatment should be complete relief of pain. D The head is the most common location of chronic pain. E Patients presenting to the emergency department because of chronic pain are best helped by hospital admission.

The correct answer is E. 3-5 mL of anesthetic solution should be used for each rib blocked. Explanation: The correct volume of anesthetic for each intracostal block is 3-5 mL. Pneumothorax occurs in 8-9% of patients or about 1.4% for each individual intercostal block. There are no rigorous controlled trials comparing parental analgesia with intercostal nerve blocks. An intercostal block should provide anesthesia for 8-18 hours. Due to the position of the scapula and rhomboid muscles, ribs 1-6 are difficult to block.

Which is TRUE regarding an intracostal nerve block? A Anesthesia duration is generally 24-48 hours. B Pneumothorax occurs in about 25% of patients who have this procedure. C Ribs 1-6 are easiest to block. D This type of block is superior to parenteral analgesia. E 3-5 mL of anesthetic solution should be used for each rib blocked.

The correct answer is B. Epinephrine intramuscularly and antihistamines Explanation: Anaphylaxis from a Hymenoptera sting should be treated like any other cause of anaphylaxis. Intramuscular epinephrine is the mainstay of treatment. Antihistamines, both H1- and H2-blockers, may be helpful adjuncts but should not delay or precede administration of epinephrine. Corticosteroids may prevent delayed reactions but do not treat acute anaphylaxis. Beta-agonists such as albuterol are helpful adjuncts for patients who are wheezing. Intravenous epinephrine is reserved for patients with hypotension refractory to initial treatment or those in cardiac arrest. No antivenom is currently commercially available. Discharge with epinephrine autoinjector (e.g., Epi-pen™) is appropriate for all patients who recover from acute anaphylaxis. Outpatient referral for desensitization with venom immunotherapy is recommended for those patients with anaphylaxis due to Hymenoptera stings. 80% of users answered correctly. Hymenoptera is a large order of insects, comprising the sawflies, wasps, bees, and ants

Which is the best immediate treatment for anaphylaxis due to a Hymenoptera sting? A Corticosteroids and H2-receptor antagonists B Epinephrine intramuscularly and antihistamines C Epinephrine intravenously and antivenom D Removal of stinger and nonsteroidal anti-inflammatory agent E Venom immunotherapy and discharge with a prescription for an epinephrine autoinjector (e.g., Epi-pen™).

The correct answer is C. Flexion of the hips (McRoberts maneuver) Explanation: When shoulder dystocia is recognized, the patient's legs should be immediately flexed up to the abdomen with the legs held by the patient or assistant known as the McRoberts maneuver. Next suprapubic pressure should be applied to disimpact the anterior shoulder from the pubic symphysis. Fundal pressure should never be applied because it will further impact the shoulder on the pelvic rim. If McRoberts maneuver and suprapubic pressure do not work, the Wood corkscrew maneuver can be performed by grasping the posterior scapula and rotating the infant's torso 180 degrees to rotate the posterior shoulder into the anterior position to deliver the shoulder. If this fails, the physician may try to deliver the posterior shoulder. 61% of users answered correctly.

Which is the first maneuver that should be performed when managing a shoulder dystocia? A Clockwise rotation of the baby's torso (Wood corkscrew) B Delivery of the posterior shoulder C Flexion of the hips (McRoberts maneuver) D Fundal pressure E Suprapubic pressure

The correct answer is A. Limb Explanation: The odd ratio (OR) for limb immobility is 2.24 that is the highest amongst a multicenter study of 7940 patients with VTE. Neurologic immobility has an OR of 2.23. Whole body immobilization was 1.76 and traveling >8 hours was not a statistically significant risk factor. 45% of users answered correctly.

Which kind of immobility confers the greatest risk for VTE? A Limb B Neurologic C Travel >4 hours D Travel >8 hours E Whole body

The correct answer is C. PR segment depression, ST-segment elevation, ST/T amplitude >0.25 Explanation: Early acute pericarditis can mimic the ST-T-wave changes of early repolarization, a normal variant. The early ECG stage of acute pericarditis is characterized by PR segment depression (especially in II, aVF, and V4-V6), ST-segment elevation (especially in I, V5, and V6), and ST/T amplitude >0.25 (Table 59-7). Using the PR segment baseline as 0 mV, the height in millivolts, of the ST segment at its onset, is divided by the height in millivolts of the T wave, from its baseline to peak. An ST/T ratio >0.25 suggests acute pericarditis, while a ratio <0.25 does not. The ST/T ratio >0.25 has a sensitivity and specificity greater than 85% and 80%, respectively. As acute pericarditis progresses, the ST segment becomes isoelectric with inverted T waves (especially in I, V5, and V6). In the final stage, the T wave normalizes to an upright position. 50% of users answered correctly.

Which of the following ECG features is indicative of the early stage of acute pericarditis? A Isoelectric ST segment with T-wave inversion B PR segment depression, ST-segment elevation, ST/T amplitude <0.25 C PR segment depression, ST-segment elevation, ST/T amplitude >0.25 D PR segment elevation, ST-segment elevation, ST/T amplitude <0.25 E PR segment elevation, ST-segment elevation, ST/T amplitude >0.25

The correct answer is C. Diabetes and smoking Explanation: Diabetes and smoking are the two most important risk factors for occlusive arterial disease. In the PARTNERS study, patients older than 50 years with diabetes or a smoking had a prevalence >29%. In other studies, more than 80% of patients with occlusive arterial disease are either former or current smokers. Other risk factors include hyperlipidemia, hypertension, hyperhomocysteinemia, and elevated C-reactive protein. Occlusive arterial disease, defined as an ankle-brachial index <0.9, increases with age, is two to four times more common in men than in women, and more prevalent in non-Caucasians. (See Figure 64-1 for description of ankle-brachial index.) 57% of users answered correctly.

Which of the following are pathophysiologically the most important risk factors for occlusive arterial disease? A Diabetes and hyperlipidemia B Diabetes and hypertension C Diabetes and smoking D Elevated C-reactive protein and hyperhomocysteinemia E Hypertension and hyperlipidemia.

The correct answer is C. Standing Explanation: The murmur of HCM intensifies with actions that decrease LV filling and the distending pressure of the LV outflow tract, such as standing, and the Valsalva maneuver (strain phase). The murmur of HCM also intensifies with increased force of myocardial contraction. Hence, the murmur is louder with the first sinus beat after a premature ventricular contraction. The systolic ejection murmur of HCM is heard best at the lower left sternal border or the apex. Maneuvers that increase LV filling (squatting, passive leg raise, and sustained handgrip) decrease the murmur of HCM (see Table 59-5). When a patient turns their chin toward a stethoscope in the right supraclavicular fossa, a venous hum decreases.

Which of the following bedside maneuvers increases the systolic ejection murmur of hypertrophic cardiomyopathy (HCM)? A Passive leg raise in supine position B Squatting C Standing D Sustained isometric handgrip E Turning chin toward a stethoscope in the right supraclavicular fossa

The correct answer is A. Bimodal age distribution Explanation: Aortic dissections occur both in patients who have predisposing conditions and in those older than 50 years with chronic hypertension. Patients who are at risk of developing aortic dissection at a young age are those with Marfan's syndrome, Ehlers-Danlos syndrome, and those with a bicuspid aortic valve. Each of these leads to higher intimal wall stress which can, in turn, lead to dissection. 60% of users answered correctly.

Which of the following best describes the ages of patients who develop aortic dissections? A Bimodal age distribution B Patients between 45 and 65 years C Patients between 65 and 85 years D Patients older than 85 years E Young patients with risk factors

The correct answer is A. Early diastolic sound 60-120 milliseconds after second heart sound Explanation: A pericardial knock, an early diastolic sound, that can be heard at the apex 60-120 milliseconds after the second heart sound but before an S3 is associated with constrictive pericarditis but is clinically very challenging to differentiate from an S3. Both conditions share the signs of JVD and Kussmaul, which is an increase in venous pressure with inspiration. The dip and plateau filling pattern, also known as the square root sign, is seen in both conditions. A marked decrease followed by a rapid rise and plateau in early-diastolic ventricular pressure is seen in hemodynamic studies, such as Doppler echocardiography or cardiac catheterization. CT and MRIs of the heart can differentiate constrictive pericarditis from restrictive cardiomyopathy. Pericardial thickening is seen with constrictive pericarditis, but not with restrictive cardiomyopathy. Constrictive pericarditis can be treated surgically. Restrictive cardiomyopathy is symptomatically managed with diuretics and ACE inhibitors and medical therapies specific to the underlying etiology, for example, corticosteroids for sarcoidosis and chelation for hemachromatosis. 35% of users answered correctly.

Which of the following can distinguish constrictive pericarditis from restrictive cardiomyopathy? A Early diastolic sound 60-120 milliseconds after second heart sound B Dip and plateau filling pattern C Jugular venous distention D Kussmaul sign E Square root sign

The correct answer is A. Angiotensin-converting enzyme inhibitors Explanation: ACE inhibitors are contraindicated in pregnant women due to potential teratogenic effects on the fetus. Hydralazine is no longer considered first-line therapy for eclampsia due to an unpredictable therapeutic profile. Nifedipine has been used in preeclamptic women without significant side effects, but should be avoided in other types of nonpregnancy-related hypertensive emergencies. Labetalol and magnesium sulfate are indicated in the treatment of eclampsia. 81% of users answered correctly.

Which of the following cardiac medications is contraindicated in the treatment of pregnant women? A Angiotensin-converting enzyme inhibitors B Hydralazine C Labetalol D Magnesium sulfate E Nifedipine

The correct answer is B. Iron Explanation: Elevated anion gaps are usually associated with metabolic acidosis. While the differential for elevated anion gap acidosis is broad, it is important to separate out processes that cause the elevation directly and those that cause it indirectly by producing lactate. Without this separation, important components in the differential for lactic acidosis will be missed. Biguanides, cyanide, iron, and theophylline are not unmeasured ions causing an elevated anion gap. These substances actually raise the anion gap by producing a lactic acidosis. Related Topics:anion gap acidosis 25% of users answered correctly.

Which of the following causes an anion gap acidosis solely by the production of lactate? A Ethylene glycol B Iron C Ketoacidosis D Methanol E Uremia

The correct answer is E. Volvulus Explanation: Acute constipation is secondary to intestinal obstruction until proven otherwise. Common causes of intestinal obstruction include rapidly growing tumors, strictures, hernia, adhesions, inflammatory conditions, and volvulus. Hyperparathyroidism, hypomagnesemia, and Parkinson disease cause chronic constipation. Related Topics:intestinal volvulus 69% of users answered correctly.

Which of the following conditions is a cause of ACUTE constipation? A Amyloidosis B Hyperparathyroidism C Hypomagnesemia D Parkinson disease E Volvulus

The correct answer is D. Staphylococcus scalded skin Explanation: A positive Nikolsky sign occurs when there is a detachment of the epidermis from the dermis with gentle traction on the skin as demonstrated in the photo. Disease processes that exhibit this finding include staphylococcus scalded skin, toxic epidermal necrolysis, and pemphigus vulgaris. Eczema herpeticum, toxic shock syndrome, HSV, bullous pemphigoid, and exfoliative erythroderma all involve detachment of skin but do not demonstrate a positive Nikolsky sign. Related Topics: staphylococcal scalded skin syndrome 53% of users answered correctly.

Which of the following conditions is associated with the following clinical finding (Figure 3-13)? A Bullous pemphigoid B Eczema herpeticum C Exfoliative erythroderma D Staphylococcus scalded skin E Toxic shock syndrome

The correct answer is B. Coma Explanation: Hypothermia causes multiple metabolic changes. As temperature decreases, the victim becomes initially uncoordinated, progressively confused, and finally comatose. Respiratory depression (not distress) occurs, and cough and gag reflexes are also diminished. Hypothermic patients are prone to bleeding due to inhibition of platelet function, alterations in coagulation cascade enzyme function, and may be associated with disseminated intravascular coagulation. Patients with hypothermia are usually volume depleted for a variety of reasons, and may have renal dysfunction secondary to rhabdomyolysis. Patients are at risk for cardiac arrhythmias. The ECG may show the classic Osborne or J-wave, a terminal deflection at the end of the QRS complex. A delta wave is a slurring of the QRS complex seen in patients with Wolff-Parkinson-White syndrome. 61% of users answered correctly.

Which of the following conditions is most likely to be observed in an alcoholic man found in an alley in January with a core temperature of 31°C? A Delta wave on the ECG B Coma C Hypercoagulability D Respiratory distress E Volume overload

The correct answer is C. Hypertensive heart failure syndrome Explanation: Patients with hypertensive heart failure and pulmonary edema would be expected to respond well to the decrease in preload and afterload associated with vasodilator therapy. Since patients with aortic stenosis, HCM, and right-ventricular infarction are all preload dependent, standard doses of vasodilator therapy could precipitate profound hypotension. Vasodilators would not be indicated in a patient with volume depletion.

Which of the following conditions would MOST likely to respond well to vasodilator therapy (nitroglycerin or nitroprusside) in standard doses? A Aortic stenosis B Hypertrophic cardiomyopathy C Hypertensive heart failure syndrome D Right-ventricular infarction E Volume depletion

The correct answer is C. Hypertensive heart failure syndrome Explanation: Patients with hypertensive heart failure and pulmonary edema would be expected to respond well to the decrease in preload and afterload associated with vasodilator therapy. Since patients with aortic stenosis, HCM, and right-ventricular infarction are all preload dependent, standard doses of vasodilator therapy could precipitate profound hypotension. Vasodilators would not be indicated in a patient with volume depletion. 76% of users answered correctly.

Which of the following conditions would MOST likely to respond well to vasodilator therapy (nitroglycerin or nitroprusside) in standard doses? A Aortic stenosis B Hypertrophic cardiomyopathy C Hypertensive heart failure syndrome D Right-ventricular infarction E Volume depletion

The correct answer is B. Epigastric auscultation of air insufflated through the tube Explanation: Nasogastric tube placement is confirmed by epigastric auscultation of air insufflated through the tube, aspiration of visually recognizable GI secretions, or pH testing of aspirates (pH <6 indicates gastric placement). Related Topics:nasogastric tubenasogastric tube placement 75% of users answered correctly.

Which of the following confirms appropriate placement of a nasogastric tube? A Air bubbles when proximal end of tube is placed in water B Epigastric auscultation of air insufflated through the tube C Patient choking or coughing D Patient's inability to speak E pH testing of aspirates (pH >6 indicates gastric placement)

The correct answer is D. Rapid rewarming in warm water bath and aloe vera cream for blisters Explanation: Frostbite should never be rewarmed in the field if there is a chance of refreezing. Rapid rewarming should be done with a water bath at 40-42°C after there is no chance for refreezing. Dry heat should not be used. Elevation, immobilization, and meticulous wound care will aid in healing. Debridement of blisters is controversial, but application of aloe vera cream is considered beneficial. Amputation and soft tissue debridement should be deferred, as full demarcation of devitalized tissue may take weeks. MRI and radionuclide angiography with bone scan (but not CT) have been used for early assessment of tissue viability. Heparin is not beneficial, but tissue plasminogen activator (TPA) has been used successfully to prevent amputation. Prophylactic antibiotics are controversial in all but deep or clearly infected cases. Tetanus prophylaxis is indicated. 67% of users answered correctly.

Which of the following describes the most appropriate treatment for frostbite? A Debridement of blisters and early amputation to prevent spread of tissue damage B Immediate rewarming by any means possible in the field C Heparin and prophylactic antibiotics in all cases D Rapid rewarming in warm water bath and aloe vera cream for blisters E Early mobilization for lower extremity injuries and CT scan to assess degree of injury

The correct answer is B. Decompression sickness—recompression with hyperbaric oxygen Explanation: Decompression sickness and arterial gas embolism are treated with recompression using hyperbaric oxygen (HBO) in a specially designed chamber. Nitrogen narcosis occurs at depths of 100 feet seawater or greater and improves upon ascent. Pulmonary barotrauma (e.g., pneumothorax) should be treated with tube thoracostomy drainage but does not require recompression with HBO unless there is an associated arterial gas embolism. Sinus barotraumas is best treated symptomatically with decongestants, whereas inner ear barotrauma may require otologic intervention. 69% of users answered correctly.

Which of the following diving injuries is correctly paired with its appropriate treatment? A Arterial gas embolism—heparin and thrombolytics B Decompression sickness—recompression with hyperbaric oxygen C Nitrogen narcosis—recompression with hyperbaric oxygen D Pulmonary barotrauma—recompression with hyperbaric oxygen E Sinus barotrauma—immediate otolaryngology consult

The correct answer is D. Use of assisted reproduction techniques Explanation: The following are major risk factors for ectopic pregnancy: assisted reproduction techniques, PID, history of tubal surgery, use of an intrauterine device, and previous ectopic pregnancy. Young maternal age and low socioeconomic class are risk factors associated with postpartum endometritis. Ovarian cysts are a risk factor for ovarian torsion. 74% of users answered correctly.

Which of the following factors is a major risk factor for ectopic pregnancy? A Depression B History of ovarian cysts C Low socioeconomic class D Use of assisted reproduction techniques E Young maternal age

The correct answer is B. Exercise-induced mitral regurgitation Explanation: Patients with mitral valve prolapse and exercise-induced mitral regurgitation have an increased risk of morbidity. Patients with mitral valve prolapse (MVP) and mitral regurgitation require endocarditis prophylaxis before invasive procedures. Other factors that place patients at risk for complications include male sex and age older than 45 years. Low body weight and scoliosis are associated with mitral valve prolapse. 54% of users answered correctly.

Which of the following features INCREASES morbidity in a patient with mitral valve prolapse who is otherwise asymptomatic? A Age <45 years at symptom onset B Exercise-induced mitral regurgitation C Female gender D Low body weight E Scoliosis

The correct answer is E. Pectus excavatum Explanation: Pectus excavatum, scoliosis, and low body weight have been found to be associated with mitral valve prolapse. Studies have not demonstrated an association of chest pain, dyspnea, or anxiety with the disorder. 46% of users answered correctly.

Which of the following features have proven association with mitral valve prolapse? A Anxiety B Chest pain C Dyspnea D Obesity E Pectus excavatum

The correct answer is A. A history of cirrhosis or ascites on examination Explanation: Clinical features predicting adverse outcomes include initial hematocrit less than 30%, initial systolic blood pressure lower than 100 mm Hg, red blood in the NG lavage, a history of cirrhosis or ascites on examination, and a history of vomiting red blood. 63% of users answered correctly.

Which of the following features predicts adverse outcomes in patients with upper GI hemorrhage? A A history of cirrhosis or ascites on examination B A history of melena C Coffee-ground emesis in the nasogastric lavage D Initial hematocrit less than 40% E Initial systolic blood pressure less than 110 mm Hg

The correct answer is D. Leptospirosis and freshwater Explanation: Obtaining a detailed travel history is tantamount to deciphering disease in a returning traveler. Key aspects are destination, adherence to chemoprophylaxis, immunization history, rural or remote travel, eating uncooked meats or vegetables, freshwater exposure, and insect bites. Leptospirosis, also known as mud fever, occurs after exposure to freshwater and has been seen in adventure travelers and triathletes. Patients abruptly develop high fever, myalgias, hepatitis, and often a subconjunctival injection. This can progress to meningitis, renal failure, and respiratory failure. Amebiasis is a protozoal disease transmitted by the fecal-oral route. Complications include dysentery and invasive extraintestinal disease, most commonly amebic liver abscess. Chagas disease is transmitted through the bite of the reduviid bug. After biting, it deposits its feces nearby, and the victim autoinoculates after rubbing the wound. Cysticercosis is a widespread global disease that is transmitted through undercooked pork. It can infest almost any tissue and is the leading cause of adult-onset seizures worldwide. In schistosomiasis, a freshwater fluke penetrates intact skin and begins a complex life cycle that can lead to both localized and systemic disease. 46% of users answered correctly.

Which of the following infections that may be seen in a returning traveler is paired with the correct exposure? A Amebiasis and percutaneous inoculation through intact skin B Chagas disease and tick bite C Cysticercosis and uncooked beef D Leptospirosis and freshwater E Schistosomiasis and uncooked pork

The correct answer is C. Fewer than 10% of patients are at risk of prescription medication abuse. Explanation: Addiction is the misuse of a drug or medication to the patient's detriment and can occur after as little as one use of heroin. Dependence usually requires daily use of a narcotic for 4-6 weeks and is confirmed when the abrupt cessation of the medication results in withdrawal. At most, 10% of patients are at risk of prescription medication abuse.

Which of the following is CORRECT regarding addiction, abuse, and dependence? A Addiction is confirmed when stopping a medication results in acute withdrawal symptoms. B Addiction usually requires daily use of narcotics for several weeks. C Fewer than 10% of patients are at risk of prescription medication abuse. D Dependence is the misuse of a medication or drug to the detriment of a patient's well-being. E Dependence may occur after one use of heroin.

The correct answer is D. Fern pattern burn Explanation: High-voltage DC electricity can throw the victim from the electrical source causing blunt traumatic injuries. High-voltage AC injury can result in tetanic contractions where the victim cannot let go of the source of injury. High-voltage injury is associated with deep tissue destruction and may cause compartment syndrome. Any type of electrical injury may cause cardiac or respiratory arrest. Fern pattern burns, also known as Lichtenberg figures, are seen with lightning injuries. These are not true burns and resolve quickly. 44% of users answered correctly.

Which of the following is NOT a complication of high-voltage electric injury? A Blunt trauma from being hurled from the source B Cardiac or respiratory arrest C Compartment syndrome D Fern pattern burn E Tetanic contractions

The correct answer is D. Milk allergy Explanation: Both egg and soy allergies are contraindications to the use of propofol. Milk allergy is not a contraindication. Hypovolemia increases the risk of hypotension from propofol use and should be corrected before the medicine is given. Additionally, sudden respiratory depression and apnea, are adverse effects seen with this drug, so ventilatory support must be feasible if this medication is used.

Which of the following is NOT considered a contraindication to the use of propofol as a sedative in the emergency room for procedural sedation? A Egg allergy B Hypovolemia C Lack of ability to intubate and ventilate the patient D Milk allergy E Soy allergy

The correct answer is A. A short burst of oral prednisone will hasten resolution of this condition. Explanation: While oral steroids may be indicated in severe cases of allergic contact dermatitis (poison ivy as displayed in the photo), a short burst will likely precipitate worsening of the patient's symptoms upon withdrawal of the steroid. Therefore, 2-3-week tapered dosing is required to attempt to avoid this rebound effect from systemic steroid therapy. Intermediate or low potency topical steroids can be used for symptomatic management. The rule of 9s, which is typically applied to burn patients, can be used to estimate the amount of topical steroids required, and dosing will be less frequent if fluorinated steroid preparations are prescribed. Caution and close monitoring is advised when prescribing steroids to immunosuppressed or diabetic patients. Related Topics:dermatitis, allergic contact 33% of users answered correctly.

Which of the following is NOT correct regarding the prescription of steroids for this eruption (Figure 3-11)? A A short burst of oral prednisone will hasten resolution of this condition. B Fluorinated topical steroids provide increased potency, therefore do not have to be applied as frequently or for as long duration. C If this patient is diabetic or immunocompromised, caution should be used when prescribing oral steroids. D One treatment for an eruption like this on the hand would be intermediate or low potency topical steroids. E The burn rule of 9s can be used to estimate the amount of topical steroid required in a prescription.

The correct answer is D. Pain and/or itching at the site of this lesion are characteristic. Explanation: The photo demonstrates a classic presentation of erythema migrans, a stage 1 dermatologic finding associated with Lyme borreliosis. IgM antibodies to this infection are developed prior to IgG, with a peak between 3 and 6 weeks after initial exposure. IgG antibodies can take months to develop and infer previous infection, rather than active acute infection. The erythema migrans rash typically presents as a macule or papule initially, which develops an expanding annular lesion with central clearing. It is not typically pruritic or painful, but a burning sensation is possible. Most of these lesions fade within 1 month of presentation. The spirochete causing this infection can spread to multiple organ systems hematogenously, and has a predilection for the skin, nervous system, and joints, causing the typical clinical findings in stages 2 and 3 of the disease. Related Topics: erythema chronicum migrans 50% of users answered correctly.

Which of the following is NOT correct regarding the tick-borne illness demonstrated in Figure 3-20? A IgG antibodies develop months after infection. B IgM antibodies peak between the third and sixth week of infection. C Most lesions fade within 1 month of presentation. D Pain and/or itching at the site of this lesion are characteristic. E Skin, nervous system, and joints are commonly affected by this infection.

The correct answer is D. Prophylactic systemic antibiotics and tetanus toxoid for a 3% BSA full-thickness burn to the arm in a 45-year-old woman with no known medical problems Explanation: Prophylactic systemic antibiotics are not indicated for any burns. Tetanus prophylaxis is indicated when the patient's tetanus status is not up to date or is unknown. Topical antibiotics such as bacitracin, 1% silversulfadiazine cream and triple antibiotic ointment are appropriate dressings for minor burns. The Parkland formula and others may be used to calculate fluid requirements in adults and children with major burns, but these are only guides. Maintaining urine output at 1 mL/kg/hr is a reasonable goal for fluid administration in children. Burn patients may require high doses of opiates to achieve adequate analgesia. Patients with full-thickness burns, those with associated inhalation injury, children, elderly, and those with underlying medical problems that could complicate management are all criteria for referral to a burn center. 54% of users answered correctly.

Which of the following is NOT correct treatment for the burn patient? A Fluids to maintain urine output at 1 mL/kg/hr for a child with 50% body surface area (BSA) partial thickness burns B High doses of intravenous morphine for a 20% BSA partial thickness burn in a 30-year-old healthy man C Referral to a burn center for an 18% BSA partial thickness burn in a 69-year-old diabetic man D Prophylactic systemic antibiotics and tetanus toxoid for a 3% BSA full-thickness burn to the arm in a 45-year-old woman with no known medical problems E Topical bacitracin ointment and 24-hour recheck for a 5% BSA partial-thickness burn to leg of a 16-year-old boy

The correct answer is D. It occurs when ringworm larvae migrate into exposed epidermis. Explanation: The photo demonstrates the clinical presentation of cutaneous larva migrans, which is an infection caused by a hookworm. Ringworm is a fungal infection of candidal origin, which is not associated with these clinical findings. Cutaneous larva migrans does occur in tropical and subtropical climates worldwide. It typically presents with a pruritic, erythematous serpiginous tract that marks the migration of the larva in the skin. The main portal of entry is exposed feet, which contact feces infested with the larva. It can cause an otherwise asymptomatic eosinophilia on peripheral blood count. One dose of albendazole or ivermectin is adequate for treatment and eradication of the larva. Related Topics: larva migrans, cutaneous 47% of users answered correctly.

Which of the following is NOT true about this disease (Figure 3-17)? A A complete blood count may demonstrate peripheral eosinophilia. B A single dose of oral albendazole or ivermectin is effective treatment. C Erythematous serpiginous tracts are formed by the causative organism. D It occurs when ringworm larvae migrate into exposed epidermis. E It is commonly found in subtropical or tropical climates.

The correct answer is C. Lesions are symmetrical with a predilection for the flexor surfaces. Explanation: This picture shows a lesion caused by psoriasis. The lesions are typically silver and scaly with erythematous plaques and papules. The areas affected may include scalp, nails, sacrum, gluteal cleft, umbilicus, and typically symmetrical extensor surfaces. The disease is common and has a strong hereditary predisposition. The peak of incidence occurs when people are in their early 20s. It has an insidious onset. Flares can occur with Streptococcal infections, severe sunburns, AIDS, as well as with multiple drugs including beta-blockers, lithium, steroid withdrawal, and antimalarials medications. Related Topics: psoriasis 56% of users answered correctly.

Which of the following is NOT true of the pictured disease (Figure 3-1)? A Age of peak incidence occurs in patients in their early 20s. B It has a hereditary predisposition. C Lesions are symmetrical with a predilection for the flexor surfaces. D Stress or alcohol ingestion can be associated with a flare. E The disease is common affecting 1.5-2% of the population.

The correct answer is D. Paradoxical agitation is seen in some patients. Explanation: Flumazenil will reverse the effects of midazolam; naloxone reverses the effects of opioids. Intranasal midazolam does cause irritation of the mucosa, but buffering the solution does not decrease this irritation. Both the rectal and IM routes have unreliable onset and depth of sedation but may be appropriate in some patients. Hypotension can arise with the use of midazolam in patients who are hypovolemic. Paradoxical agitation has been reported in 1-15% of patients who receive this medication for procedural sedation.

Which of the following is TRUE about midazolam? A Buffering it will reduce the mucosal irritation with intranasal use. B Hypertension can be a side effect in some patients. C Naloxone will reverse the effects of this drug. D Paradoxical agitation is seen in some patients. E Rectal and intramuscular administration are the preferred methods of administration.

The correct answer is D. Fluid and electrolyte imbalances and secondary infections are the most common cause of death. Explanation: Stevens-Johnson syndrome (SJS) is characterized by dermatologic and mucous membrane involvement. Patients complain of malaise, fever, myalgias, and arthralgias. Skin eruptions typically involve epidermal detachment of less than 30%, no greater than 50% of epidermal surfaces. Ophthalmic involvement can occur in almost 70% of cases. Patients with ophthalmic involvement should be monitored by an ophthalmologist. Precipitants of SJS include infections such as mycoplasma and HSV and medications; however, the cause is unknown in 50% of cases. The most frequent complications of SJS and most common cause of death include fluid and electrolyte imbalances and secondary infections. Patients rarely are left with scars unless they develop a superinfection. Be aware that any patient with ophthalmic involvement should be monitored by an ophthalmologist. Related Topics: stevens-johnson syndrome 72% of users answered correctly.

Which of the following is TRUE concerning this condition (Figure 3-12)? A Blindness is the most common complication. B Etiology of this condition is unknown. C Scaring is typical. D Fluid and electrolyte imbalances and secondary infections are the most common cause of death. E Symptoms in addition to the rash include diffuse arthralgias.

The correct answer is B. Iatrogenic perforation is the most frequent etiology. Explanation: It is important to distinguish between iatrogenic and spontaneous causes of esophageal perforation. Iatrogenic perforation from esophageal instrumentation accounts for the majority of cases. Spontaneous perforations can be due to Boerhaave syndrome, caustic ingestions, and pill esophagitis among others. Boerhaave syndrome is a full-thickness perforation of the esophagus due to a sudden rise in intraesophageal pressure such as in forceful emesis or coughing. It accounts for 10-15% of perforations. Most spontaneous perforations occur through the left posterolateral wall of the distal esophagus. Distal perforations tend to be more severe as they can leak esophageal contents into the mediastinum, pleural, or peritoneal spaces. This can result in fulminant necrotizing mediastinitis, pneumonitis, or peritonitis. The absence of mediastinal emphysema does not rule out esophageal perforation. Related Topics:esophageal perforation 66% of users answered correctly.

Which of the following is TRUE regarding esophageal perforation? A Boerhaave syndrome is responsible for more than 30% of esophageal perforations. B Iatrogenic perforation is the most frequent etiology. C Most iatrogenic perforations occur through the left posterolateral wall of the distal esophagus. D Proximal perforations tend to be more severe than distal perforations. E The absence of mediastinal emphysema rules out esophageal perforation.

The correct answer is C. Symptom-targeted use of anxiolytics and antiemetics in addition to opioid analgesics is generally appropriate. Explanation: It is recommended that the addition of anxiolytics and antiemetics to opioids should be symptom targeted. Adjuncts can be used as they do enhance the analgesic effect and reduce the amount of opioid required, but since appropriate titration with opioids in the emergency department is highly effective, they should not be routinely given in this setting. When treating patients in the emergency department for acute pain, it is best to remove delayed-release narcotic patches to better titrate opioid dose and to minimize adverse reactions from combining agents. Cross sensitivity has been suggested among the piperidine class of opioids, which includes fentanyl, alfentanil, sufentanil, and meperidine; if a patient reports an allergy to one of these medications, it would be better to try a medication in a different class instead of one of these. Opioid agonist-antagonist agents may precipitate withdrawal symptoms and should be used with caution in patients who are addicted to narcotics.

Which of the following is TRUE regarding opioid use in the emergency department? A Fentanyl (Sublimaze) is a good alternative medication in a patient with an allergy to meperidine (Demerol). B A patient on chronic high-dose oxycodone hydrochloride can be safely switched to an opioid agonist-antagonist. C Symptom-targeted use of anxiolytics and antiemetics in addition to opioid analgesics is generally appropriate. D When giving opioids, promethazine or hydroxyzine should routinely be added to enhance analgesic effects and to reduce the amount of opioid required. E When treating acute pain in the emergency department, any existing transdermal opioid patches should be left in place to provide ongoing levels of medication.

The correct answer is C. Pain at a percussed tooth with mild gingival edema with scant expressible pus and a panorex showing slight widening of the periodontal ligament space is best treated with oral antibiotics and referral for root canal or extraction. Explanation: Acute periradicular periodontitis infection extends to the apex of the tooth and is best treated with oral antibiotics, analgesics, and prompt root canal or tooth extraction. Episodes of cold sensitivity lasting seconds at a time recurring several times a day over greater than 1 week is a sign of reversible pulpitis and should be treated with antibiotics and pain meds and referred to a dentist. If the duration of symptoms last longer than minutes or hours at a time, the likelihood for irreversible pulpitis is high and usually best treated with root canal or extraction. Pain over an eruption wisdom tooth known as pericoronitis is usually secondary to severe inflammation from impacted food under the gingival flap and can progressed to masticator space tissue infections and should be treated with saline irrigation, oral antibiotics, and pain medications. Patient with complaint of dental pain and a normal exam should be worked up for sources of referred pain. Exposed dentin is best treated with emergent cover with glass ionomer to prevent pulpitis, pain medications, and urgent dental referral. 44% of users answered correctly.

Which of the following is TRUE? A Episodes of cold sensitivity lasting seconds at a time and recurring several times a day over greater than 1 week is a sign of irreversible pulpitis and should be referred for root canal within 48 hours. B Pain over an erupting wisdom tooth is pericoronitis and is treated with saline irrigation. C Pain at a percussed tooth with mild gingival edema with scant expressible pus and a panorex showing slight widening of the periodontal ligament space is best treated with oral antibiotics and referral for root canal or extraction. D Episodes of paroxysmal tooth pain and a normal exam should be considered drug seeking behavior and discharge only with nonnarcotic pain medication. E Pain and cold sensitivity after biting a hard kernel showing chip fracture with exposed dentin is best treated with pain medication, antibiotics and 24 hour dental follow-up.

The correct answer is B. Diuretic therapy Explanation: Frequent causes of hypokalemia in ED patients are alkalosis, gastrointestinal loss (vomiting, diarrhea, nasogastric suctioning, etc.), and diuretic therapy. Other less common causes are from sweat loss, other causes of renal loss (primary or secondary aldosteronism, renal tubular acidosis, postobstructive diuresis, licorice ingestion, and osmotic diuresis), extracellular to intracellular potassium shifts such as hypokalemic periodic paralysis, and increased plasma insulin. Symptoms of hypokalemia such as hypertension, orthostatic hypotension, dysrhythmias, weakness, cramps, paralysis, and hyporeflexia usually present when potassium levels are below 2.5 mEq/L. Potassium should be replaced either orally with 20 mEq of K+ every 60 minutes until anticipated results are achieved, or through a peripheral IV. No more than 40 mEq should be added to each liter of IV fluid and infusion rates should be no greater than 20 mEq/hour. 82% of users answered correctly.

Which of the following is a frequent cause of hypokalemia in ED patients? A β-Adrenergic therapy B Diuretic therapy C Lithium therapy D Poor dietary intake E Primary aldosteronism

The correct answer is D. Progressive pain and swelling Explanation: CroFab™ is indicated in any rattlesnake victim with progressive signs and symptoms, evidence of systemic toxicity such as hypotension or altered mental status (AMS), serious neurological toxicity (e.g., limb paralysis), or serious hematologic toxicity such as elevated PT or falling (not elevated) platelet count. 76% of users answered correctly.

Which of the following is an indication for polyvalent Crotalidae Immune Fab (CroFab™) administration after a rattlesnake bite? A Elevated platelets B Hypertension C Localized pain and redness D Progressive pain and swelling E Normal prothrombin time (PT)

The correct answer is E. Paralysis Explanation: Paralysis and anesthesia are late findings of ischemic injury that portend loss of limb viability. An adequate collateral blood supply may mask signs of ischemia in a severely diseased artery. Pain can be the sole and earliest symptom of ischemia in a patient with underlying peripheral vascular disease. Peripheral nerves and muscles are very sensitive to ischemia, which causes early findings of hypesthesia, hyperesthesia, and muscle weakness. Skin changes associated with complete arterial obstruction include pallor, cyanosis, petechiae or blisters, and decreased temperature. Ultimately, skin and fat necrosis ensue. 71% of users answered correctly.

Which of the following is considered a late finding of complete arterial obstruction? A Hypoesthesia or hyperesthesia B Muscle weakness C Pain D Pallor E Paralysis

The correct answer is C. Multiple bilateral pulmonary infiltrates Explanation: Patients with right-sided endocarditis often have septic pulmonary emboli. They can present with chest pain, hemoptysis, and dyspnea. Right-sided endocarditis is more likely in injection drug users infected with S. aureus, malformed right-sided valves, prosthetic right-sided valves, and bilateral pulmonary infiltrates suggesting septic emboli. Streptococcus and enterococcus both have a predilection for left-sided endocarditis even in injection drug users. Pulmonary edema suggests a left-sided lesion, as do all other embolic phenomena, unless the patient has a large septal defect. 34% of users answered correctly.

Which of the following is likely to have right-sided endocarditis? A Blood cultures positive for streptococci in an injection drug user B Native-valve disease in a noninjection drug user C Multiple bilateral pulmonary infiltrates D New left bundle-branch block E Pulmonary edema

The correct answer is C. Submandibular gland stones are best treated with massage and lemon drops. Explanation: Submandibular gland stones are best treated with analgesics, massage, sialogogues, and antibiotics if superinfection is suspected. Eighty percent of sialoliths occur in the submandibular gland (Wharton's duct) because of its more viscous secretions and uphill course. Suppurative parotitis occurs in states of dehydration, and can be brought on by medications that dehydrate of decrease salivary flow such as antihistamines or tricyclic antidepressants. It occurs in patients with compromised salivary flow and retrograde migration of bacteria into the duct; the treatment should involve antibiotics that cover staphylococcus and anaerobic bacteria, but also maneuvers that stimulate salivary flow, such as hydration, massage, applied heat, and stimulatory sialogogues such as lemon drops. In contrast to viral infection, the onset is rapid and without prodrome and shows erythema and warmth usually absent in viral parotitis. Viral parotitis is usually benign in children but can be severe in adults. Treatment is supportive. 41% of users answered correctly.

Which of the following is most accurate about salivary gland enlargement? A 80% of sialoliths occur in the parotid gland (Stenson's duct) because of its more viscous secretions. B Mumps is most commonly caused by paramyxovirus, follows a viremic prodrome, can be associated with orchitis, and is more severe in children than in adults. C Submandibular gland stones are best treated with massage and lemon drops. D Suppurative parotitis is not affected by the use of antihistamines and tricyclic antidepressants. E Suppurative parotitis should be treated by minimal stimulus to the gland.

The correct answer is D. Transitory wide anion gap Explanation: Distinguishing grand-mal seizures from true syncope is essential yet very challenging. Historical features such a classic aura, prolonged postictal confusion, witnessed head turning, unusual posture or automatisms, and muscle pain suggest seizure. Patients with syncope may have premonitory symptoms of nausea or light headedness if the event is reflex mediated (vasovagal), but a dysrhythmia may occur without prodrome. Brief tonic-clonic movements can occur with syncope yet be mistaken for seizure. Trauma with or without defensive injuries to the hands or knees suggests a sudden event without warning, which could occur with dysrhythmia, noncardiac syncope, or seizure, so trauma is not a distinguishing feature. Urinary incontinence could occur with seizure or syncope. A transitory wide anion gap occurs after a grand-mal seizure but not after an uncomplicated syncopal event. 47% of users answered correctly.

Which of the following is most useful in distinguishing a grand-mal seizure from syncope? A Orofacial trauma B Prodromal symptoms C Tonic-clonic movements D Transitory wide anion gap E Urinary incontinence

The correct answer is B. NSAIDs are an appropriate long-term treatment for chronic low back pain in patients at low risk of side effects. Explanation: Administration of steroids for acute sciatica significantly reduces pain scores, but the benefit appears to last only about 24 hours. A systematic meta-analysis of opioids for chronic back pain found no significant long-term benefit. Despite some lack of long-term studies, it is considered reasonable to use NSAIDs in patients with chronic back pain provided that side effects are anticipated when the decision is made to prescribe them. Cyclobenzaprine (Flexeril) appears to have little benefit beyond that of placebo in chronic back pain. Tricyclic antidepressants have been shown to improve mild to moderate chronic back pain.

Which of the following is the BEST answer regarding medication for back pain and sciatica? A Cyclobenzaprine (Flexeril) is an appropriate long-term medication for chronic back pain. B NSAIDs are an appropriate long-term treatment for chronic low back pain in patients at low risk of side effects. C Opioids are appropriate long-term agents to use for chronic back pain. D Steroids are an effective and long-lasting treatment for acute sciatica. E Tricyclic antidepressants are NOT appropriate long-term medications for use in chronic back pain.

The correct answer is C. Kerley lines Explanation: Kerley lines are thickened interstitial septal markings that can represent pulmonary edema, as well as other interstitial processes. Kerley B lines are short parallel lines that extend from the periphery and intersect the pleura. The frequency of left-side heart failure radiographic findings are, in decreasing order, dilated upper lobe vessels, cardiomegaly, interstitial edema, enlarged pulmonary artery, pleural effusion, alveolar edema, prominent superior vena cava, and Kerley lines. 38% of users answered correctly.

Which of the following is the LEAST frequently seen radiographic finding of left-sided heart failure? A Alveolar edema B Enlarged pulmonary artery C Kerley lines D Pleural effusion E Prominent superior vena cava

The correct answer is D. Mitral valve prolapse Explanation: Mitral valve prolapse affects about 2.4% of the population in the industrialized world. Rheumatic heart disease is uncommon in the United States but is a major cause of aortic valve disease worldwide. Calcific aortic stenosis is the most common cause of adult aortic stenosis in the United States, while younger adults have a bicuspid aortic valve or underlying congenital heart disease. 49% of users answered correctly.

Which of the following is the MOST common valvular heart disease in the industrialized world? A Aortic insufficiency B Bicuspid aortic stenosis C Calcific aortic stenosis D Mitral valve prolapse E Tricuspid insufficiency

The correct answer is D. A resident who was stuck with a hollow-bore needle after performing a thoracentesis on a patient with disseminated M. avium complex Explanation: Basis for treatment and the regimen depends upon the mechanism of exposure and the disease state of the source. Risks for seroconversion include deep injury, visible blood on the device, previous needle placement in a vein or an artery, and a source with late-stage HIV infection or high viral load. Specific advice is available through a national hotline. Low-risk exposure to an unknown source does not support treatment. Two-drug PEP is utilized in low-risk exposure to low-risk patients (those with low viral loads and asymptomatic HIV), and three-drug PEP is advised with high-risk exposure to low-risk patients and any significant exposure to a high-risk source (end-stage AIDS or high viral load). Unless saliva or vomit contains blood, each is unlikely to cause transmission. Blood, semen, and vaginal secretions, along with pleural, peritoneal, and amniotic fluid, all can harbor and transmit HIV. Nonoccupational PEP is traditionally recommended only for high-risk exposure within the previous 72 hours. Treatment outside 72 hours is at the discretion of the physician. 67% of users answered correctly.

Which of the following is the highest risk exposure and supports three-drug postexposure prophylaxis (PEP) against HIV? A A police officer who was spit in the eyes by an otherwise healthy male who is refusing testing B A male who tested negative for HIV 6 months ago who last had anal intercourse 5 days ago and is worried the condom broke C A nurse who was vomited in the face by an HIV patient with CD4 count of 600 D A resident who was stuck with a hollow-bore needle after performing a thoracentesis on a patient with disseminated M. avium complex E A gynecology resident who was splashed in the eyes with amniotic fluid from a woman with a known low viral load of HIV

The correct answer is D. Idiopathic Explanation: Most cases of restrictive cardiomyopathy are idiopathic, often with an autosomal-dominant familial pattern. The remainder of causes are systemic disorders such as those listed earlier, as well as progressive systemic sclerosis (scleroderma), and hypereosinophilic syndrome. All etiologies affect the heart muscle, either by scarring or infiltration, and cause stiffness that affect diastolic filling and raises end-diastolic pressures. Systolic LV function is normal. The EP should consider restrictive cardiomyopathy in a patient presenting with signs of CHF but without cardiomegaly or systolic dysfunction.

Which of the following is the most common cause of restrictive cardiomyopathy? A Amyloidosis B Endomyocardial fibrosis C Hemachromatosis D Idiopathic E Sarcoidosis

The correct answer is D. Idiopathic Explanation: Most cases of restrictive cardiomyopathy are idiopathic, often with an autosomal-dominant familial pattern. The remainder of causes are systemic disorders such as those listed earlier, as well as progressive systemic sclerosis (scleroderma), and hypereosinophilic syndrome. All etiologies affect the heart muscle, either by scarring or infiltration, and cause stiffness that affect diastolic filling and raises end-diastolic pressures. Systolic LV function is normal. The EP should consider restrictive cardiomyopathy in a patient presenting with signs of CHF but without cardiomegaly or systolic dysfunction. 58% of users answered correctly.

Which of the following is the most common cause of restrictive cardiomyopathy? A Amyloidosis B Endomyocardial fibrosis C Hemachromatosis D Idiopathic E Sarcoidosis

The correct answer is D. Petechiae Explanation: Epistaxis, gingival bleeding, hematochezia, and subconjunctival hemorrhage may be seen in a patient with idiopathic thrombocytopenic purpura. Petechiae, however, are the most common physical finding. 73% of users answered correctly.

Which of the following is the most common sign of idiopathic thrombocytopenic purpura? A Epistaxis B Gingival bleeding C Hematochezia D Petechiae E Subconjunctival hemorrhage

The correct answer is B. Dilated upper lobe vessels Explanation: Normally, vascularity in the lower lung fields is more prominent than in the upper lung fields. When this situation is reversed, dilated upper lobe vessels create a pattern called cephalization. The frequency of left-side heart failure radiographic findings are, in decreasing order, dilated upper lobe vessels, cardiomegaly, interstitial edema, enlarged pulmonary artery, pleural effusion, alveolar edema, prominent superior vena cava, and Kerley lines. Pleural effusions can be difficult to detect on a supine chest x-ray. In one study of patients with pleural effusions, the sensitivity of a supine chest x-ray for heart failure was 67% and specificity was 70%. 52% of users answered correctly.

Which of the following is the most frequent radiographic finding of left-sided heart failure? A Cardiomegaly B Dilated upper lobe vessels C Enlarged pulmonary artery D Interstitial edema E Pleural effusion

The correct answer is D. Chemical burn to the eye Explanation: Full vital signs, including visual acuity, the vital sign of the eye, and a full eye exam should be done before initiating treatment except for chemical exposure to eye, in which high volume irrigation must be initiated as soon as possible. A few drop of topical anesthetic are often indicated prior to the full exam in cases of acute eye pain, but a full exam including visual acuity, field of vision testing, pupillary exam, slit lamp, and fundoscopic exam should be done. Blunt trauma may require eye speculum for examination if the lids are swollen shut. 80% of users answered correctly.

Which of the following is the one opthalmologic emergency in which measurement of visual acuity and eye exam should not precede treatment? A Acute monocular visual loss B Acute red eye with photophobia C Blunt trauma to the eye D Chemical burn to the eye E Foreign body to the eye using high-powered tool

The correct answer is D. Sumatriptan succinate (Imitrex) Explanation: Preferred treatment for migraine headaches includes serotonin agonists (triptans such as Imitrex [sumatriptan succinate]) or dopamine antagonists (phenothiazines such as Compazine [prochlorperazine]) rather than opiates. In multiple trials, opiates have demonstrated poor performance.

Which of the following is the recommended treatment for migraines? A Fentanyl (Sublimaze) B Hydrocodone and acetaminophen (Vicodin) C Meperidine hydrochloride (Demerol) D Sumatriptan succinate (Imitrex) E Morphine sulfate

The correct answer is C. Rapid treatment shortens healing time. Explanation: The pictures show Ramsey Hunt syndrome in an individual with the characteristic lesions of herpes zoster. Ramsey Hunt syndrome occurs when there is involvement of the facial and auditory nerve from the reactivation of the varicella-zoster virus. Pain or dysesthesia will precede the rash by 3-5 days. Lesions on the auditory canal and tympanic membrane along with a Bell's palsy, deafness, or vertigo may suggest the diagnosis. Treatment includes oral acyclovir or valacyclovir for 1 week. HIV patients should only receive acyclovir due to the risk of hemolytic uremic syndrome with valacyclovir. Treatment should be initiated within 72 hours to shorten healing time, decreases new lesion formation, and prevents postherpetic neuralgia. Related Topics: herpes zoster oticus 54% of users answered correctly.

Which of the following is true concerning the condition involving this patient's right ear (Figure 3-9)? A Methicillin-resistant S. aureus causes this infection. B Oral valacyclovir is the treatment of choice for patients with HIV. C Rapid treatment shortens healing time. D Rash is typically followed by pain. E Treatment is indefinite for patients with this condition.

The correct answer is B. Immunocompromised hosts may develop crusted plaques on the scalp, hands, and feet. Explanation: The figure shows the characteristic burrows of a scabies infestation. The rash occurs approximately 30 days after exposure to the mite, Sarcoptes scabei. The rash will typically be more pruritic at night. The distribution of the lesions is different between adult, children, and immunocompromised hosts. Adults typically have lesions on the feet, ankles, wrist, hands, axilla, groin, and extensor surfaces of the extremities. The lesions spare the head and neck. Children have a more generalized distribution that includes the face, scalp, and neck, as well as the diaper area. Immunocompromised hosts may have crusted, confluent plaques that is termed "crusted" or Norwegian scabies. Treatment includes topical Permethrin 5% cream, Lindane shampoo, as well as oral ivermectin. Lindane and oral ivermectin are not safe to treat pregnant women. Related Topics: scabies 48% of users answered correctly.

Which of the following is true concerning this intensely pruritic rash (Figure 3-10)? A Children typically present with a milder rash. B Immunocompromised hosts may develop crusted plaques on the scalp, hands, and feet. C Pruritus is most notable upon awakening. D Rash usually spares the head and neck in children. E Treatment in pregnant women is limited to oral ivermectin.

The correct answer is E. Rapid sequence medications, including paralytics, cross the placenta. Explanation: Because pregnant women are at increased risk for aspiration and more prone to hypoxia due to decreased oxygen reserves, early intubation is recommended in the critically ill or seriously injured pregnant woman. Rapid sequence intubation is the first-line method used. These medications do cross the placenta, but are generally well tolerated by the fetus. Weight gain and fluid retention are two reasons the intubation can be more technically difficult in a pregnant patient. Progesterone stimulates an increased respiratory drive in pregnancy producing a respiratory alkalosis. Functional residual volume is decreased because of a rise in the level of the diaphragm. 60% of users answered correctly.

Which of the following is true regarding airway management in pregnancy? A Aspiration risk is decreased in pregnancy. B A respiratory acidosis is normal in pregnancy. C Because of difficult airway anatomy, intubation should be avoided if possible. D Functional residual volume remains unchanged. E Rapid sequence medications, including paralytics, cross the placenta.

The correct answer is D. Metoprolol Explanation: The photo demonstrates a photosensitivity reaction. This occurs most commonly with phototoxic medications, including tetracycline, doxycycline, furosemide, amiodarone, fluoroquinolones, and chlorpromazine. Metoprolol does not exhibit phototoxicity. Related Topics: dermatitis, phototoxic 62% of users answered correctly.

Which of the following medications does NOT typically cause the reaction shown in Figure 3-19? A Amiodarone B Chlorpromazine C Furosemide D Metoprolol E Tetracycline

The correct answer is A. Acetazolamide Explanation: Acetazolamide is the drug of choice for treating acute mountain sickness. Corticosteroids are effective, but are generally reserved for severe cases of acute mountain sickness or high-altitude cerebral edema. Sildenafil and nifedipine can be used for prophylaxis of high-altitude pulmonary edema due to their ability to lower pulmonary artery pressure. There is conflicting evidence on the use of Ginkgo biloba for prevention of acute mountain sickness and no studies have evaluated its used for treatment. 78% of users answered correctly. Acetazolamide belongs to a class of drugs known as carbonic anhydrase inhibitors. It works by decreasing the production of fluid inside the eye. It is also used to decrease a buildup of body fluids (edema) caused by heart failure or certain medications. Acetazolamide, sold under the trade name Diamox among others, is a medication used to treat glaucoma, epilepsy, altitude sickness, periodic paralysis, idiopathic intracranial hypertension (raised brain pressure of unclear cause), and heart failure.

Which of the following medications is the best choice for treating mild to moderate acute mountain sickness? A Acetazolamide B Dexamethasone C Ginkgo biloba D Nifedipine E Sildenafil

The correct answer is E. Nonsteroidal anti-inflammatory agents Explanation: Nonsteroidal anti-inflammatory agents inhibit the effects of ACE inhibitors and diuretics and can worsen cardiac and renal function. The emergency physician (EP) should review the medication lists of heart failure patients and also avoid prescribing nonsteroidal anti-inflammatory drugs (NSAIDs) when such patients present for injuries or chronic pain. The main risk of acetaminophen, at excessive levels, is hepatotoxicity. Stable heart failure patients are discharged on ACE inhibitors and beta-blockers because these medications have been conclusively shown to decrease mortality. Angiotensin receptor blockers have fewer side effects than ACE inhibitors but can still cause cough or angioedema. 75% of users answered correctly.

Which of the following medications should be avoided in stable patients with a history of heart failure? A Acetaminophen B Angiotensin-converting enzyme (ACE) inhibitors C Angiotensin receptor blockers D Beta-blockers E Nonsteroidal anti-inflammatory agents

The correct answer is C. Rapid injection to minimize the duration of pain Explanation: There are multiple ways to reduce pain of infiltration of a local anesthetic. They include slow (not fast) injection, warming the solution to body temperature, and injecting through the margins of the wound. Bupivacaine can be buffered as described. Buffering the anesthetic solution with sodium bicarbonate will decrease the pain of injection.

Which of the following methods is NOT an appropriate way to decrease the pain of infiltration from a local anesthetic? A Buffering the anesthetic solution with sodium bicarbonate prior to infiltration B Injecting through the wound margins instead of through intact skin C Rapid injection to minimize the duration of pain D Using 1 mL of sodium bicarbonate 8.4% to buffer 29 mL of bupivacaine 0.25% E Warming the solution to body temperature before injecting

The correct answer is A. Etomidate—suppression of the adrenal-cortical axis Explanation: Etomidate is associated with suppression of the adrenal-cortical axis as well as myoclonic jerking. Ketamine is known for emergence reactions. Fentanyl has been associated with rigid chest syndrome in a small percentage of patients, especially if given as a rapid bolus. Paradoxical agitation has been reported with the use of midazolam. Nitrous oxide may promote expansion of internal gas filled structures.

Which of the following pairs correctly identifies a drug with its associated side effect? A Etomidate—suppression of the adrenal-cortical axis B Fentanyl—paradoxical agitation C Ketamine—rigid chest syndrome D Midazolam—emergence reactions E Nitrous oxide—myoclonic jerking

The correct answer is C. A 44-year-old diabetic woman with no previous vaccinations with a small 1-cm laceration while washing a kitchen knife. Explanation: TIG is indicated for nonminor wounds in a patient who had less than three tetanus toxoid doses. It is not necessary for clean, minor wounds, even with an incomplete vaccination series. TIG is typically not necessary for major wounds if the primary vaccination series has been completed. TIG may be considered in patients who should receive tetanus toxoid (Td or Tdap) based on time since last booster but who have a contraindication to toxoid administration. Contraindications include Guillain-Barré syndrome within 6 weeks of tetanus toxoid, previous Arthus reaction within 10 years, current moderate illness, or unstable neurologic condition. 32% of users answered correctly.

Which of the following patients would MOST likely benefit from tetanus immunoglobulin (TIG)? A A 30-year-old male who underwent the full vaccination series with last booster 4 years ago who has an open Grade 3 tibia fracture after a dirt bike crash. B A 44-year-old diabetic woman with no previous vaccinations with a small 1-cm laceration while washing a kitchen knife. C A 45-year-old male who completed the primary vaccinations series but developed Guillain-Barré after his last booster 15 years ago, now presenting with an open crush injury of bilateral thighs. D A 52-year-old farmer who completed the primary vaccination series but no booster in over 10 years who has a large jagged laceration to his shin due to farm equipment. E A 60-year-old male who completed the vaccination series but has not had a booster in over 10 years who lacerated his finger while cutting carrots.

The correct answer is E. The emergency physician must perform a medical screening exam prior to transfer. Explanation: With regard to the EMTALA, emergency physicians should approach pregnant patients in the same fashion as all ED patients. A screening exam must be done, and the patient must be stabilized as much as possible at the current facility. This does not necessarily mean delivery. When a patient is to be transferred, the transferring physician must certify that a facility and doctor has accepted the patient and that the benefits of transfer outweigh potential risks. If an emergency physician works in a hospital with no obstetric coverage, he or she should be familiar with protocols in place for transfer and equipment available for an emergency delivery. 59% of users answered correctly.

Which of the following regarding pregnant patients and Emergency Medical Treatment and Active Labor Act (EMTALA) is the most accurate? A An emergency physician may transfer to a facility with obstetric coverage if none is available in the hospital without prior notification of the facility. B If an emergency physician works in a hospital with no obstetric coverage, he or she should not accept ambulances with pregnant patients. C It is an EMTALA violation to transfer a pregnant patient in active labor under any circumstances. D Under EMTALA, stabilization of the patient is equivalent to delivery. E The emergency physician must perform a medical screening exam prior to transfer.

The correct answer is D. Respiratory depression Explanation: Adverse effects of opioids are more common with intravenous, transmucosal, and epidural administration. They include pruritus, urinary retention, confusion, and respiratory depression. Other side effects of opioids include nausea, vomiting, pruritus, constipation, and hypotension. Gastrointestinal bleeding is not generally considered a side effect of opioid analgesics.

Which of the following side effects is more common when an opioid analgesic is administered intravenously rather than orally? A Constipation B Gastrointestinal bleeding C Nausea D Respiratory depression E Vomiting

The correct answer is C. Orthopnea Explanation: The specificities of orthopnea and dyspnea for heart failure are, respectively, 88%, and 50%. Rales have a predictive accuracy of 70%. Peripheral edema and weight gain are more likely to occur in normotensive acute decompensated heart failure, which develops over days or weeks. Hypertensive heart failure, which occurs over 48 hours or less, is the most common acute presentation. Hypotensive acute heart failure is the least common presentation of acute decompensated heart failure. 71% of users answered correctly.

Which of the following signs or symptoms has the highest specificity in the diagnosis of acute heart failure? A Dyspnea B Edema C Orthopnea D Rales E Weight gain

The correct answer is E. Tinnitus may be heard by the examiner. Explanation: Tinnitus can be divided into objective, heard by the examiner (i.e., secondary to AV malformations, arterial bruits, mechanical abnormalities), and subjective tinnitus (i.e., secondary to cochlear/neuro sensory insult from toxins or vascular insufficiency). Pharmacological side effects are the cause of tinnitus in 10% of cases. Aspirin, followed by nonsteroidal anti-inflammatory drugs, and antibiotics such as amniogycosides are the most commonly implicated pharmacological causes of tinnitus. Antidepressants are not implicated in tinnitus; in fact, they are the only class of drug found to be useful in alleviating idiopathic tinnitus. SHL is defined as hearing loss occurring within 3 days or less, and "idiopathic" is the most common cause of SHL, though infectious causes such as mumps, genetic degenerative diseases, vascular insufficiency from a variety of causes, and toxic affects of drugs need to be considered in the differential. 35% of users answered correctly.

Which of the following statements about acute hearing loss and tinnitus is most accurate? A Antidepressants are the most commonly implicated drugs in tinnitus. B Metoprolol is the most commonly implicated drug causing tinnitus. C Sudden hearing loss (SHL) by definition is hearing loss occurring within 10 days. D Side effects of medications cause the majority of cases of tinnitus. E Tinnitus may be heard by the examiner.

The correct answer is B. In adults older than 40 years, up to 75% of neck masses persisting more than 6 weeks are malignant. Explanation: In patients older than 40 years, 75% of persistent neck masses are neoplastic. The most common cause of unilateral neck masses is squamous cell carcinoma of the upper gastrointestinal tract. They can be initially mobile and then fixed as the cancer invades surrounding tissue. Other tumors are neoplasms of the salivary gland and thyroid and lymphomas. They can become superinfected and present as abscesses. Neck masses in adolescents and young adults are usually due to infection. Midline masses suggest thyroglossal duct cysts. Single tender solitary thyroid nodules are usually benign. 47% of users answered correctly.

Which of the following statements about neck masses is most accurate? A In adults older than 40 years, the most common cause of unilateral neck masses is Hodgkin lymphoma. B In adults older than 40 years, up to 75% of neck masses persisting more than 6 weeks are malignant. C Midline masses suggest branchial cleft cysts. D Neck masses in adolescents and young adults are usually due to congenital cyst inflammation. E Single tender solitary thyroid nodules are usually malignant or premalignant.

The correct answer is A. Approximately 70% of all OM is viral. Explanation: In one study, viruses were found to be the causal agent in 70% of all OM when an organism could be cultured. When bacteria were causal, Streptococcus, Haemophilus, and Moraxella were most common. Treatment of choice for OM in patients older than 2 years is pain management with 48 hours recheck. Amoxicillin, high dose for 7-10 days, is still the initial treatment of choice with change to alternative agents if no improvement after 72 hours. Acute mastoiditis, a complication of OM, requires admission and IV antibiotics with urgent myringotomy. OM with systemic symptoms, signs of mastoiditis, and elevated intracranial pressure are best imaged with magnetic resonance imaging, which is more sensitive than CT to rule out lateral sinus thrombosis, an ominous complication of OM. 34% of users answered correctly.

Which of the following statements about otitis media (OM) is most accurate? A Approximately 70% of all OM is viral. B 70% of all OM is secondary to Streptococcus and Moraxella. C Initial treatment of choice for OM in patients older than 2 years is pain management and for patients younger than 2 years is second-generation cephalosporins. D OM with fever, posterior auricular erythema and tenderness, headache, papilledema, and sixth nerve palsy should get CT and be treated with broad-spectrum antibiotics. E OM with fever and posterior auricular erythema and tenderness should get mastoid radiographs or a CT and be treated with broad-spectrum antibiotics and close follow-up with otolaryngology

The correct answer is A. Patients with tracheostomies and respiratory distress should first be suctioned through the inner cannula with sterile saline and preoxygenation. Explanation: Tracheostomy tube obstruction with mucous plugging is common. It is best treated with preoxygenation (as suctioning may cause transient hypoxia) and then placing of sterile saline into the trachea and suctioning through large flexible catheters through the inner cannula. If this does not succeed, remove the inner cannula and clean with hydrogen peroxide and rinse with water. Sometimes, the entire tracheostomy tube needs to be removed and cleaned. Tracheostomy tubes less than 7 days in maturation should be manipulated only by otolaryngology surgeons. Pediatric tracheostomies do not have inner cannulas. 41% of users answered correctly.

Which of the following statements about the management of tracheostomy tubes is most accurate? A Patients with tracheostomies and respiratory distress should first be suctioned through the inner cannula with sterile saline and preoxygenation. B Patients with tracheostomies and respiratory distress should first have the inner cannula removed and cleaned and be suctioned through the tracheotomy tube. C Patients with tracheostomies and respiratory distress should first have the tracheostomy tube removed and be suctioned through the stoma. D Patients with tracheostomies less than 7 days old can be manipulated by an ED physician if the proper technique is used. E When pediatric tracheostomy tubes are crusted or clogged, only the inner cannula should be removed for cleaning.

The correct answer is E. The rash is intensely pruritic. Explanation: The picture shows infantile seborrheic dermatitis. Adults can have seborrheic dermatitis too. Infantile seborrheic dermatitis occurs most commonly in the first 3 months of life and is also known as "cradle cap." Fungal cultures should be done in children up to puberty to exclude tinea capitis as a cause of the lesions. The rash occurs in areas of active sebaceous glands. Certain medications may exacerbate the lesions. Common ones are buspirone, cimetidine, chlorpromazine, haloperidol, and lithium. The infantile form is nonpruritic, and in adults, it may be mildly pruritic. Seborrheic dermatitis is a chronic disorder and treatment includes 1% ketoconazole shampoo for infants and for adults may include antidandruff shampoo or ketoconazole shampoo for scalp lesions. For the face, 1-2.5% hydrocortisone is mainstay applied twice a day. Higher potency steroids should be avoided. Related Topics: seborrheic dermatitis 39% of users answered correctly.

Which of the following statements about this skin disorder is NOT true (Figure 3-8)? A Adults with AIDS and Parkinson disease are predisposed to severe forms of this. B Fungal culture should be performed in patients up to puberty. C Ketoconazole shampoo and antidandruff shampoo may be used to treat this. D M. furfur may play a role in the pathogenesis of this disease. E The rash is intensely pruritic.

The correct answer is B. Ethmoid sinusitis is the leading cause of orbital cellulites. Explanation: Acute sinusitis, like OM, is usually viral. Thus, acute sinusitis is best treated supportively with saline irrigation and decongestants. Postseptal/orbital cellulitis occurs most frequently from spread of paranasal sinusitis; the ethmoid sinus is most frequently implicated, probably due to perforation of the thin lamina papyracea of the medial wall. Meningitis, cavernous sinus thrombosis, and intracranial abscesses are rare, but important, complications of sinusitis. Amoxicillin is the recommended first-line therapy for most adults. 52% of users answered correctly.

Which of the following statements are most accurate about sinus disease? A Acute sinusitis is best treated with saline irrigation, topical decongestants, and antibiotics B Ethmoid sinusitis is the leading cause of orbital cellulites. C Macrolides and trimethoprim-sulfamethoxazole are recommended as first-line treatment. D Meningitis, cavernous sinus thrombosis, and intracranial abscesses occur in up to 15% of sinusitis. E Most acute rhinosinusitis is bacterial in origin.

The correct answer is B. Cholecystitis is the most common surgical disease in these patients. Explanation: Cholecystitis is the most common surgical disease in elderly patients with abdominal pain, followed by small bowel obstruction, perforated viscus, appendicitis, and large bowel obstruction. Fever is not a reliable marker for surgical disease. Instead the elderly patient with a serious abdominal infection may be hypothermic. Elderly patients who present to the ED with abdominal pain have a high mortality that doubles if the diagnosis is not made correctly at the time of admission. Diarrhea occurs in 31-40% of patients with mesenteric ischemia. When compared with younger patients, elderly patients present later, with more mild and vague symptom. However, despite this atypical presentation, they have more serious illness. 68% of users answered correctly.

Which of the following statements is TRUE in elderly patients with abdominal pain? A Symptoms tend to manifest sooner. B Cholecystitis is the most common surgical disease in these patients. C Fever is a reliable marker for serious abdominal infection. D It is not crucial to determine the etiology in the emergency department (ED) prior to admission. E The presence of diarrhea reliably excludes mesenteric ischemia.

The correct answer is B. Elevated serum lipase is highly sensitive and specific for the diagnosis of acute pancreatitis. Explanation: An elevated serum lipase has a sensitivity of 90% and a specificity of 93% for the diagnosis of acute pancreatitis. It is important to be aware of the limitations of laboratory testing in patients with acute abdominal pain. Laboratory information alone should not be relied upon for the diagnosis. Up to 25% of patients with mesenteric ischemia will have a normal serum lactate on presentation. A significant number of patients with acute appendicitis and/or perforation have normal WBC. Opioid analgesics do not obscure physical findings, delay diagnosis, or lead to increased morbidity/mortality in patients with undifferentiated abdominal pain and should not be withheld. The therapeutic value of nasogastric tube placement in patients with abdominal pain has not been demonstrated. Instead, the clinician should liberally administer antiemetics in patients who are actively vomiting. Related Topics:abdominal pain 76% of users answered correctly.

Which of the following statements is TRUE in patients with abdominal pain? A Early administration of opioid analgesia should be avoided as it will obscure physical findings, delay diagnosis, or lead to increased morbidity/mortality. B Elevated serum lipase is highly sensitive and specific for the diagnosis of acute pancreatitis. C Normal serum lactate rules out mesenteric ischemia. D Normal WBC reliably excludes surgical disease. E The therapeutic value of placing a nasogastric tube in patients who are actively vomiting is well established.

The correct answer is A. Eradication of infection dramatically reduces 1-year recurrence rates for duodenal and peptic ulcers. Explanation: Eradication of infection reduces 1-year recurrence rates from 35% to 2% for duodenal ulcers and from 39% to 3% for gastric ulcers. Eradication is typically done with "triple therapy" that includes a proton pump inhibitor, clarithromycin, and either amoxicillin or metronidazole. Infection is a definite risk factor for adenocarcinoma of the stomach. In addition, H. pylori is a causative agent of mucosa associated lymphoid tissue (MALT) lymphoma. Ninety-five percent of patients with duodenal ulcer and 70% of patients with gastric ulcers are infected. Only 10-20% of infected people develop peptic ulcer disease. H. pylori antigens can be detected in the stool with a sensitivity and specificity of greater than 90%. Serologic studies that detect immunoglobulin G antibodies to H. pylori have sensitivity and specificity of 76-84% and 79-90%, respectively. 69% of users answered correctly.

Which of the following statements is TRUE regarding H. pylori infection? A Eradication of infection dramatically reduces 1-year recurrence rates for duodenal and peptic ulcers. B Infection is not a risk factor for adenocarcinoma of the stomach. C Infection is present in <50% of patients with duodenal and gastric ulcers. D Most infected people will develop peptic ulcer disease. E Serologic studies are superior over stool antigens for diagnosing infection.

The correct answer is C. Earlier irrigation is associated with improved prognosis. Explanation: Immediate copious irrigation with water is the mainstay of treatment for most chemical burns. Prognosis is related to the amount of chemical involved and the duration of exposure. Thus, earlier irrigation is associated with improved outcomes. Many chemical agents causing burns are also associated with systemic toxicity, particularly acids such as hydrofluoric acid, chromic acid, and phenol. However, acetic acid, which is found in hair products and can cause extensive scalp burns, is not generally associated with systemic symptoms. Some chemical burns require treatment with specific antidotes, for example, calcium gluconate is required to detoxify the damaging fluoride ion in hydrofluoric acid burns. Chemical burns to the eye are a serious emergency and require immediate, copious irrigation and ophthalmology consult. 85% of users answered correctly.

Which of the following statements is TRUE regarding chemical burns? A Acids generally require longer irrigation times than alkalis. B Acetic acid burns may be associated with serious systemic toxicity. C Earlier irrigation is associated with improved prognosis. D The is no benefit to use of calcium gluconate over irrigation with water in hydrofluoric acid burns. E Most chemical burns to the eye can be treated with delayed outpatient ophthalmology referral.

The correct answer is E. Objects that are irregular, sharp, particularly wide, or long are at risk for becoming lodged distal to the pylorus. Explanation: Most objects continue through the GI tract uneventfully once they have passed through the pylorus. However, irregular, sharp, wide (>2.5 cm) or long (>6 cm) objects are at risk for becoming lodged distal to the pylorus. A button battery lodged in the esophagus is a true emergency as perforation can occur within 6 hours of ingestion. These patients should undergo immediate endoscopic removal. Button batteries that have passed the esophagus can be managed expectantly with 24 hours follow-up. Repeat films should be obtained in 48 hours to ensure that the battery has passed the pylorus. Endoscopic retrieval is contraindicated in body packers because of the risk of iatrogenic packet rupture. So long as there is no evidence of rupture, these packets should be allowed to traverse naturally through the GI tract with the use of whole bowel irrigation to aid the process if necessary. Intestinal perforation from ingested sharp objects that have passed distal to the stomach is common (35%). Therefore, the recommendation is to remove sharp objects by endoscopy while they are still in the stomach or duodenum. If the object passes distal to the duodenum, then daily films will be necessary to document the object's passage. If the object does not pass within 3 days, then surgical retrieval may be necessary. Related Topics:foreign body in esophagus 62% of users answered correctly.

Which of the following statements is TRUE regarding esophageal foreign bodies? A Button batteries lodged in the esophagus can be managed expectantly. B Button batteries do not need further follow-up once they have passed the esophagus. C Endoscopic retrieval is the preferred method to remove small drug packets in body packers. D Intestinal perforation from ingested sharp objects is rare once they have passed distal to the stomach. E Objects that are irregular, sharp, particularly wide, or long are at risk for becoming lodged distal to the pylorus.

The correct answer is D. Patients with severe abdominal pain, fever, and bloody stools should have stool studies sent for invasive bacterial organisms and E. histolytica. Explanation: Patients with severe pain, fever, and bloody stool are at increased risk for infection with C. jejuni, shigella, salmonella, E. coli O157:H7, and E. histolytica. The Wright stain allows for the detection of fecal leucocytes. Traditionally, the Wright stain was used to differentiate between invasive and noninvasive infectious diarrhea. However, the Wright stain has a sensitivity of 52-82% and does not adequately screen for invasive diarrhea. Recent literature does not show that treating infectious diarrhea with antibiotics leads to a prolonged Salmonella carrier state. E. histolytica, G. intestinalis, and C. parvum lead to episodes of prolonged diarrhea (>7 days). Ciprofloxacin is the antibiotic of choice in all patients with infectious diarrhea who do not have a contraindication to antimicrobial treatment (pediatric age group, allergy, pregnancy, drug interaction). Trimethoprim/sulfamethoxazole is a second-line antibiotic due to resistant organisms. Related Topics:diarrheainfectious diarrheal disease, nos 68% of users answered correctly.

Which of the following statements is TRUE regarding infectious diarrhea? A A Wright stain is a good screening test for the presence of invasive organisms. B Antibiotics should be avoided in the treatment of infectious diarrhea because they can lead to a prolonged Salmonella carrier state. C Entamoeba histolytica, Giardia intestinalis, and Cryptosporidium parvum cause short-lived, self-limited episodes of infectious diarrhea. D Patients with severe abdominal pain, fever, and bloody stools should have stool studies sent for invasive bacterial organisms and E. histolytica. E Trimethoprim/sulfamethoxazole is the treatment of choice in patients with infectious diarrhea.

The correct answer is D. Meperidine (Demerol) is metabolized to normeperidine, a neuroexcitatory compound that has a long elimination half-life. Explanation: Meperidine (Demerol) interacts with many drugs (such as monoamine oxidase inhibitors) in ways that may precipitate a serotonergic crisis. Meperidine metabolizes to normeperidine, a substance that has neuroexcitatory properties. Codeine in doses from 30 to 60 mg has a similar effect on pain as standard doses of acetaminophen or NSAIDs. Codeine produces more nausea, vomiting, and dysphoria than other opioids. Opioid hypersensitivity is not common; however, true allergic reactions are extremely rare. It is not clear if there is clinical cross sensitivity within opioid classes, but it is reasonable to switch to a drug from a different opioid class if a patient develops a hypersensitivity reaction to an opioid.

Which of the following statements is TRUE regarding opioids? A Codeine in standard doses of 30-60 mg is significantly better than standard doses of nonsteroidal anti-inflammatory drugs (NSAIDs) or acetaminophen. B Hydrocodone (Vicodin) produces more nausea, vomiting, and dysphoria than other opioids. C Hydromorphone hydrochloride (Dilaudid) can interact with many drugs to precipitate a serotonergic crisis. D Meperidine (Demerol) is metabolized to normeperidine, a neuroexcitatory compound that has a long elimination half-life. E Morphine sulfate should never be given to a patient who reports itching when given fentanyl (Sublimaze) in the past.

The correct answer is D. The majority of peptic ulcers are caused by infection with Helicobacter pylori or by the use of NSAIDs. Explanation: The majority of peptic ulcers are caused by infection with H. pylori or by the use of NSAIDs. Not all patients with dyspepsia require endoscopy. Patients with "alarm features" are at increased risk for gastric or esophageal cancer as well as other potentially serious conditions and should be referred for endoscopy. Alarm features include age >55, unexplained weight loss, early satiety, persistent vomiting, dysphagia, anemia or GI bleeding, abdominal mass, persistent anorexia, or jaundice. Emotional stress is a predisposing factor for the development of peptic ulcer disease, while diet and alcohol are not. UGI endoscopy has a sensitivity and specificity of greater than 95% for the visualization of an ulcer and is more sensitive and specific than GI barium studies. The incidence of peptic ulcer disease is decreasing in the United States. The decrease is thought to be secondary to improved standard of living, lack of H. pylori infection, and increased use of proton pump inhibitors. 78% of users answered correctly.

Which of the following statements is TRUE regarding peptic ulcer disease? A All patients with dyspepsia require endoscopy. B Diet and alcohol use are predisposing factors to the development of peptic ulcer disease, but emotional stress is not. C GI barium studies are the test of choice for diagnosing peptic ulcers. D The majority of peptic ulcers are caused by infection with Helicobacter pylori or by the use of NSAIDs. E The incidence of peptic ulcer disease is increasing in the United States.

The correct answer is C. It is associated with advanced cases of ulcerative colitis and Crohn disease. Explanation: Toxic megacolon develops in advanced cases of inflammatory and C. difficile colitis when the disease process begins to extend through all layers of the colon. The loss of muscular tone within the colon leads to dilation, localized peritonitis, and, if left untreated, eventual perforation. Most authors recommend avoiding antidiarrheal medications in patients with suspected inflammatory or C. difficile colitis because they may precipitate toxic megacolon. Related Topics:megacolon, toxic 76% of users answered correctly.

Which of the following statements is TRUE regarding this patient's diagnosis? (See Figure 7-1) A Antidiarrheal medications are good preventative agents. B Colonic perforation is not a complication. C It is associated with advanced cases of ulcerative colitis and Crohn disease. D It is not associated with C. difficile colitis. E The disease process is limited to the mucosa and submucosa.

The correct answer is A. A patient with an aortoenteric fistula will classically present with a self-limited "herald" bleed that precedes massive intestinal hemorrhage. Explanation: An aortoenteric fistula secondary to a preexisting aortic graft is an unusual, but important, cause of intestinal bleeding. Classically, these patients will have a self-limited "herald" bleed that precedes massive hemorrhage. Peptic ulcer disease, not erosive gastritis and esophagitis, is the most common cause of UGI bleeding. Bright red or maroon rectal bleeding originates from an UGI bleed 14% of the time. Concerns that nasogastric tube passage may provoke bleeding in patients with varices are unwarranted. 55% of users answered correctly.

Which of the following statements is TRUE regarding upper gastrointestinal (UGI) bleeding? A A patient with an aortoenteric fistula will classically present with a self-limited "herald" bleed that precedes massive intestinal hemorrhage. B Bright red blood or maroon rectal bleeding rules out an UGI bleed. C Erosive gastritis and esophagitis are the most common causes of UGI bleeding. D Mallory-Weiss syndrome is an UGI bleed secondary to a full-thickness esophageal tear from repeated episodes of vomiting. E The passage of a nasogastric tube in patients with varices may provoke further bleeding.

The correct answer is A. Aphthous ulcers involve the nonkeratinized surfaces of the mouth especially the labial and buccal mucosa and is best treated with topical steroids. Explanation: The location of mouth ulcerations can help differentiate the pathological entities. - Herpes simplex ulcer or herpetic gingivostomatitis involves primarily keratinized surfaces of the mouth (gingival hard palate, outer lip) and is treated with pain medications, oral hydration, and oral antivirals. - Herpangina caused commonly by Coxsackievirus causes shallow painful ulcerations primarily on nonkeratinized mucosa (soft plate, tonsillar pillars, sparing the buccal mucosa, and keratinized surfaces) and is best treated with pain medication. - Hand, foot, and mouth disease is commonly associated with Coxsackie type A16 virus and is characterized by vesicles and ulcers of all surfaces of the oral cavity surfaces and is best treated with pain medication. - Traumatic ulcers are best treated supportively with removal of the offending source (i.e., sharp tooth or ill-fitting dentures) and pain medication. 36% of users answered correctly.

Which of the following statements is TRUE with respect to oral ulcerative disease? A Aphthous ulcers involve the nonkeratinized surfaces of the mouth especially the labial and buccal mucosa and is best treated with topical steroids. B Hand, foot, and mouth disease is commonly associated with Coxsackie-type A16 virus and is characterized by petechiae on the posterior pharynx and the lips and is best treated with pain medication. C Herpangina is caused commonly by Coxsackie's group viruses, which causes painful ulcers on keratinized tongue, gingival, hard palate, and is best treated with pain medication. D Herpetic gingivostomatitis involves primarily nonkeratinized surfaces of the mouth especially the buccal mucosa and soft palate and is best treated with oral antivirals. E Traumatic ulcers are best treated with pain medication and topical steroids.

The correct answer is D. Hematochezia originates from an upper GI source 10% of the time. Explanation: Hematochezia is associated with lower GI bleeding. However 10-14% of the time hematochezia is associated with an upper GI source. Endoscopy is more accurate than arteriography or scintigraphy in localizing lower GI bleeding. Angiography requires a relatively brisk bleeding rate (at least 0.5 mL/min) to detect the site of lower GI bleeding. Technetium-labeled red cell scans are more sensitive than angiography in localizing the site of lower GI bleeding. They can localize the site of bleeding at a rate as low at 0.1 mL/min. 48% of users answered correctly.

Which of the following statements is TRUE? A Angiography requires a slow rate, lower GI bleed to detect the site of bleeding. B Endoscopy is less accurate than arteriography or scintigraphy in localizing lower GI bleeding. C Gastrointestinal bleeding leads to a drop in BUN levels. D Hematochezia originates from an upper GI source 10% of the time. E Technetium-labeled red cell scans (scintigraphy) require a relatively brisk, lower GI bleed to detect the site of bleeding.

The correct answer is D. Patients with a history of diabetes and HIV, or elderly presented with OE and trismus, cranial nerve palsies, or any systemic signs should get CT of the brain. Explanation: Any patients with a history of being immunocompromised presenting with OE, or patients with persistent OE despite 2 weeks of treatment, or any patients with associated cranial nerve palsy or fever, should get a CT to rule out malignant OE, which requires parenteral antibiotics, admission, and ENT consultation. Staphylococcus aureus and Pseudomonas aeruginosa along with Enterobacteriaceae and Proteus are the most common organism implicated in OE. Otomycosis is seen in tropical climates and in the immunocompromised. Aspergillus, and Candida, accounts for most of the cases and are best treated with systemic antifungals, and Aspergillus may require itraconazole. OE is best treated with gently irrigation with hydrogen peroxide and suctioning, acidifying agents, topical antimicrobial, and sometimes steroids. Cortisporin Otic preparations have not been approved by the Food and Drug Administration (FDA) for OE with perforations due to the theoretical concern of the neomycin (aminoglycoside) toxicity on the inner ear. However, the ENT literature recommends the suspension but not the solution, which is more easily absorbed across the round window. Ofloxacin is the only antibiotic drop approved by the FDA for OE with perforated TM. Topical ofloxacin would be the treatment of choice for OE with perforated TM in a 6-year-old boy and is the only FDA-approved medication in kids. 59% of users answered correctly.

Which of the following statements is most accurate about outer ear infections? A Candida and Aspergillus are the most common cause of otomycosis and best treated with topical antifungals. B Cortisporin solution eardrops are safe with perforated tympanic membranes (TMs). C Otitis externa (OE) is best treated with gently irrigation with hydrogen peroxide and suctioning, alkalinizing agents, topical antimicrobials, and steroids. D Patients with a history of diabetes and HIV, or elderly presented with OE and trismus, cranial nerve palsies, or any systemic signs should get CT of the brain. E Topical ofloxacin is contraindicated in OE with perforated TM in a 6-year-old child.

The correct answer is A. A physical exam suggestive of peritonsillar abscess with negative needle aspiration requires imaging to rule out abscess. Explanation: A physical exam suggestive of peritonsillar abscess with negative needle aspiration requires CT or intraoral ultrasound to rule out abscess despite the fact that the most common mimic is peritonsillar cellulitis. Needle aspiration will effectively treat 90%. The proper technique is described in answer E except the needle should penetrate no more than 1 cm because the internal carotid artery lies just lateral and posterior to the tonsil. Adjuvant single-dose steroids improves severity and duration of symptoms, given in conjunction with 10 days of broad-spectrum antibiotics. Acute tonsillectomy may be considered if general anesthesia is required for incision and drainage. 40% of users answered correctly.

Which of the following statements is most accurate about peritonsillar abscesses? A A physical exam suggestive of peritonsillar abscess with negative needle aspiration requires imaging to rule out abscess. B Acute tonsillectomy is contraindicated especially in children. C Adjuvant steroids have not been found to be helpful. D Needle aspiration effectively treats 65% of patients and often incision and drainage will be required. E The proper technique of needle aspiration is application of topical anesthesia followed by introduction of an 18-gauge needle halfway between the base of the uvula and the maxillary alveolar ridge penetrating no more than 2 cm.

The correct answer is A. Oral metronidazole is more effective than metronidazole gel for the treatment of T. vaginalis. Explanation: Because the trichomonads infect the urethra and glands, metronidazole gel is less effective than oral metronidazole and is not recommended for the treatment of trichomonas. Trichomonas is associated with increased risk of transmission of herpes, HIV, and human papillomavirus. Up to 90% of men with trichomonas are asymptomatic, but they should be treated if they have had a known exposure. Since oral fluconazole is a class C drug, topical therapy is recommended for pregnant women with vulvovaginal candidiasis. Treatment of atrophic vaginitis includes topical estrogens not progesterone creams. 65% of users answered correctly.

Which of the following statements is true regarding vulvovaginitis? A Oral metronidazole is more effective than metronidazole gel for the treatment of T. vaginalis. B Patients infected with T. vaginalis are not at increased risk for transmission of other infections including HIV and herpes. C Since most males are symptomatic with T. vaginalis, male partners do not need to be treated unless symptomatic. D Single-dose oral fluconazole is recommended for pregnant women with vulvovaginal candidiasis. E Treatment of atrophic vaginitis consists of topical progesterone creams.

The correct answer is C. Most common site for oral cancer is the posterior aspect of the tongue. Explanation: The most common site for oral cancer is the posterior aspect of the tongue and is usually nontender. Well-demarcated zone of erythema on the tongue with atrophy of filiform papillae is known as erythema migrans or geographic tongue and is a common benign finding. Leukoplakia requires biopsy especially if found on the floor of the mouth, tongue, or vermillion border but only as 2-4% of the time they show dysplastic changes. Erythroplakia, red patches that persist and cannot be attributed to other diseases, is less common than leukoplakia but more likely to be associated with malignancy. Squamous cell carcinoma of the tongue is the common form of oral malignancy.

Which of the following statements regarding oral lesions is most accurate? A Erythroplakia, red patches, that persist and cannot be attributed to other diseases are more common than leukoplakia but less likely to be associated with malignancy. B Lymphoma is the most common form of oral cancer. C Most common site for oral cancer is the posterior aspect of the tongue. D Well-demarcated zones of erythema on the tongue with atrophy of filiform papillae is known as erythroplakia and is a precancerous lesion. E White plaques that cannot be scraped off the buccal mucosa known as leukoplakia requires biopsy as 20% of the time they show dysplastic changes.

The correct answer is C. Most common site for oral cancer is the posterior aspect of tongue. Explanation: The most common site for oral cancer is the posterior aspect of the tongue and is usually nontender. Well-demarcated zone of erythema on the tongue with atrophy of filiform papillae is known as erythema migrans or geographic tongue and is a common benign finding. Leukoplakia requires biopsy especially if found on the floor of the mouthterm-152, tongue, or vermillion border but only as 2-4% of the time they show dysplastic changes. Erythroplakia, red patches that persist and cannot be attributed to other diseases, is less common than leukoplakia but more likely to be associated with malignancy. Squamous cell carcinoma of the tongue is the common form of oral malignancy. 29% of users answered correctly.

Which of the following statements regarding oral lesions is most accurate? A Erythroplakia, red patches, that persist and cannot be attributed to other diseases are more common than leukoplakia but less likely to be associated with malignancy. B Lymphoma is the most common form of oral cancer. C Most common site for oral cancer is the posterior aspect of tongue. D Well-demarcated zones of erythema on the tongue with atrophy of filiform papillae is known as erythroplakia and is a precancerous lesion. E White plaques that cannot be scraped off the buccal mucosa known as leukoplakia requires biopsy as 20% of the time they show dysplastic changes.

The correct answer is E. Subcapsular liver hematoma is a potential complication of Hemolysis, Elevated Liver Enzymes, Low Platelet Count (HELLP) syndrome. Explanation: Subcapsular liver hematoma is a potential life-threatening complication of HELLP. If the patient is hemodynamically stable, the hematoma can be diagnosed by CT scan, or, if the patient is unstable and rupture is suspected, bedside ultrasound may be helpful. Diagnostic criteria for preeclampsia include gestation greater than 20 weeks, systolic blood pressure ≥140 or diastolic ≥90 on two occasions at least 6 hours apart, and proteinuria >0.3 g in 24-hour period. Edema is no longer part of the criteria. Because this patient is only 16 weeks pregnant, she is considered to have chronic hypertension. In addition to diabetes, hypertension, multiple gestations, and obesity, nulliparity is a risk factor for preeclampsia. Magnesium should be started on pregnant women who meet the criteria for severe preeclampsia, have evidence of HELLP syndrome, or have seizures after 20 weeks of gestation or within 4 weeks postpartum. ACE-Is are contraindicated in pregnancy because of toxic effects on the fetus. Methyldopa, labetalol, hydralazine, and nifedipine are safe to use in pregnancy. 64% of users answered correctly.

Which of the following statements regarding preeclampsia and eclampsia is correct? A A 34-year-old female G2P1 at 16 weeks, seen in the obstetric office on two consecutive visits with a blood pressure of 150/80, should be classified as mild preeclampsia. B After magnesium, intravenous ACE-Is are the next line therapy for hypertension in eclampsia. C Magnesium should be started in the ED on any patient in the third trimester with a blood pressure greater than 140/90. D Multiparity is a risk factor for preeclampsia. E Subcapsular liver hematoma is a potential complication of Hemolysis, Elevated Liver Enzymes, Low Platelet Count (HELLP) syndrome.

The correct answer is A. 8-year-old child pulled from an icy lake after a brief submersion; no cardiopulmonary resuscitation (CPR) was required on scene Explanation: Submersion victims requiring CPR on the scene or in the ED have a generally poor prognosis. Initial presentation and extent of required resuscitation are associated with the degree of anoxic insult. Asystole associated with warm water drowning is a particularly poor prognostic indicator. Rare reports describe recovery in both children and adults following icy water submersion; therefore prolonged resuscitative measures may be indicated in these patients. Victims who do not require CPR on the scene or in the ED usually have complete recovery. Patients who present asymptomatic with normal oxygenation and normal pulmonary exam after submersion injury may be observed for 4-6 hours and safely discharged home if there is no evidence of deterioration. 84% of users answered correctly.

Which of the following submersion injury patients has the best prognosis? A 8-year-old child pulled from an icy lake after a brief submersion; no cardiopulmonary resuscitation (CPR) was required on scene B 75-year-old man with Alzheimer's who was recovered from his neighbor's swimming pool after a witnessed fall and submersion; CPR was performed by bystanders and the patient remained unconscious on arrival to the ED C 16-year-old boy found floating in a swimming pool after drinking at a party; bystanders performed CPR and CPR was ongoing in ED D 30-year-old man with history of epilepsy who had a seizure in a hot spring and was asystolic on arrival of the paramedics E 2-year-old child who fell into a warm fresh water lake; bystanders performed CPR and patient was resuscitated by the paramedics

The correct answer is A. Bloody diarrhea and fever for 3 days in a 42-year-old female returning from South America. Explanation: Initial therapy in the majority of patients with food-borne illness focuses on treating dehydration. Occasionally, patients will benefit from antibacterial therapy. Patients with bloody diarrhea and fever often have invasive infections and may benefit from antibiotics. The caveat to treatment lies in the similarity of enterohemorrhagic E. coli 0157 (EHEC) to shigella. It produces a shiga-like toxin and may have similar symptoms. EHEC typically has an antecedent watery diarrhea for 2 days that develops into bloody diarrhea, as opposed to the more immediate and very frequent bloody diarrhea of shigellosis. Patients with EHEC also tend to be afebrile. Differentiation may be prudent as antibiotics during EHEC infection have a high correlation with hemolytic-uremic syndrome. Frequent nonbloody stools for less than 1 day are more likely due to rotavirus or noroviruses. Both may lead to severe dehydration necessitating intravenous therapy, but neither is abated by antibiotics. Diarrhea associated with vomiting and flushing suggests scombroid, which is best treated with antihistamines. Diarrhea after recreational freshwater exposure is often due to cryptosporidium. It is typically a self-terminating disease without complications in immunocompetent individuals. However, consideration for Giardia lamblia may be indicated. Related Topics: diarrhea food borne disorder 61% of users answered correctly.

Which of the following supports the use of antibiotics for diarrhea? A Bloody diarrhea and fever for 3 days in a 42-year-old female returning from South America. B A 9-year-old female with confirmed enterohemorrhagic E. coli 0157:H7. C An 11-year-old boy with frequent watery stools for 1 day with moderate decreased skin turgor. D A 68-year-old female with multiple episodes of watery diarrhea, vomiting, and diffuse blanching erythema after eating fresh tuna. E An otherwise healthy 17-year-old male with 2 days of diarrhea after swimming in a lake.

The correct answer is D. Neutrophil count greater than 250/μL in the peritoneal fluid Explanation: Ascitic fluid is tested for cell count, glucose and protein, Gram stain, and culture to identify bacterial peritonitis. A total white blood cell count greater than 1000/μL or a neutrophil count greater than 250/μL is diagnostic for spontaneous bacterial peritonitis. Low glucose or high protein values suggest infection. Gram stains and culture results can be falsely negative 30-40% of the time, and empiric antibiotics should be started in the ED based on clinical suspicion. Culture sensitivity is increased by using 10 mL of ascitic fluid per blood culture bottle. Additional studies of ascites that can help with inpatient evaluation are cytology, albumin, LDH, and tumor markers. 74% of users answered correctly.

Which of the following test results MOST accurately predicts spontaneous bacterial peritonitis? A Low serum glucose B High protein in peritoneal fluid C High protein in serum D Neutrophil count greater than 250/μL in the peritoneal fluid E Total white count greater than 500/μL

The correct answer is B. Giemsa stain showing intracytoplasmic inclusion bodies Explanation: This picture shows lesions of Molluscum contagiosum. These lesions may occur in single or multiple lesions. It is spread skin-to-skin contact and is most common in infants, sexually active adults, and immunocompromised individuals. A Giemsa stain of the keratotic core will show intracytoplasmic inclusion bodies or "molluscum bodies." These lesions usually resolve in 6-24 months without specific treatment. Tape may be applied after showers to prevent friction and spread of the lesions. Other treatments that could be prescribed by a dermatologist include topical cantharidin (blister beetle fluid) applied in the office or imiquimod (Aldara) every day three to five times per week at home to speed the resolution of the lesions. Donovan bodies are seen with granuloma inguinale. KOH prep is used to help visualize the hyphae seen in fungal etiologies of skin lesions. Serology is used for syphilis diagnosis. Tzanck smear is used to help diagnose herpes lesions. Related Topics: molluscum contagiosum 50% of users answered correctly.

Which of the following tests can be used to make the definitive diagnosis of this disease (Figure 3-3)? A Biopsy showing Donovan bodies B Giemsa stain showing intracytoplasmic inclusion bodies C Potassium hydroxide preparation (KOH prep) D Serology E Tzanck smear

The correct answer is C. Sulfonamides: neonatal kernicterus Explanation: Sulfonamides may cause neonatal kernicterus at near term. Valproate and other antiseizure medications have been associated with dysmorphic syndrome. Lithium can cause congenital heart disease specifically Ebstein's anomaly. Aminoglycosides are associated with ototoxicity. Tetracyclines can cause fetal teeth and bone abnormalities. 55% of users answered correctly.

Which of the following therapeutic agents is paired with the correct adverse effect in pregnancy? A Aminoglycosides: dysmorphic syndrome B Lithium: ototoxicity C Sulfonamides: neonatal kernicterus D Tetracycline: congenital heart disease E Valproate: fetal teeth and bone abnormalities

The correct answer is E. Sexual assault examination in a 6-year-old girl who was assaulted an hour ago Explanation: Emergent indications for procedural sedation and analgesia include cardioversion for life-threatening arrhythmias, neuroimaging for head trauma, reduction in fractures or dislocations with soft-tissue or vascular compromise, care of contaminated wounds, or intractable pain. Sexual assault examinations require less urgent sedation.

Which of these is NOT considered an indication for EMERGENCY procedural sedation? A Cardioversion for a wide complex tachycardia in a 78-year-old female with chest pain and dizziness B Intractable pain from a dislocated shoulder in a 34-year-old male C Obtaining a CT scan in an agitated 15-year-old football player who sustained a head injury D Reduction of an elbow dislocation in a 65-year-old female with an absent radial pulse E Sexual assault examination in a 6-year-old girl who was assaulted an hour ago

The correct answer is D. Patient's subjective report of pain Explanation: The patient's subjective reporting of pain, not the health care worker's impression, is the basis for pain assessment and treatment. There is poor correlation between vital signs such as tachycardia and tachypnea or changes in patient expression and movement, and the patient's report of pain. The amount of opioids required to treat pain can vary a great deal among different patients. Response to a fixed amount of pain medication does not correlate with the amount of pain an individual is experiencing.

Which of these should be relied upon in the assessment and treatment of pain? A Health care worker's impression B Patient's position, expression, and movement C Patient's response to a generous dose of an opioid analgesic D Patient's subjective report of pain E Vital signs

The correct answer is D. Staphylococcus aureus Explanation: Streptococci and staphylococci account for up to 90% of all cases of infective endocarditis. There has gradually been a change in the microbiology of infective endocarditis and currently S. aureus is thought to be the most common pathogen of native-valve endocarditis in both injection drug users and nonusers. Enterococcus and coagulase-negative staphylococcus are the next most common after staphylococcus and streptococci. The rest of the cases in noninjection drug users are usually attributable to the HACEK group, Bartonella species, and Coxiella burnetti. 53% of users answered correctly.

Which pathogen is the MOST common cause of infective endocarditis in the industrialized world? A Bartonella spp. B Eikenella corrodens C Enterococcus spp. D Staphylococcus aureus E Streptococcus viridans

The correct answer is C. 24-year-old Caucasian male athlete Explanation: Certain patients including minorities, those at the extremes of age, and those with diminished cognitive function are more at risk for inadequate pain control. In addition, there is also a tendency to undertreat pain in women.

Which person is most likely to receive adequate pain control in the emergency department? A 2-year-old Caucasian male B 18-year-old African American female college student C 24-year-old Caucasian male athlete D 30-year-old Caucasian male with retardation E 80-year-old Hispanic grandmother

The correct answer is B. Lidocaine: overdose can cause refractory seizures. Explanation: Lidocaine in high doses can cause refractory seizures. Benzocaine generally has a long duration of action. High doses of benzocaine or prilocaine, not lidocaine, may lead to methemoglobinemia. Ropivacaine is significantly less cardiotoxic than other agents. The use of local anesthetic with epinephrine appears to be safe for use in end-arterial fields (fingers, toes, etc.) in selected healthy patients.

Which problem is associated with the CORRECT local anesthetic? A Benzocaine: may wear off before long procedures are completed. B Lidocaine: overdose can cause refractory seizures. C Lidocaine: overdose can cause methemoglobinemia. D Lidocaine with epinephrine: may cause ischemia of a digit in an otherwise healthy patient. E Ropivacaine: highest risk of cardiotoxicity.

The correct answer is D. Typhoid fever Explanation: Vaccination history is extremely important when discerning the cause of an illness in anyone, let alone an immigrant or returning traveler. The vaccinations for hepatitis A and B and yellow fever are very effective and nearly 100% effective if neutralizing antibodies are present. Rabies vaccination requires multiple doses and requires further management after an exposure, but, if following the guidelines, is also extremely effective. The vaccine for typhoid fever is highly recommended, as it is widely prevalent in the developing world and is easily transmitted by contamination of food with feces or urine. However, the vaccine is only 75% effective. 41% of users answered correctly.

Which travel-related disease can still be commonly acquired despite proper vaccination? A Hepatitis A B Hepatitis B C Rabies D Typhoid fever E Yellow fever

The correct answer is C. Inguinal Explanation: Seventy-five percent of all hernias occur in the inguinal region making it the most common form of hernia, with two-thirds of these being of the indirect type. Although there is a clear male predilection, inguinal hernias are also the most common hernias in women. Related Topics:herniahernia, inguinal 45% of users answered correctly.

Which type of hernia is the most common in women? A Epigastric B Femoral C Inguinal D Spigelian E Umbilical

The correct answer is C. Oxygen saturation Explanation: It is recommended that oxygenation be monitored continuously for all levels of sedation. Pulse, respiration, and end-tidal carbon dioxide should be monitored continuously for moderate to deep sedation, and blood pressure should be monitored every 5 minutes in these cases. For minimal sedation, heart rate, respiratory rate, and blood pressure be monitored every 15 minutes with blood pressure readings after medication boluses as well.

Which vital sign is recommended to be continuously monitored for all levels of sedation, from minimal to deep? A Blood pressure B End-tidal carbon dioxide (capnography) C Oxygen saturation D Pulse E Respiratory rate

The correct answer is E. Vasovagal syncope Explanation: This patient's report of lightheadedness and the prodrome of pallor and sweating after viewing an intense visual stimulus suggest vasovagal syncope, rather than orthostatic hypotension. Brief, tonic-clonic, or asynchronous movements may occur with any form of syncope. The duration and intensity of movements during grand-mal seizures often causes muscle pain. The patient's rapid recovery also makes the diagnoses of hypoglycemia or seizure unlikely. Although an acute dysrhythmia might not be entirely excluded in this scenario, it was not the most likely cause in a previously healthy young person. 82% of users answered correctly.

While observing the arrival of a trauma patient with an open tibia fracture, an otherwise healthy 22-year-old male hospital volunteer states he feels lightheaded. He appears pale and diaphoretic and slowly sinks to the floor. Fortunately, an observant nurse prevents him from striking his head. He is unresponsive for 10 seconds, has a brief tonic-clonic movement but spontaneously awakens, and states "That was weird." A brief neurologic exam is nonfocal. Which of the following is the MOST likely cause of his condition? A Dysrhythmia B Grand-mal seizure C Hypoglycemia D Orthostatic hypotension E Vasovagal syncope

The correct answer is D. Metabolic alkalosis with respiratory acidosis Explanation: It is important to recognize that the pH is the least important component of an ABG in assessing a patient's acid-base status. Acidosis is any process that increases the H+ concentration. Any condition that decreases it is called an alkalosis. Acid-base disorders are further separated into respiratory or metabolic. Respiratory disorders represent changes in the pCO2, whereas metabolic disorders represent changes in the HCO3. This patient has an elevated HCO3 and a mildly elevated pCO2, which represents the poorly understood relationship between pCO2 in the alveoli and H+. Metabolic alkalosis is classified as either chloride sensitive or chloride insensitive. 61% of users answered correctly.

You are called to see a 35-year-old liver transplant patient on immunosuppressant therapy who has been admitted for vomiting and diarrhea but is boarding in the ED. The nurse is concerned because the hospitalist ordered a blood gas and she cannot get a hold of them to address the abnormal values. The patient is hemodynamically stable and has received 2 L of NS. The blood gas shows pH 7.44, pCO2 48, HCO3 32.6, base excess 7.3, pO2 100. What is the patient's acid-base status? A Metabolic acidosis B Metabolic acidosis with respiratory alkalosis C Metabolic alkalosis D Metabolic alkalosis with respiratory acidosis E Normal acid-base status

The correct answer is C. Begin infusion of norepinephrine 5 μg/min IV. Explanation: Early goal directed therapy in severe sepsis is guided by an algorithmic approach (Figure 10-9). After initial supplemental oxygen or intubation, fluid resuscitation, and broad-spectrum antibiotics, patients must be repeatedly reassessed for response to therapy. Target values include a mean arterial pressure >65 mm Hg, central venous pressure of 8 mm Hg, venous oxygen saturation of 70%, and urine output >0.5 mL/kg/hr. This patient demonstrates persistent hypoperfusion after initial fluid resuscitation with a mean arterial pressure of 52 mm Hg (mean arterial pressure is approximated by diastolic plus one-third of the pulse pressure). Although many patients have large fluid deficits, a central venous pressure of 11 mm Hg suggests adequate resuscitation. The next step in management is inotropic support, and norepinephrine or dopamine is the typical agent of choice for initial management. Dobutamine may be used in select settings with invasive monitoring, but typically not until after dopamine administration. The patient should be transfused until the hematocrit is greater than 30%, but would not be indiscriminately transfused unless anemia and severe blood loss were obvious.

You are treating an obtunded 72-year-old female for severe sepsis due to pneumonia. You have administered 3 L normal saline and broad-spectrum antibiotics. After performing endotracheal intubation, you place a central venous oxygen saturation catheter and arterial line in preparation for early goal directed therapy. The patient's blood pressure is now 88/34, central venous pressure is 11 mm Hg, and venous oxygen saturation is 66%. Lab values are not yet available. What is the MOST appropriate next step? A Administer 1 L normal saline intravenously. B Begin infusion of dobutamine 0.5-1.0 μg/kg/min IV. C Begin infusion of norepinephrine 5 μg/min IV. D No further actions, the endpoints of early goal directed therapy have been met. E Transfuse 2 units packed red blood cells.

The correct answer is B. Laryngospasm and arrest of respiration Explanation: This classic description of muscular rigidity is highly suggestive of tetanus. Initial symptoms may be described as jaw pain, dysphagia, or generalized muscular rigidity or cramping. This can progress to generalized spasticity as described, and can lead to rhabdomyolysis and lactic acidosis. It can involve laryngospasm and can lead to respiratory arrest. Early attention to airway management is prudent, as nearly two-thirds of cases require intubation. Tetanus may eventually lead to infectious complications due to a protracted course, but it does not typically involve overwhelming sepsis early in the disease. Although the central nervous system is involved, permanent injury can be avoided if hypoxia is adverted. Person-to-person transmission of tetanus does not occur. The presentation of rabies involves mental status changes, hyperactivity, and may include paralysis rather than spasticity. It is typically fatal even with immunoglobulin administration after development of the disease. Of note, this presentation also mirrors strychnine poisoning, which can be a contaminant in heroin or cocaine. Treatment for strychnine is supportive. 71% of users answered correctly.

You are volunteering in Haiti in the ED and see a patient who appears to be seizing. He is a 30-year-old male who is brought in by his family because of jaw pain and muscle spasms. You also notice many small abscesses with central necrosis on his upper extremities. He is arching his back, flexing his arms, clenching his fists, and extending his lower extremities, but appears to be awake and in pain. What complication should you anticipate? A Death unless treated with rabies immunoglobulin B Laryngospasm and arrest of respiration C Overwhelming sepsis unless treated with penicillin D Permanent central nervous system injury E Spread by respiratory droplets

The correct answer is E. Prolonged QT syndrome. Explanation: Quinidine is a class 1A antiarrhythmic. Cardiac sodium channel blockade can lead to prolonged QT syndrome, and all patients receiving quinidine intravenously must be on cardiac monitor. Quinidine and especially quinine cause a hyperinsulinemia that can cause profound hypoglycemia, in addition to that caused by P. falciparum itself. Primaquine causes massive hemolysis in patients with glucose-6 phosphate deficiency. Both chloroquine and primaquine can cause methemoglobinemia. Artemesin-derived antimalarial agents are relatively safe and more effective but are currently only available as artesunate through Centers for Disease Control supplies only. 53% of users answered correctly.

You have a high suspicion of malaria caused by P. falciparum in a toxic appearing patient. You initiate therapy with intravenous quinidine. Which of the following is a complication of therapy? A Hemolysis in the setting of glucose-6-phosphate dehydrogenase deficiency. B Hyperglycemia. C Methemoglobinemia. D None, it is relatively safe therapy. E Prolonged QT syndrome.

The correct answer is A. Acute optic neuritis if you notice that there is a Marcus Gunn pupil and the posterior chamber is clear Explanation: A normal exam with a Marcus Gunn pupil suggests optic neuritis or other causes of afferent papillary defect. Initial findings of optic neuritis may include problems with color vision and visual field defects. Acute retinal artery occlusion is monocular. Hysterical blindness is suggested when nystagmus does occur when confronted by moving vertical lines. Occipital stroke causes hemianopsia. Uveitis is almost universally painful and associated with photophobia. It may cause blurry vision but not usually visual loss. 68% of users answered correctly.

You pick up a patient's chart that states "acute blindness for 1 hour with pain." Which of the following diagnoses is likely the cause of this patient's symptoms? A Acute optic neuritis if you notice that there is a Marcus Gunn pupil and the posterior chamber is clear B Acute retinal artery occlusion secondary to carotid cholesteatoma if visual loss is bilateral C Hysterical blindness if the patient exhibits no lateral nystagmus when vertical lines are passed before the eyes D Occipital stroke E Uveitis

The correct answer is B. Norovirus Explanation: Outbreaks of viral gastroenteritis are most likely due to norovirus. In addition to the consumption of contaminated food and water, noroviruses are easily transmitted person to person. Rotavirus is a major cause of diarrheal illness in infants and children. Ciguatera toxin comes from infected reef fish such as barracuda. Symptoms typically are neurologic and they include paresthesias, hypesthesias, and reversal of hot/cold sensation. Enterotoxigenic E. coli is the most common cause of traveler's diarrhea but is typically not associated with outbreaks on cruise ships. Shigella infections are often associated with fever and bloody diarrhea. 70% of users answered correctly.

You volunteered to be a physician for a Mediterranean cruise and are enjoying your much deserved vacation until the third day when dozens of patients and cruise members present to you complaining of nausea, vomiting, and nonbloody diarrhea. Which of the following is the MOST likely culprit? A Rotavirus B Norovirus C Ciguatera D Enterotoxigenic E. coli E Shigellosis


Related study sets

Human Anatomy and Physiology 1 HOMEWORK 15

View Set

APUSH Unit 5 - Antebellum Period

View Set

Psychology, Module 41. Social Thinking

View Set

Chapter 24: The New Deal, 1933-1939

View Set